Legal Writing Legal Method 7e TM

You might also like

Download as pdf or txt
Download as pdf or txt
You are on page 1of 263

TEACHER'S MANUAL

A Practical Guide
to Legal Writing
and
Legal Method

Seventh Edition

John C. Dernbach
Cathleen S. Wharton
Catherine J. Wasson
Copyright © 2021 John Dernbach.

This Manual is made available as a courtesy to law teachers with the understanding that it will
not be reproduced, quoted, or cited, except as indicated. Only teachers who have adopted the
book may incorporate portions of this Manual into their class notes or presentation slides for
class, but no portion of this Manual may be distributed to students. Sale or distribution of this
Manual is strictly prohibited. In the event that anyone would like to cite the Manual for
thoughts drawn from it, a reference to the relevant page number of the materials text (with the
formula “suggested by”) is appropriate. For information about permissions or to request
permissions online, visit us at www.aspenpublishing.com.

To contact Customer Service, e-mail customer.service@aspenpublishing.com,


call 1-800-950-5259, or mail correspondence to:

Aspen Publishing
Attn: Order Department
PO Box 990
Frederick, MD 21705

i
ACKNOWLEDGMENTS

We thank the thousands of students who constantly inspire us to find new and better ways
to teach.

SUGGESTIONS OR COMMENTS

If you have suggestions or comments about the text or this teacher’s manual, or about
teaching with these materials, please contact us:

John C. Dernbach jcdernbach@widener.edu


Cathleen S. Wharton wharton1@uga.edu
Catherine J. Wasson cwasson@elon.edu

ii
CONTENTS

INTRODUCTION TO THE SEVENTH EDITION ............................................................. 1

TEACHING WITH THE SEVENTH EDITION.................................................................. 3

Guiding Principles .......................................................................................................... 3


Using the Exercises ........................................................................................................ 4
Guide to Chapter Examples and Exercises ..................................................................... 5
Using the Appendices ..................................................................................................... 5

CHAPTER NOTES

Part A – Introduction to Law ................................................................................................ 7


1. Rules and Policies ..................................................................................................... 8
2. Sources of Law ....................................................................................................... 10
3. Case Analysis and Case Briefs ............................................................................... 12
4. Precedent and Stare Decisis ................................................................................... 17
5. Reading and Understanding Statutes ...................................................................... 21

Part B – Basic Concepts of Legal Method .......................................................................... 24


6. Understanding Legal Rules .................................................................................... 25
7. Identifying and Selecting Issues for Analysis ........................................................ 39
8. Common Law Analysis .......................................................................................... 52
9. Statutory Analysis................................................................................................... 67
10. Reaching a Conclusion ........................................................................................... 80

Part C – Basic Concepts of Legal Writing.......................................................................... 89


11. Organization ........................................................................................................... 90
12. Describing the Law ............................................................................................... 105
13. Applying the Law ................................................................................................. 112
14. Drafting the Analysis ............................................................................................ 116
15. Revising and Editing ............................................................................................ 124

iii
Part D – Office Memoranda and the Opinion Letter ...................................................... 129
16. Objective Writing ................................................................................................. 130
17. Elements of a Summary-of-the-Law Memorandum ............................................. 134
18. Elements of an Analytical Legal Memorandum ................................................... 138
19. Opinion Letters ..................................................................................................... 148

Part E – Briefs and Oral Argument .................................................................................. 150


20. Elements of a Brief ............................................................................................... 151
21. Structure of an Argument ..................................................................................... 156
22. Persuasive Writing Techniques ............................................................................ 166
23. Write to Your Audience ....................................................................................... 169
24. Preparing and Presenting an Oral Argument ........................................................ 175

SUPPLEMENTAL MATERIAL
Course Planning ................................................................................................................... 181

Course Goals and Learning Objectives ...................................................................... 181


Syllabus Design .......................................................................................................... 187

Skills Development and Assessment of Student Learning .............................................. 194

Skills Development..................................................................................................... 194


Assessment of Student Learning ................................................................................ 197

APPENDICES
ABOUT THE APPENDICES ............................................................................................. 200

Appendix I – Annotated Discussions................................................................................ 202

Appendix II – Assessment Tools ...................................................................................... 232

Appendix III – Other Teaching Aids ............................................................................... 247

iv
INTRODUCTION TO THE
SEVENTH EDITION

The seventh edition of A Practical Guide to Legal Writing & Legal Method retains the
practical, process-oriented approach to teaching Legal Writing and Legal Method that has been
the hallmark of this text since the first edition of the book was published in 1981. Accordingly, the
structure and purpose of the book remain unchanged. We continue to offer students a
straightforward guide to the essential skills of legal reasoning, analysis, and communication that
they will use every day in their practice of law. The seventh edition, however, continues our
commitment to expand and update the material to suit the needs of a new generation of law students
and law professors.
For those of you who have used earlier editions of the book, major changes from the sixth
edition are highlighted below. More detailed information appears at the beginning of each chapter.
For those of you who are new to the book, we believe that you will find a wealth of ideas to help
you become the best teacher that you can be. Like its predecessors, the seventh edition is easy to
teach with. It does not require teachers to adopt a particular teaching style or adhere to a specific
pedagogical philosophy. Rather, it provides a flexible framework that encourages teachers to make
creative, thoughtful choices about their work in the classroom, and allows legal writing professors
to amend, supplement, and expand each lesson to meet the needs of their students. Whether you
are a new or a returning user of A Practical Guide, we hope that the seventh edition provides
your students with the opportunity to learn and practice the skills they need and that this Teacher’s
Manual supports you in your teaching.

Major Changes from the Sixth Edition

Coverage

The seventh edition does not include any new chapters. Instead, we have streamlined and
reorganized the material to consolidate related topics, thereby making the text more accessible for
both student and teacher. If you have used the book previously, you will find that although
related topics have been integrated the coverage of the text has not changed. It still includes all of
the rules, examples, and exercises included in the sixth edition.

Exercises

Since its publication, A Practical Guide has been valued for the number and variety of
thoughtfully-designed exercises contained in each chapter. The seventh edition continues that
tradition, retaining our large selection of exercises. Students should be able to complete nearly all
of the exercises outside of class in 60 to 90 minutes each. In a few cases (e.g., exercises requiring
drafting of a discussion or argument), the time required out of class may be longer, depending on
the level of in-class preparation. Many of the exercises can also be completed in class or
assigned as either individual or group exercises. All exercises have been reviewed to ensure that
each one gives students the opportunity to practice specific skills taught in the text. When
appropriate, the exercises have been edited to focus students’ efforts on the most important skills.

1
Appendices

The appendices still include examples of each type of document addressed in the text. The
citations have been updated where necessary to reflect the current correct citation for the material
cited. In addition, there is an expanded Appendix A that now provides several examples of email
memos of various lengths, in order to address the changing face of law practice and the need to
respond to queries both quickly and professionally.

Teacher’s Manual

As in past editions, the teacher’s manual for the seventh edition alerts readers to
significant changes in the text and includes suggested approaches to the exercises. For the first
time, however, we have created a section dedicated to teaching, course design and syllabus
development. This section includes new sample syllabi and course objectives for everything from
a one-credit introductory course to a three-term legal writing course. It also addresses teaching
methods and assessment, gathering together and expanding on material from the sixth edition
teacher’s manual with several new ideas and samples. The new section specifically references
ABA standards on experiential learning and student assessment, to help teachers design a course
that demonstrably responds to the ABA’s curricular requirements and priorities. If you are a
novice teacher, this new section will allow you to design a course knowing that the sample
lessons and materials have been used successfully in the classroom. If you are an experienced
teacher, this new section may inspire you to refresh and update your course.

2
TEACHING WITH THE
SEVENTH EDITION

Guiding Principles

The best part of teaching with A Practical Guide is that the book works. From week to
week, month to month, we have seen substantial (if not always consistent) progress from the
overwhelming majority of our students. At the end of the year, they are much better writers
than they were at the beginning.

Teaching with A Practical Guide is based upon three pedagogical principles:

1. Legal writing is taught most effectively when it is taught together with legal
method. Writing about a specific hypothetical problem gives students an
extended opportunity to think deeply about that problem. Most of the
hypothetical problems in other first- year classes are answered orally and
during an individual class; writing an answer over a period of days and weeks
reinforces and deepens students’ ability to think like lawyers. Legal writing
is not English composition for lawyers. Rather, it is perhaps the only course
that brings together the entire law school curriculum.

2. Legal writing and legal method involve at least several dozen discrete basic
skills. These skills are best taught several at a time and sequentially, with the
most fundamental skills first.

3. These skills are learned best by practicing them in exercises. A considerable


amount of research, and much experience with the book, indicates that people
learn more by doing than by simply listening or taking notes. The ABA
curriculum requirements regarding feedback on student writing reflect this.

Although we care deeply about these principles, there is no one right way to teach with
the book. Each teacher enriches A Practical Guide with his or her own experience, ability, and
personality. For the beginning legal writing teacher, this manual provides specific suggestions.
For the experienced teacher, this manual provides points of departure.
Just as there is not only one way to teach with the book, there is not only one correct
answer to every exercise. The manual reflects the points of view of its several authors.
Suggested answers and approaches to exercises, therefore, reflect differences in emphasis,
teaching style, and professional experience. Moreover, when an exercise requires a student to
draft a statement of facts, discussion, or other element, the manual shows preliminary steps but
may not provide an example of the final written product. The suggested answers or approaches
in the book are not necessarily the only answers or approaches. In many exercises, you might
analyze or resolve a problem somewhat differently than we have here.
Finally, the book shows a good deal of our writing. We encourage you to show students
other writing (including yours or theirs). Reading the writing of others – from the best to the
worst – is an essential step in learning to be a good writer.

3
Using The Exercises

In our experience, the principles, explanations, and examples in the chapters require
little or no additional explanation in class. Students usually have no difficulty understanding
what they are reading. As a result, we spend a few minutes at the beginning of class answering
questions or emphasizing one or two points, but we do not lecture from the book. Instead, we
spend our time where the students are likely to experience problems—applying what they have
read to specific situations. You might want to spend some class time working through some of
the detailed examples in the chapter(s) assigned. Most of the class time with this book,
however, involves the exercises.
There are many exercises from which to choose. The book contains more than seventy-
five exercises; with a little imagination, you can create many more. The exercises are based on
several dozen factual situations. You could take a factual situation illustrating one skill (such as
common law analysis) and use it again to illustrate another skill (such as writing questions
presented in a memorandum). In some cases, the book uses the factual situations that way, but
in many cases it does not. You can also use the ideas behind the exercises to create your own.
(For users who want to track a particular problem, the “Guide to Chapter Examples and
Exercises” on page 5 of this teacher’s manual shows where various factual problems are used
throughout the book.)
Work through an assigned exercise yourself before you go over it in class. This manual
contains answers to some of the exercises and a method of approaching others. It should go
without saying that you should not use that as a substitute for your own preparation of the
exercise. It is difficult to either present an exercise or to answer some questions unless you have
worked through the exercise beforehand.
As you work initially through an exercise, decide what you want to emphasize. Each
exercise can be taught in different ways and can be used to focus on different points. In
addition, decide how the exercise can be made most useful for the writing problem that you have
assigned. The exercises are not a substitute for memo- or brief-writing assignments. However,
they can and should be used to give students practice in the skills they will need to demonstrate
on those assignments. The ability to transfer a skill from one project to another will serve
students well in their law study and in practice.
Similarly, when you assign an exercise, emphasize to students the importance of
working through the exercise before class. We suggest to students that they come to class with
their notes from this work and that they have prepared a written work product for the
assignment. Even though they will be well rewarded for the time they spend working on an
exercise out of class, students sometimes try shortcuts of simply reading the exercise over or
making a few margin notes. Decide what standard you expect for each assignment; your
expectations need not always be the same, but they should be clearly communicated to your
students and the consequences for not participating as assigned should be clear.

4
Guide to Chapter Examples and Exercises

Teacher’s
Problem Examples & Exercises
Manual
Aggravated Menacing Example used in Chapters 6, 12, & 17
McKay Exercises 7-A, 10-A, 12-A, 13-A, 16-A Appendix I.A
Banner-Patriot Exercises 7-B, 10-B, 12-B, 13-B, 16-B
Hughes Exercises 7-C, 10-C, 12-C, 13-C, 18-B
Dooley Example used in Chapters 8 & 14; Exercise 8-A
Quale Exercises 8-B, 14-B, 17-A, 19-B, 23-C
Greenleaf Exercises 8-C, 14-C, 17-B, 23-E Appendix I.B
Smith Exercises 9-A, 14-D
Ejusdem generis Example used in Chapter 9; Exercise 9-B
Welfare League Exercises 9-D, 14-E, 23-D
Ferguson Exercises 11-A, 16-C
All-Rite Exercises 11-B, 16-D, 18-C, 18-G
Langford Exercises 12-D, 13-D, 17-C
Watson/
Exercises 14-A, 19-A Appendix II.B
Worthington
Vosberg Example used in Chapter 18; Exercise 18-A
Magaro/Shannon
Example used in Chapter 20; Exercise 20-A
Development
Jones Exercises 21-A, 22-A
Hansen/Tri-State Exercises 21-B, 22-B, 24-B, 24-C, 24-D, 24-E
Example used in Chapters 23 & 24; Exercises 23-A,
Chisolm
23-B, 24-A, 24-C, 24-D

Using the Appendices

The appendices in the text are good teaching tools, particularly as students are writing
memos or briefs, because they show the entire document. The negligent misrepresentation
problem in the appendices is essentially an elements problem. This problem is very much like
those that first-year students encounter in their other courses.
Additional appendices are included in the teacher’s manual. These include annotated
sample discussions based on some of the exercises in the text, annotated versions of the
summary-of-the-law memoranda and the analytical memo discussion in Appendices B and C
of the text, sample student assessment tools, and some additional teaching aids.

5
CHAPTER
NOTES

6
Part A
Introduction to Law

7
1
Rules and Policies

Changes from the Sixth Edition

There are no major changes from the sixth edition.

Teaching Notes

This chapter is a good introduction to law school in general and legal writing in
particular. We have found that the chapters in Part A can also be useful in preliminary or
orientation classes for first-year students.

Suggested Approach to Exercises

These exercises work well as a basis for class discussion during the first or second
class. The discussion is often lively. Assigning written answers to one of the exercises after
class discussion is an easy introduction to legal writing because students are writing about ideas
already explored in class.

Exercise 1-A

1. Because the questions involve the extent to which a government ought to regulate the
activities of its citizens, the answers should be consistent with the balance that the
students strike between the common good and individual liberty. Encourage students to
develop these questions:

 Is there a proven connection between the harm and the remedy? (Will the remedy
work?)
 Is the harm to be prevented great enough to justify the intrusion on individual liberty?
 Is the harm to be prevented great enough to justify the cost to enforce it?
 Is the remedy practical, in the sense that it will have sufficient public support to be
enforceable?

2. One way to explore consistency is to ask students questions about the views implicit in
their answers to 1. Their “gut” views about the bills in 1 may reveal a more complex
understanding of freedom and the social good than they have previously stated. Some
students will argue, for example, that freedom from wearing motorcycle helmets is
different from and less important than the freedom from government-sponsored testing
before getting married.

3. This question is intended to encourage students to think about the tension between
philosophical consistency and political necessity. In a democratic society, the public
(and therefore the voters) may not want consistency. Legislators are expected to be
principled and responsive—a tension that is not always easy to resolve. Judges are
expected to be principled but are not expected to respond to public demands. Thus, it is
probably more important that judicial decisions be consistent.
8
Exercise 1-B

1. (a) No. Price’s guilt or innocence does not depend on what crimes the other
men committed.

(b) No. That other people violated the same law does not excuse Fong’s violation.
Even though her arrest might seem arbitrary, it is related to the purpose of the
law, which is to enforce speed limits on the highway.

(c) No. That other people violated the same law does not excuse Hyde’s violation,
even though she was arrested “at random.” Her arrest “as an example to others”
is consistent with the purpose of discouraging the use of marijuana.

(d) Probably. It is no defense that the cohabitation law has not been enforced
against others in recent years. However, if Gilman can show that her arrest was
not made in good faith but in an attempt to intimidate her into silence, she has a
valid defense. On these facts, she can probably make that showing.

2. It is no defense that the defendant was arrested at random, that other persons breaking
the same law at the same time were not arrested, or that the law had not been enforced
in the past, unless the defendant can show intentional discrimination in enforcement.
See the cases listed in the Bibliography.

9
2
Sources of Law

Changes from the Sixth Edition

There are no major changes from the sixth edition.

Teaching Notes

This chapter is the cornerstone of much that you will be teaching. We recommend
assigning it early as an overview of our system of laws and coming back to it as you teach
specific principles of legal method. If you also teach legal research, this chapter is a good
starting place.
When going through the exercises, students will have no difficulty recognizing binding
authority and secondary authority, but they will not agree on the relative merits of persuasive
authority. This will give you an opportunity to discuss the value of each source. Realistically,
they might use all of the cases listed.
Students may also have difficulty when they have a larger number of cases to choose
from and cannot decide which ones to use. Before the first research memo, suggest the
following guidelines:

In general, choose cases as follows:

(1) For a question of state law, prefer state court over federal court decisions.
(2) Prefer a case with similar facts over one with dissimilar facts.
(3) Prefer a case in which the court addressed the specific issue in your case over
one where the issue was more general.
(4) Prefer decisions of a state’s highest court over decisions of its middle level
appellate court.
(5) Prefer a case with a well-reasoned basis for the decision over one where the
rationale is superficial or nonexistent.

Balance these preferences with your own judgment and common sense. For example, a well-
written and well-reasoned federal district court opinion on a question of state law might be
more useful than a state court decision on the same issue. If you have kept an open mind during
your research, the cases most helpful to you in analyzing and resolving the issues will usually
be the ones most helpful to the reader.

Suggested Approach to Exercises

Exercise 2-A

Category (A)—primary authority that is binding: 1, 5.


Category (B)—primary authority that is persuasive: 3, 4, 6, 7, 8.
Category (C)—secondary authority: 2.

10
1. Sources 6 and 7 are most persuasive for two reasons: (1) This is a matter of state law,
and the court will look at how other state courts have resolved a similar issue of state law
before they look at federal court decisions. (2) They are decisions by the highest court
of the respective states.

2. Source 3 is better authority than source 4, the federal case, because it is a decision by a
state court on a question of state law. The federal court in Source 4 necessarily based its
decision on a prediction of how the highest court in that state might rule on the identical
issue.

3. Source 8 has the least precedential value because it is a minority opinion, not a holding.

Exercise 2-B

Category (A)—primary authority that is binding: 7.


Category (B)—primary authority that is persuasive: 3, 4, 5, 8, 9.
Category (C)—secondary authority: 1, 2, 6.

This exercise presents a situation in which many of the decisions on a question of


contract law, traditionally the province of the states, have been decided by federal courts trying
to ascertain state law. This exercise also emphasizes a concept that students sometimes lose
sight of—that federal courts cannot make state law. Sources 8 and 9 are examples of the same
court reaching different conclusions based on the state law it was construing.

1. Sources 3 and 5 are both highly persuasive. Source 3 is a case nearly on point on a
matter of state law decided by the highest court of another state. Source 5 is an opinion
from a federal trial court judge applying the law of your state. A state court would be
very interested in how a federal court applied that state’s law.

2. Sources 8 and 9 have some persuasive value because they are cases in which the federal
court had some state law upon which to base a decision, though Source 9 is slightly
more persuasive because the court was relying on a decision by the state’s highest court.

3. Source 4 has less precedential value than the other sources because it is an opinion from
a federal judge deciding an issue without any guidance from the courts in that state.

This exercise serves as a good example of the importance of developing an overview of


the law before narrowing your research to the most persuasive sources. Explain to students that
only after reading the secondary sources and some of the cases would they have discovered the
courts disagree about the application of a pollution exclusion clause - whether it applies only to
traditional environmental pollution, like industrial discharge of pollutants into the air and
water, or whether it also applies to a contaminant released in an enclosure. This distinction
should focus their research on cases involving indoor releases, such as those listed in the
exercise.

11
3
Case Analysis and Case Briefs

Changes from the Sixth Edition

There are no major changes from the sixth edition.

Teaching Notes

While we have suggested answers to the exercises, you may want to emphasize that
there is room for discussion about some of the elements. The purpose of any case brief is to
provide the person writing it with a concise and accurate summary of the case. This can only be
accomplished if the writer learns to read cases carefully and think about them critically.
You can encourage students to focus on the substantive content of a case rather than on
the format of their case brief by pointing out that the text does not instruct them to develop the
elements of a brief in the order in which those elements will appear in the final product. Rather,
the text tells students to identify the fourth element, the holding, first. Students then work
backwards from the holding, identifying the essential elements of the case as they go. We
encourage each student to develop a style of case briefing that he or she finds most helpful.
Students’ increased use of computers makes it much easier for them to construct a case brief in
the order suggested in the text, or in any other order that suits them, even though the elements
of the final brief are typically presented in a different order.
Several elements of a case brief can be difficult for students. For example, if the court
does not explicitly identify them, it is not always clear which facts are legally significant. It can
be helpful to point out that courts often reiterate the facts that they deem most significant either
immediately before or immediately after the holding. Also, students may find it helpful to list
the legally significant facts and the procedural history of the case separately, as illustrated by
the alternative case briefing format and sample brief below. Another difference in the
alternative format is that it requires students to include both the significant facts and the legal
question in the issue statement (thus anticipating the questions presented element in a memo, as
explained in Chapter 19), so that the holding can be stated as “yes” or “no.”
The difference between rules and reasoning or policy may be particularly difficult for
students to discern. Generating a discussion about what your students included in their briefs,
where they placed certain information and why, and how their brief compares with those of
their classmates enables you to illustrate how two different answers may both be accurate and
helpful. Explaining their answers will also help the students to define and understand these
elements more clearly.
Some teachers find it helpful to have students brief longer cases after briefing some or
all of the cases in these exercises. Longer cases that we have found useful for that purpose
include those cited in the Bibliography for Exercises 14-A and 15-A. One advantage of using
these or other cases in the bibliography for case briefing is that students will be more familiar
with the law when they subsequently do the exercises. The materials in the “closed” memo
projects you might assign fairly early in the semester can also be used to practice case briefing.
Since these case briefs also provide a foundation for their understanding of the law, the obvious
benefit of this work can be a powerful motivator for students.

12
Emphasize that, although all lawyers must be able to identify the essential elements of a
case, there is more than one format for a case brief. Some lawyers combine the facts and
procedural history; others do not. Some combine the holding and the disposition; others do not.
Some put the parts of the brief in a different order. Giving your students an example of an
alternative format like the one below can encourage them to focus on the content of the brief,
rather than on its formal structure.

Alternative Format for a Case Brief.

I. Heading. Include the name of the case, the court that decided the case, and the date.

II. Facts. Include all facts directly relevant to the issue. Relevant facts include the
relationship of the parties to each other and a summary of the occurrences giving rise
to the litigation. It saves time to use abbreviations such as P for plaintiff, D for
defendant, TC for trial court, and so forth.

III. Procedural history. Explain how the court below decided the case and why the
decision was appealed.

IV. Rule. State the rule–the legal principle that applies to the facts of the case.

V. Issue or issues. State the substantive question or questions the appellate court decided.
A statement of an issue includes (a) the point of law in dispute and (b) the key facts
relating to that point of law.

VI. Holding. The holding is the court’s answer to the question presented in your
issue statement. If your issue statement is complete, the answer can be “yes” or
“no.”

VII. Disposition. State whether the court affirmed or reversed the lower court’s decision
and whether the appellate court remanded the case to the trial court for further
proceedings.

VIII. Reasoning. Explain the reasoning of the court in reaching its decision.

IX. Comments. This section is for your benefit. Include here any points you want
to remember that were not included in other sections.

A final note about teaching this chapter: One of the authors assigns the first half of
Chapter 6 (Understanding Legal Rules) very early in the fall term so the lessons from that
chapter can be used when discussing the rule portion of a case brief. (The remainder of the
chapter is assigned later, when students begin to study rule synthesis.)

13
Alternative Format for Sample Case Brief

State v. Phillips
Facts: D set fire to a building and the building burned to the ground. A fireman
on his way to the fire was killed when he fell off the truck and was run
over by a car. The driver of the car was speeding and following too
closely.

Procedural History: D was convicted of arson, which is a felony, and felony murder. She
appealed the felony murder conviction.

Rule: If someone is killed during the commission of a felony, the perpetrator


of the felony is guilty of felony murder.

Issue: Is D guilty of felony murder when a fireman fell off a truck and was run
over by a speeding motorist while the fireman was on his way to the
scene of an arson that D had committed?

Holding: No.
Judgment: Reversed.
Reasons: The purpose of the rule, which is to deter people from committing
felonies inherently dangerous to human life, would not be served in this
case. D could not foresee that an intervening force, a third party, would
cause the death of a fireman. There must be a causal connection between
the felony and the death.

Comment: The court noted that because felony murder borders on strict liability, the
scope of the crime should not be expanded beyond its intended purpose.

14
Suggested Approach to Exercises

Exercise 3-A
Toad v. Ulrich (2002)

Facts: P sells wooden stools called “Toad Stools” at a roadside stand. D sells similar stools
from another roadside stand and has advertised them as “Toad Stools.” When D began selling
his stools, P asked him not to use the name “Toad Stools.” Even though D continued to do so, P
took no further action until he filed this lawsuit for trademark infringement two years later. The
trial court awarded damages and granted P an injunction. D appealed.

Rule: If a person does not actively defend his trademark against known infringements,
including the bringing of a lawsuit if necessary, a competitor is free to use the trademark after
two years.

Issue: Did P actively defend his trademark?


Holding: Yes. Asking D not to use the name “Toad Stools” was sufficient for a small
businessman to actively defend his trademark.

Disposition: Affirmed.
Reasons and policies: Small businesses have fewer resources than large businesses and should
not be held to the same standard. The smaller the business, the easier it should be to satisfy the
active defense requirement.

Exercise 3-B

Bronson v. Road Runner Shoe Co. (1976)

Facts: P was injured when struck by a truck owned by D and driven by one of D’s employees.
The employee was on his way back to work after visiting his girlfriend during his lunch hour.
He had not been given permission to use the truck, but normally he had sole possession of the
keys during working hours. D had never objected to his private use of the truck. P sued D
under the theory of respondeat superior. The trial court granted D’s motion for summary
judgment, and P appealed.

Rule: An employer is liable for the torts of its employees when they are acting within the scope
of their employment.

Issue: Was the employee acting within the scope of his employment?

Holding: Yes. Even though the employee did not have permission to use the truck during
lunch, he was acting within the scope of his employment while he was driving back to work.

Disposition: Reversed.

15
Reasons and policies: Because employers have control over their employees, employers are
responsible for the torts their employees commit. Employers cannot avoid such responsibility
by asserting that the actions of their employees were not authorized; few employers actually
authorize employees to commit tortious acts.

Exercise 3-C

State v. Phillips (1999)

Facts: D set a building on fire. While on the way to the fire, a fireman was killed when he fell off
the truck and was run over by a car that was speeding and following too closely. D was
convicted of arson, a felony, and felony murder. She appealed from her conviction for felony
murder.

Rule: If someone is killed during the commission of a felony, the defendant is guilty of felony
murder.

Issue: Is D guilty of felony murder?


Holding: No. D cannot be held criminally liable for the death of the fireman from an
unforeseeable risk, even though it occurred during the commission of a felony.

Reasons and policies: The felony murder rule is intended to deter people from committing
felonies, especially those that are inherently dangerous to human life. Holding a defendant
responsible for a death from an unforeseeable risk created by an intervening force would not
further that purpose. There must be a limit to the nearly strict liability imposed under the felony
murder rule.

16
4
Precedent and Stare Decisis

Changes from the Sixth Edition

There are no major changes from the sixth edition.

Teaching Notes

Exercises 4-A, 4-B, and 4-D work better as vehicles for discussion than as writing
assignments because the concepts of precedent and stare decisis need considerable exploration
in the classroom. Exercise 4-B also gives you a chance to introduce an idea that will be
developed in Chapter 28 (Briefs to a Trial Court): trial courts are reluctant to depart from
established law in the jurisdiction.
Class discussion of Exercise 4-C can take well over an hour because it is difficult to
develop precise rules in a large group. We have had success dividing the class into small
groups of five or so, with each group working through the exercise and one member of each
group recording the answers. We then had the answers typed and distributed them during the
next class meeting. You could also require written answers to Exercise 4-C and use the best
ones as a basis for class discussion.
Some professors have worked through some of the exercises in this chapter in the
context of a motion argument. For exercise 4-D, for example, students would be assigned to
represent Fowler or the insurance company and instructed to prepare an argument based on the
only relevant authority, Eckersly. This, of course, requires one party to interpret the precedent
more broadly and the other to interpret it more narrowly. If you like, you can assign three
students to sit as a panel of judges. While the “lawyers” develop their arguments, the “judges”
must prepare one or two questions for each side. After the argument, the judges can decide the
case and explain their decision.
The need to actually argue the case, rather than discuss it in the abstract, helps many
students understand that “the law” is elastic and that there is not always a single “right answer”
to every question. This kind of hands-on approach is a good teaching technique for the
experiential learners in your class. It can also invigorate students who came to law school eager
to argue a case, only to discover that they must spend hours every day reading and briefing
cases and being told that the answer to every question is “it depends.”

Suggested Approach to Exercises

Exercise 4-A

1. Yes. Even though the later case did not mention Waterford, the definition of reckless
endangerment set out in Waterford is no longer accurate. Under Waterford, operation of
a motor vehicle in violation of the law is still reckless operation. However, after Seperic
reckless operation must also endanger the lives or safety of others in order to
constitute reckless endangerment. This is an example of implicit overruling, although
the Waterford decision is still helpful in defining reckless operation of a motor vehicle.

17
2. Students may find support for either conclusion. How they explain and support their
decision is more important than how they decide the case. Some (but not necessarily
all) points students might make:

Conviction affirmed:
Under Waterford, Buckler’s violation of the provision that prohibits driving while
under the influence of alcohol establishes that his operation of his vehicle was reckless.
Not only is there an analogy to be made regarding the collision that each driver had, but
there is also an analogy in that both drivers were under the influence of alcohol. Further,
the Seperic requirement that the reckless operation endanger the life or safety of another
seems to be satisfied because Buckler was not only driving after drinking heavily but
also was doing so with a passenger in his car. Drunk driving in our society has come to
be synonymous with endangering the lives and safety of others.

Conviction reversed:
It is difficult to conclude beyond a reasonable doubt that Buckler violated the provision
prohibiting drunk driving; he was not charged with that violation, so it was not proved
at trial. Thus, it is questionable whether, under the definition of reckless conduct given
in Waterford and Seperic, his conduct can be so characterized. The type of accident that
occurred in Waterford is distinguishable from that which occurred here. In Waterford
the driver actually ran down the decedent. Here, the accident occurred when Buckler’s
car left the road. There is no indication as to why it did so; thus, there is no evidence to
show beyond a reasonable doubt that he was operating his car in a manner that
endangered anyone’s life or safety.

Exercise 4-B

1. Under the only binding case on point, Elson would not be entitled to a refund. In Aaron,
the court reaffirmed the difference between a land contract and a mortgage. Because
Elson chose to purchase her land under a land contract, she “risked losing everything.”
The buyer in Aaron had only paid about one-third of the purchase price before the
default and Elson had paid approximately three-fourths. The Aaron court’s reasoning,
however, does not seem to leave this avenue of distinction open.

In an appellate setting, Deal could be used to try to persuade the court that the
difference between land contracts and mortgages is more form than substance and that
treating the two transactions differently is unfair. Elson could even argue that under the
concurring opinion, she should recover. The concurrence reaffirms the difference
between these two types of contracts, but opens the door for equity to step in. Here,
because Elson has paid so much of the purchase price, she has a very strong argument
in equity. But students need to realize that Deal is only persuasive and, as such, has a
limited impact on this decision.

18
2. Students may disagree as to whether this is fair. Elson, as an elderly blind woman who
stands to lose everything, will likely generate a good bit of sympathy. Some students,
however, may look at the problem from the perspective of the mortgage company and
reason that the mortgage company went into this transaction with certain expectations
and that it is not fair to change the rules at this point. You may find it helpful to
designate groups of students to represent each side and let them take turns arguing their
position.

3. Students should be able to see how the cases could be used to argue the other side: a
fairness and equity argument for Elson and a fairness and legal argument for the
mortgage company. But emphasize again the different precedential value of these two
cases.

Exercise 4-C

1. If the terms in a written contract are fully negotiated and understood by the parties, the
contract should be enforced as it is written. In this case, Tubbs can collect the money
because Hoffman agreed to pay full price for the stereo even if it was damaged in
transit.

2. Applying the same rule, students may reach different conclusions.

 Yes, she can recover. The terms of a policy should be negotiated and the parties
informed of all pertinent terms. The insurance company did not discuss flood
insurance with Goldberg or tell her that flooding was not covered in the policy. An
exclusion in the middle of a seven-page document is not an acceptable substitute for
full discussion of the terms of the contract.

 No, she cannot recover. The terms of the policy were set out in the contract and
agreed to by both parties. Goldberg had a duty to read the contract and discover
what she was contracting for. At the least, she had a duty to ask whether damage
from flooding was covered, or whether coverage was available. The contract was not
fully negotiated only because Goldberg chose not to negotiate.

3. Applying the same rule, students may reach different conclusions. They may also
decide that a contract that is manifestly unfair is unenforceable.

 No, the dealer cannot collect the remaining amount. The application of the rule
that a contract fully negotiated by the parties should be enforced is unfair in this
situation because Graff did not have the knowledge or sophistication to negotiate or
even understand the terms of the contract.

 Yes, the dealer can collect the remaining amount. Graff agreed to the contract price.
Unlike the Goldberg case where the seller hid a pertinent provision in the middle of
a seven-page contract, the seller in this case explained what Graff had contracted
for $30 per month for the next five years. The contract is not void simply because the
buyer lacked the skills to compute the total price of the refrigerator.

19
Exercise 4-D

1. The court has jurisdiction to hear Fowler’s case because the insurance company did not
obtain service by trickery or fraud. This case is unlike Eckersly, where the plaintiff
secured service of process by resort to what the court termed “shocking fraud.” The
plaintiff in Eckersly obtained service of process by luring the defendant, an out-of-state
resident, into the state with a false story that his mother was terminally ill. The plaintiff
in this case did not lie in order to entice Fowler into the state. Fowler entered the state
voluntarily in order to satisfy a merchant’s potential claim against her. Although the
merchant who served the papers on Fowler secretly worked for the insurance company,
his failure to disclose that fact was not fraud or trickery because there was no
misrepresentation.

2. The result is fair because Fowler submitted to jurisdiction by entering the state
voluntarily and because the defrauded insurance company will now have the
opportunity to prove its case in court without resort to fraud or trickery to obtain service
of process.

3. Eckersly could also support a decision for Fowler. That the merchant was a secret
employee of the insurance company suggests subterfuge. Collusion between the
merchant and the insurance company to entice Fowler into the state might be trickery
because the merchant did not tell Fowler about his dual role, but it would probably not
be fraud because there was no active misrepresentation.

20
5
Reading and Understanding Statutes

Changes from the Sixth Edition

There are no major changes from the sixth edition.

Teaching Notes

This chapter familiarizes students with the components of a statute so they can read
statutes more effectively. Reading cases can be a challenge for students, but at least cases are
stylistically familiar – they are narratives. Cases tell a story and discuss solutions using
regular sentences and paragraphs. Most cases follow the same basic structural rules – they set
out the facts before the law, and set out the law before applying it to the facts.
Statutes, on the other hand, may be unlike anything a student has read before. Their
structure is complex, their sentences are complex, and their meaning may be opaque. Statutes
adhere to no universally agreed-upon structure. Some have a purpose section, others do not.
Some have the governing rule at the beginning, others bury it in the middle of a section
further down. Many seem to have their own mystifying rules of punctuation.
The goal of this chapter is to provide the same kind of roadmap for reading a statute
that Chapter 3 (Case Analysis and Case Briefs) provides for reading judicial opinions. By
teaching students how to deconstruct statutes in order to find their key components, they
should be able to read statutes more efficiently and understand them better. This chapter thus
provides a foundation for the lessons in Chapter 9 (Statutory Analysis).
In addition to the exercises provided in the text, an in-class drafting exercise can help
reinforce the material in this chapter. Select a topic of interest to students and ask them to
work in small groups to draft a statute on the topic. Issues like the right to die,
decriminalization of drug possession, internet privacy, voting rights, and countless others
provide myriad topics for a drafting project. Ask students to draft a statute to address a
particular issue raised by their topic, requiring that the statute include some or all of the
components discussed in the text.
A final note about teaching this chapter: As noted with regard to Chapter 3 (Case
Analysis and Case Briefs), one of the authors assigns Chapter 6 (Understanding Legal Rules)
early in the fall term. The lessons in that chapter can be very helpful when students are
learning to articulate the rule or rules of law that can be derived from the requirements in a
statute.

Suggested Approach to Exercises

Exercise 5-A

a. Limitation of liability.

b. There is no purpose section, but an implicit purpose might be to encourage landowners


to open their land for certain beneficial public uses by limiting a landowner’s risk.

21
c. The statute applies to owners of land who want to allow others to use their land for
educational and/or recreational purposes without charging the users. It also applies
to people who are injured while using the land in this way.

d. There are no definitions in the statute.

e. The rule in this statute might be written as follows:

A landowner owes a person the same duty of care that the landowner owes a
trespasser when:

1. A landowner
2. Invites or permits a person
3. to use the landowner’s land
4. For educational or recreational purposes
5. Without charge.

f. The limitation on potential landowner liability provided by the trespasser standard of


care does not apply in four different circumstances:
1. The person is a direct invitee of the landowner and the landowner did not
inform the person of an artificial or unusual hazard that the landowner knew
about.
2. The doctrine of attractive nuisance applies to the case.
3. The person is using the land in a way in which it is normally used and for
which a fee is usually changed, even if it is not charged in this instance.
4. The owner’s purpose for allowing others to use the land is to
promote a commercial enterprise.

This statute lends itself to additional exercises if you have time. First, the statute is
based on the fourth (and last) section of Chapter 38A. Section 38A-1 is a purpose section, §
38A-2 is a definitions section, and § 38A-3 sets out some exclusions. Because Chapter 38A is
very short – less than two pages – you could have students read the rest of the sections and re-
do the exercise using the entire chapter. Second, you could have students revise their answer to
part e. of the exercise by incorporating the four listed exceptions into the rule. Here is one
example:

A landowner owes a person the same duty of care that the landowner owes a trespasser when:

1. A landowner
2. Invites or permits a person
3. to use the landowner’s land
4. For educational or recreational purposes
5. Without charge.

UNLESS

22
a. the person is a direct invitee and the LO did not inform the person of an artificial or
unusual hazard of which the owner had knowledge, or
b. the doctrine of attractive nuisance applies to the case, or
c. the person is using the land in a way in which it is normally used and for which a
fee is usually charged, even if it is not charged in that instance, or
d. the owner’s purpose in allowing others to use the land is to promote a commercial
enterprise

Exercise 5-B

a. Subject matter of copyright: In general

b. There is no purpose section, but the implicit purpose of the statute might be described
as identifying & defining the types of works that are protected by the copyright laws.

c. The scope of the statute is most clearly described in factual terms: the types of works
identified. By implication, however, it also encompasses the creators of those works.

d. Sub-section (a) seeks to define “works of authorship” by listing eight categories of


works that are included within the meaning of that term. (Students should recognize
that this is not an exhaustive list). Several of the identified “works of authorship” in
turn, are further defined. For example, “musical works” is defined to include
“accompanying words;” dramatic works” are defined to include “any accompanying
music.”

e. The rule in this statute might be written as follows:

An original work of authorship is protected by copyright when:

1. It is fixed in any tangible medium of expression


a. now known or
b. later developed and
2. It can be perceived, reproduced, or otherwise communicated
a. directly or
b. indirectly or
c. with the aid or a machine or device.

f. The second section of the statute contains expressly excludes certain intangible things
from copyright protection. A discussion of this section can explore the differences
between the intangibles that inspire a work of art and the work itself.

23
Part B
Basic Concepts of Legal Method

24
6
Understanding Legal Rules

Changes from the Sixth Edition

There are no major changes from the sixth edition.

Teaching Notes

This chapter acts as a bridge between Parts A and B of the text. It helps students make
the transition from thinking about the law in a somewhat abstract way to using the law to solve
specific problems. At the most essential level, a student’s ability to understand how the law is
“put together” will determine whether that student’s analysis of a problem is strong or weak,
compelling or unpersuasive. You can use the lessons in this chapter to further two important
teaching goals: (1) to help students become more careful readers; and (2) to begin to show
students the connection between the structure of the law, issue identification, and the
organization of a written legal discussion or argument.

How Rules Are Constructed


Many students have trouble parsing rules thoroughly. Many more have trouble with
complex rules, or have trouble transferring their understanding of a rule into an organizational
framework for the analysis of a legal issue. At whatever point your students get stuck, you can
help them by showing them how to make a “picture” of a legal rule. Several examples appear
below. Consider having students work in small groups to create a visual depiction of the rules
in the chapter exercises, using a variety of the methods shown below.
Many students find the elements charts used in Chapters 7 through 9 in the text helpful;
you might ask students to look ahead to see how those are constructed. Students with strong
verbal skills and linear thinkers may prefer to create an outline. Visual and experiential learners
might prefer a more abstract representation, such as the “modules” shown below. Students with
scientific and mathematical skills might prefer a flow chart. Whatever they prefer, it is important
for students to understand that there is more than one way to study and more than one way to
organize legal information. Some students find that the penny drops when they see information
presented in an atypical way. The examples below are based on the Restatement (Second) of
torts § 13 (1965), which is discussed in the text and reads as follows:

An actor is subject to liability to another for battery if

(a) he acts intending to cause a harmful or offensive contact with the person of
the other or a third person, or an imminent apprehension of such a contact, and

(b) a harmful contact with the person of the other directly or indirectly results.

25
Outline – This is a fairly typical rule outline. You can easily show students how to adapt this
outline for their torts class by weaving examples from key cases into the outline where
appropriate.

A defendant is liable for battery if:

I. The defendant acts

II. With intent to cause:


A. A harmful contact or
B. An offensive contact or
C. An imminent apprehension of a harmful contact or
D. An imminent apprehension of an offensive contact

III. With either of the following:


A. The other person or
B. A third person and
IV. Harmful contact with the other person results either:
A. Directly or
B. Indirectly.

Modules/Concept Map – This is an unusual approach, but every year the light seems to go on
for at least one or two students when they see a rule illustrated in this way. The approach was
used by a student when she was asked to list all the things that related to the defendant’s intent,
then to the defendant’s target, etc.

26
Flowchart

Defendant acts

Defendant intends to cause:

Harmful or Offensive or Imminent apprehension


contact contact of such contact

with

Other person or Third person

AND

Harmful contact with other or Harmful contact with other


person directly results person indirectly results

THEN

Defendant liable for battery.

Visual depictions of the law will provide valuable information about how to structure
the written legal analysis of any issue. A lawyer—or law student—who understands the
workings of a rule so well that she can design a vivid, detailed “picture” of the law will know
exactly what steps must be taken to resolve a legal question.

27
Developing a Rule from Multiple Sources—Synthesis
Students often have a difficult time teasing out relevant factors from a series of cases. If
you discuss the hate crime cases in this chapter in class, you can show students how to look for
common characteristics in a series of cases. At the same time, you can restate an important
point about reading the law. A typical judicial decision begins with an overview of the facts of
the case. Not all of those facts will turn out to be relevant, but the court will often repeat
selected facts shortly before it announces its holding on a particular issue. Remind your students
that they should probably pay attention if the court repeats facts, because these are likely the
most relevant facts--the facts upon which the court relied in reaching its decision.
It can be helpful to have students look at two or three cases to demonstrate this point.
Consider using some of the hate crime cases that formed the basis for the examples in this
chapter. You could also look at cases your students are using for a closed memo, or find
examples in a casebook from one of your students’ other classes. The lesson to take away is
this: whether you are briefing a case for Property or trying to synthesize cases in your writing
class, if you are not sure which facts matter, start with the facts that the court mentions more
than once.

Suggested Approach to Exercises

Exercise 6-A

This is a fairly simple starting place because the rule, as written, identifies three elements.
Consider reminding students, however, that the punctuation of a rule, including enumerations
like those present in this example, does not always tell the whole story. In the Restatement
(Second) of torts example in the text, for example, more than one element or sub- element
appears in some of the clauses (e.g., “a harmful contact with the person of the other directly or
indirectly results”).

1. The rule contains three elements: The first element describes requirements for the
testator, the second element describes the requirements for the beneficiary, and the third
element describes the benefit received by the beneficiary.

2. The first element has two requirements—the testator’s intellectual capacity and the time
at which that capacity is measured. Both must be present or a will could be voided if the
testator was perfectly competent when he executed the will but was of weakened
intellect at death.

3. The rule is conjunctive—all elements must be present.

4. Undue influence “sufficient to void the will” is established.

5. The result if the elements of the rule are established is not entirely clear. If the showing
is “sufficient to void a will,” does it follow that the will must be voided? Ask students
to consider the possibilities to try to arrive at an answer. Assume, for example, that a
court decides not to void a will for undue influence. Which is more likely: (1) the court

28
concluded that there was insufficient evidence of undue influence, or (2) the court
concluded that there was sufficient evidence of undue influence, but decided not to void
the will?. This exercise presents a perfect opportunity to point out that the law will not
always provide a definitive answer to every question. Sometimes students will need to
rely on their common sense—just as they did before starting law school.

Consider taking some time here to discuss the benefits and drawbacks of the various
kinds of rule consequences. Firm consequences have the benefit of being clear and
uniform, but they can be rigid and unresponsive to unique circumstances. Discretionary
rules, on the other hand, are flexible and allow a judge to shape the rule to fit the
circumstances of an individual case. That same flexibility, however, makes it difficult
to predict how the rule might be applied in a given situation, and can lead to uneven
results.

6. No exceptions.

Exercise 6-B

This is a good example to discuss in class because it allows you to work through some
common errors that students make when they are learning to read the law. First, students often
fail to distinguish between elements or sub-elements, and as a result they paint with too broad a
brush and miss important details. In this example, students may make assumptions based on the
punctuation of the rule and think of “extreme and outrageous” as a single concept. Students
should be asked to consider whether it is possible for conduct to be “extreme” but not
“outrageous,” in which case an element would not be established. Conversely, students
sometimes chop a rule into too many small pieces that cannot stand on their own. In this rule,
for example, students often separate “intent” from “toward the plaintiff.” Intent, however, has
little meaning standing alone; one cannot simply “intend.” Intent requires a focus; one must
intend something. In this example, students must understand that the defendant’s intent must
not simply be present; it must also be directed toward the plaintiff. Finally, this example allows
you to discuss causation—the mere presence of a defendant’s intentional behavior toward the
plaintiff and the plaintiff’s distress is not enough; there must be a causal link between the two.
Some further explanation of this exercise follows:

Intentional Infliction of Emotional Distress (IIED)

To establish intentional infliction of emotional distress, the conduct of the


defendant must be intentional toward the plaintiff, the conduct must be extreme
and outrageous, there must be a causal connection between the defendant’s
conduct and the plaintiff’s mental distress, and the plaintiff’s mental distress
must be extreme and severe.

1. The rule contains four elements if you divide the rule as punctuated—the rule has four
clauses, all of which require something. It contains seven elements if you consider the
pairs of ideas embedded in three of the clauses.

2. See above.

29
3. This is a conjunctive rule. All elements and sub-elements must be present.

4. The defendant is liable for IIED.

5. Mandatory. The words “to establish” imply that the result will follow if the elements
are established.

6. No exceptions in the rule as presented.

Exercise 6-C

This exercise allows you to once more focus on the importance of critical reading as
you work through the parts of the rule with your students. Some points for discussion include
the following:

1. The case concerns a minor, not an adult.

2. It is not enough that the bar served a minor, the rule requires that the bar “know
or should know” that the person served is a minor.

3. The risk of harm to another must be an unreasonable risk—what might that mean?
4. The risk of harm must also be foreseeable—make sure your students notice that
the court includes the fact that the tavern employees knew that Cooper was
going to drive.

5. One can infer from the facts of the case and the court’s decision that there has to
be a causal connection between the foreseeable risks and the actual harm that
results.

The rule from Nicholson v. Carson’s Tavern can be broken into several elements. A typical
outline of the rule from the case might look like this:

A tavern is negligent if:

1. It sells alcohol
2. to one who it knows or should know is a minor and
3. Thereby creates a risk of harm to others that is:
a. Unreasonable and
b. Foreseeable and
4. The minor causes injury to another.

Exercise 6-D

One way to handle this exercise, especially if it is your students’ first attempt at
synthesizing a rule from a series of cases, is to ask students to do a depiction of the rule from
each case first, then blend the elements together into a rule that encompasses all of the cases.
You might want to explain that this is an inductive reasoning process: one uses a series of

30
specific examples to create a general rule. Possible examples of rule depictions from the three
cases, and a synthesized rule of the four cases together, follow:

Teniel v. Olde Towne Inn – this case expands the rule from Nicholson in two ways: (1)
it applies the rule to people who are obviously intoxicated, and (2) it applies the rule to
injuries to the intoxicated person.

A tavern is negligent if:

1. It sells alcohol
2. to one who it knows or should know is intoxicated and
3. Continues to serve the person and
4. It is reasonably foreseeable that the person could harm himself or others and
5. The person causes injury to himself.

Valerio v. Glenn – This case expands the rule in Nicholson by extending its reach
beyond licensed sellers to social hosts.

A social host is negligent if:

1. The host serves excessive amounts of alcohol


2. to a guest that the host knows is a minor guest and
3. Injury to another is reasonably foreseeable and
4. The intoxicated minor injures another.

A synthesis of the rule as developed through Nicholson, Teniel, and Valerio might look
something like this:

A person is negligent if:

1. The person is a holder of a liquor license or a social host,


and
2. The person sells or furnishes alcohol
3. to someone that the person knows or should know is:
a. Intoxicated or
b. A minor
and
4. The provider of the alcohol creates a risk that is:
a. Unreasonable and
b. Foreseeable
and
5. The intoxicated person thereby injures:
a. Himself or
b. Another.

31
When your students have synthesized the three cases, you can point out that rules do not just
tell us what the law is. Rules also help us to identify what the law has not said. Ask your
students what the synthesized rule does not tell us—i.e., what questions remain unanswered?
The court’s reasoning in Valerio, for example, makes it unclear whether the case will apply to
adult guests as well as to minor guests. Similarly, it is not clear whether Valerio changed the
rule as it applies to minors: must the defendant merely “serve” a minor (Nicholson), or serve
“excessive amounts” (Valerio)? Finally, all of the cases deal with injuries due to drunk driving.
Would other kinds of behavior by the intoxicated person be deemed foreseeable? What kinds
of behavior, and why or why not?

Additional Exercises

If you use the additional exercises below you (and your students) will undoubtedly
notice that the facts and law for Exercise 6-E are very similar to State v. Klein, used in Chapter
3 (Case Analysis and Case Briefs). The example in Chapter 3 focuses on the common burglary
element of “entry” into a building to teach students how to articulate the holding in a judicial
opinion. In contrast, the exercise below presents the element of “entry” in the context of a
burglary statute, and asks students to consider how that element has been defined or interpreted
in two different cases. The exercise below thus gives students the opportunity to practice a more
difficult skill in a somewhat familiar setting.
Exercise 6-E is also somewhat similar to an additional exercise for Chapter 7
(Identifying and Selecting Issues for Analysis); Exercise 7-D also analyzes authorities on
burglary. The statutory rule in Exercise 6-E, however, does not require a “breaking,” while the
common law rule in Exercise 7- D does. If you use both of these examples with your students,
we suggest that you remind them that the definitions of many crimes and causes of action differ
from state to state.
Exercises 6-F and -G, below, are similar to an example in Chapter 13 (Applying the
Law); both involve the shopkeeper’s privilege defense to a suit for false imprisonment. The
exercises below ask students to synthesize a statute and two cases to determine what the rule is.
Chapter 13 uses the same subject matter to show students how to explain the law in context:
although the example focuses on the shopkeeper’s defense, the text reminds students to explain
that the defense only arises in the context of an alleged shoplifter’s suit for false imprisonment.
Using different versions of a rule, or applying the same rule to different fact scenarios,
can help students learn to think more flexibly and more critically by encouraging them to
revisit familiar material for different purposes. As students become more familiar with an area
of law, you can expect them to demonstrate a deeper understanding of the law and to work with
it in a more sophisticated way. On a more practical level, if you have limited in-class time with
your students, this technique allows you to spend a little less time making sure that students
understand the law on which an exercise is based and a little more time on the particular skill
being taught.

32
Exercise 6-E

Your supervisor has asked you to do some research for a burglary case she is defending.
You are particularly interested in discovering the meaning of the word “enter” as used in the
statute.

The relevant law consists of a statute and two cases:1

Section 137.02 Burglary

A person is guilty of burglary if he enters a building or occupied structure, or


separately secured or occupied portion thereof, with intent to commit a crime therein,
unless the premises are at the time open to the public or the actor is licensed or
privileged to enter.

State v. Mathers (1984)

Defendants appeal their conviction for burglary, arguing that the evidence was
insufficient to establish all elements of the charge. We affirm.
The state’s only witness was Al Szymanski, a security guard for the Garden Hill
school district. At approximately 6:15 p.m. on November 11, 1980, a school holiday,
Mr. Szymanski was notified of a possible break-in at Garden Hill High School. He went
to the scene and found Defendants inside the high school, removing laptop computers
from one of the computer labs. Some plywood that had been used to board up a broken
window had been removed, and the lock on the door to the computer lab had been
broken. Defendants were arrested and charged under § 137.02, which provides in
pertinent part that a person is guilty of burglary if he “enters” a building with intent to
commit a crime inside.
Defendants argue that there was no proof that they had “entered” the building
because there was no evidence that they removed the plywood from the broken
window. Although it is true that the plywood could have been removed hours before the
security guard arrived, proof of “entry” does not require proof of a breaking-in; it is
sufficient to prove that any part of the body of the intruder entered the premises.
Because Defendants were apprehended inside the school, they clearly had “entered” the
building. Affirmed.
State v. Holloway (1993)

Defendant appeals his conviction for burglary of a sporting goods store. He


argues that the evidence does not establish a prima facie case of burglary because no
part of his body entered the store. We disagree.
The parties stipulated to the following facts: Police responded to a silent alarm
at All Sports Xtreme. When they arrived, they found a jacket sleeve protruding from the
mail slot in the store’s front door. A straightened coat hanger was on the ground near
the door. The store’s owner told police that several knitted caps had been taken from a
rack inside the store, near the door, but there was no evidence that the store had been
1
This exercise is based on 18 Pa. Cons. Stat. Ann. § 3502(a) (West 2002); Commonwealth v. Gordon, 477 A.2d
1342 (Pa. Super. Ct. 1984); and Commonwealth v. Peterson, 21 Pa. D. & C. 4th 222 (1993)

33
broken into. Defendant was apprehended at a gas station near the store with the
stolen caps in his possession. He admitted that he had used the coat hanger to pull the
caps through the mail slot in the door. He also admitted that he had attempted to remove
the jacket, but it was too bulky to fit through the slot. Defendant then abandoned the
attempt and fled the scene.
The obvious purpose of the burglary statute is to protect the sanctity of the
building and the safety of the persons and contents within it. When that building is
penetrated by an offender with the intent of committing an offense, in this case theft of
the owner’s goods, it does not matter whether the entry was by the defendant’s entire
body, by defendant’s hand or arm, or by an instrument held and manipulated by those
body parts. We therefore hold that the entry into the store only by the instrument of the
coat hanger, under the facts of this case, constitutes an “entry” under our present
burglary statute.

Write a statement of the rule on “entering.”

*****

You can use this exercise as either a homework assignment or an in-class exercise. Ask
your students to draft their own rule, or have them draft the rule with a partner. You can then
collect and critique all of the various versions of the rule that your students create. In the
alternative, hand out the following sample rule statements (Answers A and B) and ask students
to critique each one. You can begin the discussion by reminding students what the supervising
attorney has asked for. Then ask which statement of the rule the supervisor will find most
helpful and why.

ANSWER A: A person is guilty of burglary if he enters or gains access to a building in any


way with intent to commit a crime. Section 137.02; State v. Mathers; State v.
Holloway.

ANSWER B: A person is guilty of burglary if he enters a building with intent to commit a crime.
Section 137.02. The element of entry can be established in two ways: (1) if the
defendant uses any part of his body to enter the premises, State v. Mathers, or
(2) if the defendant holds and manipulates an instrument with his body to gain
access to the premises. State v. Holloway.

Both answers cite all of the relevant authorities. Answer B is better, however, because it
states the rule on entry with absolute clarity. The reader knows immediately that the element
can be satisfied in two different ways and can easily check the rule against a new set of facts to
see if the element is met. Answer A is much weaker. It implies that there is more to the word
“enter” than meets the eye, but the phrases “gains access to” and “in any way” are too vague to
be of much use to the reader.

34
Exercise 6-F

You work in the legal department of a large chain of hardware stores. Your
supervisor asked you to find the “shopkeeper’s privilege” statute.2

§ 15-60-7. Shopkeeper’s Privilege


Whenever the owner or operator of a mercantile establishment or any agent or
employee of the owner or operator detains, arrests, or causes to be detained or
arrested any person reasonably thought to be engaged in shoplifting and, as a
result of the detention or arrest, the person so detained or arrested brings an
action for false imprisonment against the owner, operator, agent, or employee,
no recovery shall be had by the plaintiff where it is established by competent
evidence:

(1) That the plaintiff had so conducted himself or behaved in such manner as
to cause a man of reasonable prudence to believe that the plaintiff, at or
immediately prior to the time of the detention or arrest, was committing the
offense of shoplifting; and
(2) That the manner of the detention or arrest and the length of time during
which such plaintiff was detained was under all the circumstances reasonable.

1. How many elements does this rule contain?


2. Do any of the elements have sub-elements?
3. What type of rule is this—disjunctive, conjunctive, a factors test, a combination of types?
4. What result if the elements of the rule are established?
5. Is that result mandatory? Prohibited? Optional or discretionary?
6. Are there any exceptions to the rule? If so, when do they come into play? What result if
an exception to the rule exists?

The shopkeeper’s privilege can be asserted as a defense to a false imprisonment claim.


It is therefore, conceptually, a kind of exception to the general rule on false imprisonment. You
can show students how the defense arises by starting with the definition of false imprisonment:
“the unlawful detention of the person of another, for any length of time, whereby such person
is deprived of his personal liberty.” Ga. Code. Ann. § 51-7-20 (1998).
You can then begin this exercise by working through the elements of false imprisonment:
(1) detention of another, (2) for any length of time, (3) that is unlawful, and
(4) deprives the person of his or her personal liberty. The shopkeeper’s privilege relates to the
third element because if the defendant is “privileged” to detain the plaintiff the detention is not
“unlawful.” After laying this foundation, you can then work through the remainder of the
exercise with your students:

1. The rule contains six elements: Merchant – detains person – person reasonably thought
to be shoplifting – person sues for false imprisonment – person’s behavior would lead a
reasonable person to believe that person detained was shoplifting – detention was
reasonable.
2
This exercises is based on Ga. Code Ann. § 51-7-60 (1998).

35
2. This rule can be confusing to work through because students get bogged down in the
details. Focus their attention by asking students to identify the basic topics covered by the
rule: (a) the merchant, (b) the act of detention, (c) the alleged shoplifter, and (d) the nature
of the detention. Then you can work with students to identify obvious sub-elements. For
example, one can use the general term “merchant” to include owner, operator, agent, and
employee; “detain” includes detain, arrest, cause to be detained, or cause to be arrested.
Then you can discuss the nature of the detention—it must be based on a reasonable belief
that the person was shoplifting, and it must be reasonable in manner and duration. Now
there are only a couple pieces still unaccounted for. There is some overlap between the
requirement in the first paragraph of the rule, that the merchant “reasonably thought” that
the person detained was shoplifting, and the requirement in subsection (1) that a
“reasonably prudent” person would believe that the person was shoplifting. The two
concepts, however, are not precisely the same. The remaining requirement that the person
detained sue for false imprisonment is an additional element.

3. It has a combination of conjunctive elements and conjunctive or disjunctive sub-elements.

4. The detainee cannot recover for (or the merchant is not liable for) false imprisonment if the
elements of the rule are established.

5. The result is mandatory: “no recovery shall be had.”


6. There are no exceptions to the rule. As mentioned above, the statute itself can be
characterized as an exception to the general false imprisonment rule.

Exercise 6-G

Review the shopkeeper’s privilege statute in Exercise 5-F. Then read the case summaries
3
below , and create a synthesized rule using the statute and both cases. In what ways does the
synthesized rule differ from the original rule that was based on the statute only?

Kovarik v. Big Box Stores, Inc. (1986)

The Appellant, Dan Kovarik, commenced this action for false imprisonment
and intentional infliction of emotional distress after being questioned by his
employer regarding the theft of store merchandise. The trial court dismissed the
false imprisonment claim, and Kovarik appeals.
Kovarik worked as a kitchen designer for Big Box Stores. In March 1984
he was summoned to the store security office and questioned for two hours about
reports from coworkers that he was stealing store merchandise. Kovarik testified
that he agreed to talk with the guards and signed a consent form. During the
subsequent interrogation, two guards took turns questioning him, leaving three- to
five-minute intervals between the interrogation sessions. Kovarik was also allowed
to go to the rest room (accompanied by a male security guard) and spent about thirty

3
Cases in this exercise are loosely based on Crowe v. J.C. Penney, Inc., 340 S. E. 2d (Ga. Ct. App. 1986); and
Jackson v. Kmart Corp., 851 F. Supp. 469 (M.D. Ga. 1994).

36
minutes writing a statement. Kovarik testified that both guards called him a liar,
and one guard slammed his hand down on the desk and shouted while questioning
him. He admitted, however, that he did not complain to the guards and did not ask
to stop the interview. In fact, Kovarik admitted that he preferred to continue the
interview “to clear everything up.”
The trial court found that the reports from Kovarik’s coworkers that he was
stealing store merchandise furnished probable cause for him to be questioned under
the shopkeeper’s privilege statute, § 15-60-7. The court found further that the
defendant was privileged to reasonably detain Kovarik to investigate the matter.
Finally, the court found that Kovarik was not detained against his will. We agree.
Kovarik signed a consent form before the interview and at no time did he request
that the questioning be stopped. Indeed, he testified that he wanted to remain to
“clear everything up.” Under these circumstances, it cannot be said that Kovarik
was “detained,” as he was not held against his will. The trial court did not err when
it dismissed Kovarik’s false imprisonment claim.

Williams v. Southern Exposure, Inc. (1997)

Marcia Williams sued Southern Exposure, Inc. for false imprisonment after
being detained on suspicion of shoplifting by a security guard employed by the
store. Southern Exposure’s motion for summary judgment was granted by the trial
court and Williams appeals. We reverse and remand the case for trial.
Alex Gallacher, a security guard, suspected that Williams was placing small
items of clothing in her purse as she shopped at Southern Exposure, an upscale
boutique. Gallacher shadowed Williams until he clearly saw her slip an expensive
scarf into her purse and then followed her to the checkout desk. Williams paid for
a sweater, but she did not remove any other items from her purse to pay for them.
Gallacher stopped Williams as she left the store and asked her to produce her receipt
and empty her purse. When Williams refused, Gallacher grabbed her by the arm
and led her to an office in the back of the store. He locked the door and told
Williams that she should “cooperate” with him because he could “do whatever he
wanted” and no one would believe a “thief and a whore” if she complained.
Williams immediately agreed to cooperate and gave the scarf and two other items
to Gallacher. She then signed a statement admitting that she had engaged in
shoplifting, and was allowed to leave the store. The detention lasted no more than
ten minutes.
The court found that Gallacher had probable cause to suspect Williams of
shoplifting. It found further that although Gallacher subjected Williams to
“gratuitous and unnecessary indignities,” the ten-minute detention was so brief that
the detention could not be considered unreasonable. We disagree.
It is clear that Southern Exposure had probable cause to suspect Williams
of shoplifting. Her behavior aroused the suspicions of Gallacher, who followed
Williams until he personally witnessed her place store merchandise in her purse
and leave the store without paying for it. The reasonableness of the detention,
however, is not simply a question of minutes or hours. The reasonableness of the
length of the detention may be impacted by the manner in which the person is

37
detained. Although Williams’ detention only lasted about ten minutes, Gallacher’s
abusive language and thinly veiled threats of physical or sexual assault could allow
a jury to find that the detention was unreasonable in both manner and duration. We
therefore reverse and remand for a trial on the merits.

Each case adds something to our understanding of the rule. Kovarik helps to explain two
elements: (1) what kinds of facts support a “reasonable belief” that someone was shoplifting, and
(2) what “detain” means. Some students may want to discuss whether the guards’ behavior during
Kovarik’s detention—yelling, pounding on the table—was unreasonable or, conversely, whether
the fact that Kovarik was given breaks means that his detention was reasonable. This allows you
to reiterate the importance of critical reading of authorities and provides you with a good
opportunity to discuss accuracy in case description and interpretation. Can one draw conclusions
about the meaning of these facts when the court did not rely on them to reach its conclusion? The
court did not dismiss Kovarik’s claim because it concluded that the detention was reasonable. It
dismissed the claim because the employee agreed to be questioned; the detention was not “against
his will.” Without that element, the claim fails, regardless of the reasonableness or
unreasonableness of the detention. (Of course, Kovarik might be able to pursue additional claims—
e.g., for battery or emotional distress, if the facts supported those claims. Remind students that
they must focus on and answer the question asked.)
Williams helps to explain what kind of conduct will cause a court to conclude that a
detention was not reasonable in manner or duration. Be sure that students understand that there is
no specific time period that is “too long;” even a relatively short detention can be unreasonable if
the manner in which the person is being detained is egregious enough.

38
7
Identifying and Selecting Issues for Analysis

Changes from the Sixth Edition

There are no major changes from the sixth edition.

Teaching Notes

The problems faced by first-year law school teachers generally—where to begin and how
to judge student work based on their imperfect knowledge of how to find and ascertain the “law,”
how the legal system works, and how to analyze legal issues—are particularly acute in legal
writing courses. We require written analysis early, before students have much acquaintance with
substantive law, while they are still learning to read and brief cases, and before they have read the
succeeding chapters in this book on how to organize and write about legal issues. This chapter
explains and illustrates the fundamental skills of issue identification and analysis. Students must
be able to separate true issues from “givens” or other collateral legal questions and evaluate the
relative significance of each. Once the students have grasped the basic principles set out in this
chapter, you can use the exercises to preview principles addressed in later chapters.

Suggested Approach to Exercises

We have used these exercises not only to teach deriving and applying rules of law from
cases and issue-spotting, but also to prepare students to draft a discussion using common law or
statutory analysis. As a class exercise, the students outline the tentative issues, find and state the
rules that apply, parse the rules, and fill in the outline with the specific issues and sub-issues that
arise from the rules. They also list under each issue or sub-issue (1) the relevant facts of this
case, (2) the statutory language or relevant facts and holding of the precedent cases, (3) any
analogy or distinction that could be drawn between this case and precedent cases, (4) reasonable
inferences from (1), (2), and (3), and a conclusion. From this class outline, the students can draft
a paper that includes most of the requirements described in later chapters.
This early introduction to the structure and content of a legal analysis via these exercises
is critical for those courses that require papers before Part C (Basic Concepts of Legal Writing) is
covered in full. These exercises are also useful as a review of Chapter 2 (Sources of Law), with
emphasis on the precedential value of concurring opinions, dissenting opinions, and secondary
authority.

Exercise 7-A

The factual and legal materials for this exercise are also used in Exercises 10-A, 12-A, 13-
A, and 16-A.
This exercise introduces two kinds of legal issues. For offer and acceptance, the question
is “How does the law apply to the facts of this case?” For consideration, the question
is also “What is the law?” Some of us hand out an annotated sample discussion of the offer issue
and then require students to write a discussion of the acceptance issue. Annotated discussions of
these two issues can be found in Appendices I.A and I.B of this manual.

39
1. The question is whether there is an enforceable contract.

2. The elements of an enforceable contract are: (1) an effective offer, (2) a valid acceptance,
and (3) sufficient consideration.

(1) For an offer to be effective, the offeror must intend to make a binding contract. The
test for intent is whether a reasonable person in the offeree’s shoes would believe
that the offeror so intended.
(2) To be valid, an acceptance must (a) be sufficiently communicated to the
offeror and (b) mirror the offer in every respect.
(3) Consideration is sufficient if it is not so grossly disproportionate as to shock the
conscience of the court.

3.

Enforceable Contract
ELEMENTS CHART
Element Facts of Our Case Element Met?
Effective offer McKay telephoned Green and as a Maybe
joke offered to sell Green his yacht.
Valid acceptance Green telephoned McKay intending Maybe
to accept the offer, but McKay was
golfing. McKay return his call
while intoxicated and passed out
before Green said that he accepted
the offer but would like McKay’s
golf clubs as part of the deal.
Adequate consideration McKay offered to sell his $100,000 Maybe
yacht for $10,000.

Note: The only possible “given” is consideration. According to dictum in the majority
opinion in Derek, the test is whether the consideration is “so grossly disproportionate that
it would shock the conscience of the court to enforce the contract.” Under this rule, a selling
price of 10% of the value may be sufficiently shocking, and consideration will be an issue.
The dissenting judge in Derek suggested the rule that “any consideration, no matter how
small, is generally sufficient.” If the court adopts this rule, consideration will not be an
issue. The dictum in Derek would probably apply because it is from a majority opinion.
Thus, consideration is probably an issue and not a “given.”

Exercise 7-B

The factual and legal materials for this exercise are also used in Exercises 10-B, 12-B, 13-
B, and 16-B.
This is a good problem for introducing students to statutes. The problem also raises some
interesting challenges for analysis of elements. The highly charged factual situation on

40
which the problem is based also gives an opportunity to discuss the role of law and lawyering, a
discussion that the teacher or the class could develop in several ways.

1. The question is whether, under the Freedom of Information Act, the newspaper has a
right to police department records regarding the CLO.

2. The initial challenge to this problem is finding the operative statutory language. That
language is in sections 3 and 4 because they set out the legal rules on which the newspaper
would base a claim for release of the records. It often helps to walk students through these
two sections. Section 4 provides the statutory basis for a potential lawsuit to compel
disclosure of the police records. But the newspaper cannot use section 4 unless it has first
made a request under section 3(a). Walking students through the wording of section 3(a)
shows that the elements include three terms defined in section 2—person, governmental
body, and public record. That may lead to a discussion about the relationship between
substantive provisions and definitions in a statute. The discussion should also include
section 3(b).

The general rule is found in section 4: If (1) a governmental body (2) denies a request for
disclosure of (3) a public record, (4) the requesting person may commence an action to
compel disclosure (5) if the person has allowed the governmental body five business days
to respond to the request. By reference to sections 2 and 3 and the facts of the case, students
will discover that the elements of the general rule are given:

3.

Action for Disclosure of Public Records


ELEMENTS CHART
Element Facts of Our Case Element Met?
Governmental body The term includes a “department” of a Yes
“county, city, or township.”
denies a request The Police Department denied the Yes
request
by a requesting person . “Person” means an individual, Yes
corporation, partnership, firm
organization, or association.”
for a public record The term includes a “writing prepared, Yes
owned, used, in the possession of or
retained by a governmental body.”
and is allowed five business The Police Department denied the Yes
days to respond. request in three days.

4. The exceptions in section 5(a) and 5(d) may be applicable.

41
5. The elements of the rule in section 5(a) are: A governmental body may exempt from
disclosure (1) records, including internal records and notations, (2) of law enforcement
agencies (3) that deal with the detection and investigation of crime and (4) are
maintained for their internal use in matters relating to law enforcement.

The elements of the rule in section 5(d) are: A governmental body may exempt from
disclosure (1) records containing information of a personal nature (2) that would be an
unwarranted invasion of a person’s privacy.

6.

Exemption from Disclosure of Records Relating to Law Enforcement


ELEMENTS CHART
Element Facts of Our Case Element Met?
Internal records and The records contain information Yes
notations regarding a police investigation of the
CLO,
of law enforcement The Star City Police Department is a Yes
agencies law enforcement agency.
that deal with detection and The CLO is a paramilitary organization Maybe
investigation of crime that aims to arm citizens on behalf of
the “Aryan or white elements of our
society.” The police have not arrested
any member of the CLO for illegal
activities.
maintained for internal use The information relates only to the Yes
investigation of the CLO and has not
been shared with the public.

Exemption from Disclosure of Records Containing Personal Information


ELEMENTS CHART
Element Facts of Our Case Element Met?
Records containing The records contain information about Maybe
information of a personal the structure, activities, and
nature membership of the CLO, including the
involvement of the mayor’s wife, who
is an officer of the CLO.
that would be an unwarranted The same facts apply to this element. Maybe
invasion of a person’s
privacy.

42
The police records exception may apply because these are records of a law enforcement
agency that “deal with the detection and investigation of crime” and are “internal records
and notations . . . maintained for its internal use in matters related to law enforcement.”
However, the Wheeler Publishing case indicates that not all records that seem to fall under
5(a) are exempt. It recognizes a right of the public to know certain information about law
enforcement efforts in the community and balances this right to know with the police
department’s need to protect its operations and the individuals involved. That suggests this
issue: Is the information in the CLO files “information concerning crime or law
enforcement activity in the community,” even though no CLO members have ever been
arrested for illegal activity associated with their membership? Resolution of this issue will
require a determination of whether disclosing the information about the membership,
structure, and activities of the CLO would jeopardize the Police Department’s law
enforcement activities.

The personal privacy exception may apply to at least some of the information in the police
records. Although the mayor’s wife is the focus of this issue, it is not clear that her
involvement is any more “personal” than that of anyone else. Section 5(d) was added to
the Act after the Holcombe case and is intended at a minimum to protect from disclosure
the names of rape victims. Voluntary participation in a racist paramilitary organization
seems like a significantly different situation. However, to the extent that these are not law
enforcement records, they may include personal information. For example, a court might
consider the membership of CLO to be “personal,” analogizing the situation to First
Amendment freedom-of-association cases. That suggests a second issue: Is the information
in the CLO files of a “personal nature,” the disclosure of which would constitute a “clearly
unwarranted invasion of privacy”?

Exercise 7-C

The factual and legal materials for this exercise are also used in Exercises 7-C, 10-C, 12-
C, 13-C, and 18-B.

1. The question is whether the contract to sell the bar is enforceable.

2. a. A contract (1) for the sale of land or a legal interest in land (2) must be in writing
and (3) must be signed by the person to be charged with the contract. In Treacher, the
court interpreted the second element to require that the writing identify with reasonable
certainty and accuracy the parties to the contract, the subject matter, and the terms and
conditions of the contract.

b.

Statute of Frauds
ELEMENTS CHART
Element Facts of Our Case Element Met?
A contract for the sale of land Zoeller offered to sell his bar to Yes
Hughes.

43
must be in writing and identify He wrote the contract on a Maybe
napkin and included:
a. the parties to the contract the names of the parties, Yes

b. the subject matter the name of the bar, and Yes

c. the terms and conditions the asking price but not the Maybe
payment conditions or specific
dates.
and signed by the parties Neither party signed the Maybe
contract but the parties’
conversation, including
Hughes’s oral agreement, was
recorded by a third party.

3. a. The elements of fraudulent misrepresentation are: (1) false representation, (2) of


material facts, (3) that the defendant knew were false, (4) that the defendant
intended the plaintiff to rely on, (5) that the plaintiff was justified in relying on, and
(6) the plaintiff suffered damages as a result of the reliance.
b.

Elements Chart
MISREPRESENTATION
Element Facts of Our Case Element Met?
False representation Zoeller showed Hughes altered Yes
business records and
misrepresented the value of the
property.
of material facts Same facts as above Yes

that the defendant knew were Zoeller intentionally altered his Yes
false business records and also knew
the true value of the property.
that the defendant intended the Zoeller misrepresentations were Yes
plaintiff to rely on for the purpose of convincing
Hughes to agree to buy the bar.
that the plaintiff was justified in Zoeller claimed that he was a Maybe
relying on good businessman but Hughes
had concerns over the validity
of the figures. She was also
drinking and was intoxicated
when she agreed to the contract.

44
the plaintiff suffered damages The sale price exceeded the Maybe
market value by $40,000. For
the last three years, Zoeller had
made a profit of $25,000, and
the profits may double or triple
in the next few years.

Additional Exercises4

From the examples and exercises in the text, students learned that in order to decide
whether a particular element has been satisfied, one must pay attention to whether and how that
element has been defined. The exercises below reinforce this idea by requiring students to look at
the definition of an element and include that definition in their list of elements.
The burglary issue in Exercise 7-D also illustrates that elements are sometimes interrelated
so that whether one is met depends on its relationship to another. On the other hand, Exercise 7-E,
a false imprisonment problem, requires students to distinguish between two elements—
confinement and consciousness of confinement—that appear to be similar because some of the
same facts are relevant to each. They are, however, two distinct elements, as is apparent from the
majority and dissenting opinions in Cay v. City of Greenwood.

Exercise 7-D

One winter day Jeremy Howe, a homeless person, observed the occupants of a house
putting suitcases into their car and driving off. That night, he decided to go inside the house and
see if he could find something to eat and a warm place to sleep. All of the doors and windows were
locked, but he did find that a door to an unattached garage was unlocked. He pushed the door open
and went in. Part of the garage had been converted to an office, with a desk, chair, filing cabinet,
and couch. He did not find any food in the garage, so he went to sleep on the couch. The next
morning, he searched again for some food, to no avail, but he did find $500 in an envelope in a
desk drawer. He took the money and left. A neighbor saw him walking out of the garage and called
the police, who arrested him for burglary.
In this state burglary is defined as breaking and entering the dwelling of another with the
intent to commit a felony therein. Breaking is the use of force, however slight, in order to gain
entry. Entering is the act of going into the dwelling of another without the consent of the occupant.
Larceny is the taking and carrying away, with intent to steal, the personal property of another. If
the property taken is worth $20 or more, the larceny is a felony; otherwise, it is a misdemeanor.
Make an outline of the elements of burglary and indicate which elements have been
satisfied in these case and which ones are likely to be at issue, based on the facts given. The chart
below lists the elements and identifies those likely to be at issue.

4
As noted earlier, Exercise 7-D and Exercise 6-E in this teacher’s manual both analyze authorities on burglary. The
statutory rule in Exercise 6-E, however, did not require a “breaking,” while the common law rule in Exercise 7-D
does. If you use both of these examples with your students, we suggest that you remind them that the definitions of
many crimes and causes of action differ from state to state. Similarly, Exercise 7-E analyzes a false imprisonment
claim. Students will see this cause of action again as an example in Chapter 18 in the text.

45
Elements Chart
Elements Element satisfied?
Breaking a dwelling: Maybe
Use of force, however slight, Yes
In order to gain entry; Yes
Entering Maybe
Going into the dwelling of another Maybe
Without the consent of the occupant; Yes
Dwelling of another Maybe
With intent to commit a felony therein. Probably not

Comments: The elements of breaking, entering, and dwelling of another are interrelated,
since they will be satisfied only if the unattached garage is considered a dwelling. If the
garage is a dwelling, the breaking element is clearly satisfied because pushing the door
open is the use of force, however slight. Breaking and entering are also related to intent.
Some students will conclude that this element is satisfied because Howe took $500 with
the intent to steal it, which is a felony. However, one must break and enter with the intent
to commit a larceny. Howe’s intent at the time he broke and entered was to steal some
food, which is probably not a felony, and find a warm place to sleep. Therefore, this
element is probably not met.

Exercise 7-E

Your client is Juan Velasco, a photographer for a Mexican sports magazine, who was
injured while he was filming a tennis match between Jim Davis of the United States and Luis
Guerrero of Argentina during the International Tennis tournament (ITT) held in your city.
Guerrero is a very popular player and had a substantial number of vocal supporters in the audience.
Davis, who is known for his quick temper, became quite angry early in the match because of the
crowd’s support of Guerrero and also because of some questionable line calls. He had several
arguments with the chair umpire and linesmen, during which the crowd booed. Finally, the chair
umpire told Davis to either play tennis or forfeit the match. The next ball that Davis hit went into
the net. He swore and threw his racket across the court. The racket ricocheted off a metal post and
hit Velasco in the face.
Rhonda Smith was a volunteer recruited by the ITT to work in the first aid tent at the tennis
center. She was on duty during the Davis-Guerrero match. After the tennis racquet hit Velasco, an
usher escorted him to the first aid tent where Smith applied an antiseptic and adhesive bandage to
the facial cuts. She learned from the usher that a volatile situation was developing in the stadium,
and she could hear the crowd chanting and yelling. She decided to call the ITT office and ask for
advice about what she should do. Because of earlier outbreaks of violence, the volunteers and
employees of ITT had been ordered to “take whatever steps are appropriate to avoid incidents and

46
bad press.’” She was told to keep Velasco there until a delegation of ITT officials could talk to
him.
Smith and Velasco had to communicate by signs and motions because she spoke only
English and he spoke only Spanish. After talking to the ITT office, she motioned to Velasco to
remain where he was. Several times, he indicated that he would like to use the telephone but Smith
shook her head, indicating “no.” When he indicated that he would like to leave, she motioned him
back to his chair. During this interval, Velasco was still dazed from the blow he had received. He
had a severe headache and was having trouble seeing out of one eye. He knew that he needed
medical attention, but he thought that Smith had called a doctor or an ambulance. After almost an
hour had elapsed, Velasco decided to leave the tent and try to find an interpreter who could explain
to him what was going on. He tried to communicate this desire to Rhonda but again she motioned
him to stay. When you asked Velasco why he had not simply left, he replied, “I thought the lady
might be a police officer. She was wearing a uniform and seemed to have authority. So when she
motioned to me that I should stay, I stayed. I did not want to get into any kind of trouble.”
About an hour after Smith called ITT, three officials and an interpreter arrived. The ITT
representatives asked Velasco to make a public statement that the incident had been a mere
accident and should not be blown out of proportion. They also said that the city did not want any
more bad publicity, and they agreed to reimburse Velasco for his medical expenses and lost income
if he would make such a statement. Velasco would not agree to make such a statement and
demanded that he be taken to a hospital. One of the ITT representatives took Velasco to the
emergency room at a hospital, where he was treated for facial lacerations and bleeding in one eye
and released.
Your task is to determine whether Velasco has a cause of action for false imprisonment
against ITT. You may assume that Smith is an agent for ITT and therefore ITT will be liable if she
falsely imprisoned Velasco. You have found the following cases from your state.5

Cay v. City of Greenwood (1987)

Appellant Donald Cay appeals from the trial court’s dismissal of his claim
against the City of Greenwood for false imprisonment. Two police officers
employed by the City responded to a call that there was a fight in progress and
encountered Donald Cay and several other people engaged in a brawl. Everyone
ran away except Cay. The police officers determined that Cay was intoxicated,
ordered him into the patrol car, and asked where he lived. Cay told the officers he
did not have an address. Rather than taking him to the overnight lock-up facility in
the local jail, the officers drove him to a campground outside the city limits where
there were shelters and toilet facilities. The campground was not fenced in or
otherwise barricaded. The officers claim that this is standard operating procedure
when they pick up drunks who are homeless.
Cay has said that he does not remember much about the evening, but he
does remember complaining to the officers that they were leaving him “out in the
middle of nowhere.” During the night, in an apparent attempt to find his way back
to Greenwood, Cay wandered onto a highway adjacent to the campground. He was
struck by a car and severely injured.
5
The cases are based on Parvi v. City of Kingston, 362 N.E.2d 960 (N.Y. 1977), and Hoffman v. Clinic Hospital,
Inc., 197 S.E. 161 (N.C. 1938).

47
For a cause of action for false imprisonment the plaintiff must show that the
defendant intended to confine him, that the plaintiff was conscious of confinement,
and that the plaintiff did not consent to the confinement.
It cannot be contended that the police officers, in view of the direct and
willful nature of their actions, did not intend to confine Cay. Nor is it arguable that
Cay consented. Whether Cay was conscious of confinement in light of his
intoxicated state presents a closer question. The trial court concluded that he was
not because of his testimony that he had no recollection of his confinement. In so
doing, the court failed to distinguish between a later recollection of consciousness
and consciousness at the time of confinement. Cay’s statement that the officers
were leaving him “out in the middle of nowhere” demonstrates some awareness of
what was happening to him. It was for the jury to determine whether Cay was
indeed aware of his arrest at the time it took place. Reversed.
Baxter, J., dissenting. For a false imprisonment claim, there must be
evidence of actual confinement. In this case, there was no confinement, only an
exclusion from one area. Once the officers left Cay at the campground, he was
unrestrained and free to depart at any time, which he did of his own accord and to
his misfortune. I would affirm the trial court’s dismissal of the case.

Kaufman v. Central Hospital, Inc. (1948)

Edith Kaufman brought an action against Central Hospital for false


imprisonment in the hospital. The trial court entered judgment for the hospital.
Kaufman entered the hospital for treatment and three days later was told by
her doctor that she could leave. On that same day, the hospital manager came to her
room and told her she could not leave without paying the hospital bill. Kaufman
stayed for two more days and then decided to leave, even though she did not have
the money to pay her bill. She testified that no one restrained her by any kind of
force.
False imprisonment is the restraint of a person against his will by force, or
by an express or implied threat of force. Merely considering oneself restrained is
not sufficient unless there is a reasonable apprehension that force will be used upon
an attempt to assert one’s liberty. In this case, the hospital employees employed
neither force nor threats of force to restrain Kaufman. She left the hospital without
paying her bill and no attempt was made to prevent her from going. Judgment for
the hospital was property entered. Affirmed.

1. List the rule(s), and elements of the rule(s), in this problem

2. Are there any elements you would exclude from analysis because they are “givens”? If so,
which ones?

3. Identify the issues and sub-issues (if any) in this problem.

The chart below lists the elements of the rule and identifies the issues.

48
Elements Chart
Elements Element satisfied?
Intent to confine Yes
Confinement Maybe
By the use of force, or No
By the use of threats, express or implied Maybe
Consciousness of confinement Maybe
No consent Yes

Comments: The closest question is whether Velasco reasonably apprehended a threat to


use force if he left. Smith simply motioned him to stay when he tried to leave, but his
belief that she might be a police officer and their inability to communicate because of the
language barrier might make his apprehension reasonable. The lack of verbal
communication, as well as his dazed condition, also raises the issue whether he was
conscious of confinement.

Exercise 7-F

Your client is Ann Darnell. Ann’s older sister, Sarah Lucas, was a well-known artist before
her death a few weeks ago. Sarah and Ann had not been on good terms for a number of years, for
reasons that Ann did not want to divulge. On her twenty-first birthday, Ann received a card from
Sarah that stated, “My gift is a portrait of you that I have been working on for several months now.
I still have a little work to do and then it is yours. Let’s let bygones be bygones. Love, Sarah.” Ann
told you that she did not acknowledge or respond to the card. She still harbored some bitterness
toward Sarah, and she told her mother that she did not want to just take the painting and resume
an amicable relationship with Sarah before she and Sarah had a chance to sit down and talk about
their problems.
Three weeks later, Sarah left town to attend an art show and was fatally injured in an
automobile accident. The morning that she left, Sarah sent Ann an e-mail message saying that she
had finished the portrait but had not gotten a chance to deliver it before she had to leave town. She
told Ann that it was in her studio and that Ann could get a key from their mother and take the
painting home with her. Ann did not read the message until after she heard that Sarah had been
killed. When she went to the studio several days later, she discovered that it was empty. Sarah’s
husband had removed all the paintings. He is claiming that the portrait belongs to him because in
her will Sarah left him all of her personal property except for some furniture and jewelry that she
left to friends and other members of her family. Ann now regrets that she was so unforgiving and
did not welcome Sarah’s overture to settle the discord between them. She would like to have the
portrait that she claims Sarah gave her. The issue in this case is whether there was a valid gift inter
vivos. You have found three cases from your state:6

6The three cases are based on In re Will of Gorden, 27 N.W.2d 900 (Iowa 1947); In re Marriage of
Elam, No. 3- 971/03-0221, 2004 Iowa App. LEXIS 308 (Feb. 27 2004); and Gray v. Roth, 438 N.W.2d
25 (Iowa Ct. App. 1989).

49
In re Will of Bratton (1947)

Martha Bratton died in 1944. In her will, she left her estate to her nephew,
James Abbott, and her sister, Lucille Nichols. Previously, Martha had sent James
$5,000 along with a letter instructing him that at her death, he was to divide the
money equally among himself, his brother, and his two sisters. The executor of
Martha’s estate filed a petition with the court, asking that the $5,000 be adjudicated
as part of the deceased’s estate. The trial denied the petition and the executor
appeals. We affirm.
The money that Martha entrusted to James was a valid gift inter vivos. A
gift inter vivos, by definition, must be completed during the lifetime of the donor.
Martha’s letter showed her clear intent to make such a gift. By giving the money to
James with instructions to hold it for the donees, Martha relinquished all right to
and dominion over the money. Direct delivery to the donees is not required; it may
be made to a person acting as agent or trustee for the donees. If the gift is complete,
it will not be defeated by postponement of the enjoyment of the gift until the death
of the donor.

Nielson v. Keating, Executor (2002)

Lucy Nielsen appeals from a decision by the trial court that certain bonds
were the jointly owned property of Lucy and her ex-husband Frank Nielsen, now
deceased. The Nielsens were recently divorced. Shortly after the divorce, Frank
was killed in a boating accident. In the dissolution of marriage proceeding, the court
concluded that the bonds were co-owned by husband and wife and therefore marital
property. Lucy appeals, claiming that the bonds were an inter vivos gift to her.
The decedent owned the bonds at the time of his marriage to Lucy. After
the marriage, he had the bonds reissued in his name and Lucy’s name. State Code
§ 235.2 provides that in a dissolution of marriage proceeding, the court “shall divide
all property, except inherited property or gifts received by one party, equitably
between the parties.”
The trial court correctly held that there was no valid gift of the bonds from
the decedent to Lucy. A gift requires donative intent and delivery. The donor must
relinquish title, dominion, and control of the gift. By leaving his name on the bonds
as co-owner, decedent retained control of the bonds and could have cashed them at
any time. Affirmed.

In re Marriage of Judson (2004)

Lucretia Judson appeals from a provision of the divorce decree dissolving


her marriage to Edwin Judson. In the contested provision, the court granted Edwin
title to an airplane that Lucretia purchased with her separate funds while she was
married to Edwin. The court concluded that the airplane was a gift from Lucretia to
Edwin, based on the following evidence: Lucretia had title to the airplane, but she
was not a licensed pilot and never flew the plane. Edwin, who is a licensed pilot,
flew the plane regularly and took charge of maintenance, repairs, and payment of
fees and taxes. Lucretia had also given Edwin a framed picture of the airplane with

50
the caption “Edwin’s Private Airplane,” which hung on the wall in his office. State
Code § 235.2 provides that in a dissolution of marriage proceeding the court shall
divide all property “except inherited property or gifts received by one party.”
The requirements of a valid gift are: donative intent, delivery, and
acceptance. Acceptance is usually presumed if the gift is beneficial, unless there is
evidence to the contrary. Intent is the controlling element, but delivery must be as
complete as the circumstances allow. Because Lucretia gave Edwin control of and
free access to the airplane and denominated it “Edwin’s Private Plane, “thus
fulfilling the elements of intent and delivery, we find no error in the trial court’s
decision that the plane was a gift. Affirmed.

1. List the rule(s), and elements of the rule(s), in this problem.

2. Are there any elements you would exclude from analysis because they are “givens”? If
so, which ones?

3. Identify the issues and sub-issues (if any) in the problem.

The chart below identifies the elements, issues, and sub-issues.

Elements Chart
Elements of a gift inter vivos Element satisfied?
Intent to make a gift Yes
Delivery Maybe
Relinquishment of dominion and control Maybe
As complete as circumstances allow Maybe
Acceptance Maybe
During the lifetime of the donor Maybe

Comments: By leaving a message that Ann could get the key to her studio and take the
portrait, Sarah arguably relinquished dominion and control and made a delivery as
complete as the circumstances allow. Ann probably did not accept until after the death of
the donor, but a court could presume prior acceptance because she did not explicitly reject
the gift and it is beneficial to her.

51
8
Common Law Analysis

Changes from the Sixth Edition

There are no major changes from the sixth edition.

Teaching Notes

Some teachers find it helpful to teach this chapter in conjunction with Chapter 10
(Reaching a Conclusion). One of the authors works with students to combine the elements
charts demonstrated in Chapter 7 (Identifying & Selecting Issues for Analysis) with the
analysis charts demonstrated in Chapters 8 (Common Law Analysis) and 9 (Statutory Analysis)
into a single “elements and analysis chart.” Developing such a chart can be a very effective
pre-writing technique, and it is easier for some students to start with the familiar chart format
rather than with a traditional outline. A sample “elements and analysis” chart based on the
shopkeeper’s privilege statute and cases in Exercises 6-F and 6-G appears in Appendix III.D of
this manual.

Suggested Approach to Exercises

Exercise 8-A

This exercise may work better in class (without prior announcement) than as a writing
assignment because Chapter 14 (Drafting the Analysis) uses both of the libel sub-issues from
this chapter to teach drafting of the Discussion. Students are less likely to peek ahead to Chapter
14 for clues when you use the exercise this way. On the other hand, using this exercise means
that the class will better understand the analysis of this sub-issue when they reach Chapter 14.

52
1.

Falsity of the Fliers


CASE BRIEFING CHART
Our Case White Willow
Facts Dooley wrote a flier that The defendant wrote a While the parents of a
said his landlord had a letter to a construction four-year-old child were
“long history of criminal company president in a battle over custody of
convictions.” The accusing the company’s the child, the husband
landlord had received employees who had took the child without the
three notices from the worked on his house of wife’s consent and
local housing stealing a watch. The traveled with him to a
commission for watch had actually been different state. The wife
inadequate lighting and stolen by a friend of an wrote a letter to her
locks, and he had made employee when he picked neighbors stating that he
the repairs. He had no her up from work. had kidnapped her child.
criminal record.
Holding The letter was not false. The letter was not false.
Reasons Minor inaccuracies are not Technically, the husband
and enough to render the letter did not kidnap the child
Policies false. Because of the in the legal sense because
relationship between the he did not violate a
thief and the employee, the custody order. However,
letter was substantially errors in legal
true. terminology are
immaterial if the
statement is substantially
true.

2.

Falsity of the Fliers


ANALYSIS CHART
Why Element is Met Why Element is Not Met
(Fremont’s Position) (Dooley’s ’s Position)
Facts The statements were false. Fremont had Like the statements in White and
no criminal record. He had only received Willow, the statement in the fliers was
notice of three housing violations, which substantially accurate. Fremont
he corrected. In White and Willow the received three notice of violations of
inaccuracies were minor. In this case, the the housing code and only corrected the
inaccuracy was major because Fremont deficiencies after he was threatened
had never been accused or convicted of a with legal action.
crime.

53
Reasons This case differs from Willow and In Willow the defendant had used the
and White. In Willow the statement was true term “kidnap” to refer to her husband’s
Policy even though the defendant misused the taking their child to another state
legal term “kidnap.” In White the without her permission. The court said
statement was true because of the close that errors in terminology do not make
relationship between the employee and the statement false. Similarly,
the employee’s friend. Neither line of Dooley’s use of the term “criminal
reasoning applies here. There is a great record” was an error in terminology,
difference between having a long but the statement was substantially
criminal record and being put on notice true.
of three housing violations, which were
later corrected.

3.
 Determine how the facts of the decided cases support Dooley’s position. It is
often helpful to remind students that they must start their analysis of the usefulness
of cases by understanding the holdings. Both Willow and White held that the
allegedly libelous statements were substantially accurate. That is the holding
Dooley wants. Thus, to analyze the strength of Dooley’s position, the student’s job
is to develop explanations of how these cases are analogous to this situation. The
fact patterns in the two cases are similar enough to let a teacher begin to talk about
synthesizing, or analyzing several cases at the same time. You could thus ask students
to apply each case separately to Dooley’s situation, and then analogize them together.
The best answers will explain why the statement that Fremont has “a long record of
criminal violations” is substantially the same as a statement that he had three notices
of violation for housing code violations and had to be threatened to correct them. To
say that someone is criminal, students sometimes argue, is to say that the person has
done bad things, and does not necessarily mean that the person has been convicted
of a crime.

 Determine how the facts of the decided cases support Fremont’s position.
Students will generally find it easier to make distinctions. One approach is to ask
students to explain the differences between each case and Dooley’s situation, and
then explain how the two cases taken together are different from Dooley’s situation.
This type of work again anticipates the synthesis lesson in Chapter 12 (Describing
the Law). Students generally make these arguments by showing that the statement
in the fliers is much less accurate than the statements in Willow and White.

 Determine how the reasons and policies of the decided cases support Dooley’s
position. The Willow court reasoned that legal terms like kidnapping should be
analyzed as they would be ordinarily understood rather than as they would be
understood in a technical legal context. Similarly, Dooley might argue, the
statement about Fremont’s criminal record as a landlord would be understood, by
ordinary people, as meaning only that Fremont was an irresponsible landlord. The
implicit policy in White is the court’s unwillingness to impose libel liability on
persons for minor inaccuracies. The inaccuracy in this case, Dooley can argue, is
similarly minor.

54
 Determine how the reasons and policies of the decided cases support Fremont’s
position. Freemont would emphasize the other side of the same policies. Thus, if
someone is substantially wrong, he should be held liable for libel. The gist of the
matter is that Dooley’s statement was false. As the White court noted, receipt of
false statements by third persons is what makes libel so damaging.

 Evaluate the strength of Dooley’s case. Using the Chapter 10 factors for reaching a
conclusion, the evaluation looks like this in summary form:

 A decision that the statements are false involves the least extension of existing
law. The “long record of criminal convictions” language would make it difficult
for a court to hold that Dooley’s statement was mere hyperbole.

 A decision that the statements are false is most consistent with the basic purpose
of the libel law—protection of a person’s reputation.

 The fairness issue in this case is more of a toss-up because each party can make
compelling arguments (it was an accident vs. it hurt my reputation).

Thus, there is strong support for the conclusion that the statements are false.

55
Exercise 8-B

The factual and legal materials for this exercise are also used in Exercises 14-B, 17-A,19-B, and 23-C.

1.

Proximate Cause
CASE BRIEFING CHART
Our Case Moulton Mallard
Facts Quale developed a The patient went to the Mallard became
headache after being hospital because of paralyzed due to a
struck by a truck. His severe abdominal pain. degenerative disease
mother took him to The doctor diagnosed that her doctor failed to
the hospital and the the cause as a stomach diagnose and perform
doctor ordered x- “bug,” gave her pain surgery to arrest, which
rays. The x-rays did medicine, and released would have given her a
not show a skull her. She died several possibility of recovery.
fracture and the hours later from
doctor released him. hemorrhaging caused by
The doctor did not an intestinal obstruction.
examine the back of Prompt surgery would
his head, where there have allowed her to
was a red mark. recover.
Quale died the next
day from
hemorrhaging due to
a basal skull fracture.
Holding The doctor’s negligence The doctor’s negligence
was the proximate cause was not the proximate
of the death. cause of the patient’s
paralysis.
Reason The doctor prevented a There is proximate
s and substantial possibility of cause when the act or
Policy the patient’s survival. failure to act more
But for his negligence in likely than not caused
failing to perform the result. Mallard
surgery, the patient probably would not
would have survived. have recovered even if
the doctor had promptly
performed surgery.
Doctors should not be
held liable based on a
mere possibility.

56
2.

Proximate Cause
ANALYSIS CHART
Why Element is Met Why Element is Not Met
(Quale’s Argument) (Doctor’s Argument)
Facts Quale had one chance out of two, This case is distinguishable from
or maybe a little higher, of Moulton. A fifty-fifty chance of
surviving. Like the patient in survival is not a substantial
Moulton, Quale had a substantial possibility. Like the patient in
possibility of recovering. Mallard Mallard, Quale had only a
is distinguishable because in that possibility of recovering if the
case the patient only had a mere doctor had performed surgery.
possibility of recovering.
Reasons Because it was more likely than not The court in Mallard noted the
and that Quale would have survived if injustice of holding physicians
Policies the doctor had accurately accountable for harm they did not
diagnosed and treated him, the cause and could not prevent and
doctor’s negligence was the refused “as a matter of public policy,
proximate cause of his death. to hold a physician liable on a mere
possibility.” One chance out of two
of survival is a mere possibility.

3. This is an extremely close case, but Farmer is most likely to prevail. The cases are based
on a “more likely than not” standard, which is the traditional common law causation
standard. A court is likely to find that a 1 in 2 chance does not quite meet that standard.
The Mallard case supports that conclusion. The Moulton case, on which Quale must rely,
is distinguishable because the patient in Moulton would definitely have recovered. The
substantial possibility language in Moulton suggests a weaker standard, but the Mallard
court uses “substantial possibility” and “more likely than not” interchangeably. Students
will reach different conclusions, however, depending on the weight they give to Quale’s
expert’s opinion that Andrew’s chance of survival might have been “a little higher” than
50/50.

The policies underlying the cases—not holding a doctor liable based on a mere possibility,
and not requiring plaintiffs to show causation with certainty—probably tip in favor of
Farmer. The “more likely than not” standard is consistent with both policies.

Fairness is a toss-up here. The doctor was negligent, but it is not clear whether the boy
probably would not have survived or had a slightly better than 50/50 chance of survival.
Although this factor is closer, the first two tip the balance slightly toward Farmer.

57
Exercise 8-C

The factual and legal materials for this exercise are also used in Exercises 8-C, 14-C,
17-B, and 23-E.

1. Suggest to students that in order to identify the issues, they should begin by listing the
applicable rules and dividing the rules into component elements.

 The rule from Shover is that a property owner may not use the property to cause
injury to the legal rights of another.

 The rule from Blum is that a property owner’s use of the land is a nuisance if it has
an unreasonable effect on neighboring property.

 The rule from Cassells is the same as Shover.

A synthesized rule incorporating each element would look like this: A property owner
is liable for nuisance if (a) the owner’s use of the land caused injury to (b) the legal
rights of another property owner and (c) the effect of the use of the land was
unreasonable.

Nuisance
ELEMENTS CHART
Element Facts of Our Case Element Met?
The property owner’s use of Elliot erected two billboards that Yes
the land caused injury to block the sun from Greenleaf’s
solar panels.
the legal rights of another and (Whether Greenleaf has a legal Maybe
right to the free flow of sunlight is
a question of law.)
the effect of the use was Greenleaf must pay substantially Maybe
unreasonable. more in utility bills, and the
shielding of the sun will likely
have an adverse effect on his
business, which specializes in
solar design for residences.
Greenleaf uses his building as a
model for clients interested in
solar energy for their homes. The
billboards may negate this
promotional plan.

58
2. a.

Right to the Free Flow of Light


CASE BRIEFING CHART
Our Case Shover Cassells
Facts Greenleaf owns an Shover’s neighbor Cassells International
architectural firm that excavated land adjacent owns a resort hotel on
specializes in solar to his and caused the ocean beach. A
design for residences. Shover’s land to cave neighboring hotel owner
The firm is located in in, destroying a barn planned to build a ten-
a building that and some fences and story addition to its
Greenleaf retrofitted greatly devaluing the hotel that would block
with solar panels. He land. Shover sued for the sun from the
uses the building to compensation. beaches and other
promote his business. sunbathing areas.
Cassells brought suit to
enjoin the addition.
Holding Shover is entitled to The court refused to
compensation for the enjoin construction of
damage to his land. the addition.
Reasons A landowner must not Traditionally, a
and use his property to landowner has no right
Policies injure the legal right of to the free flow of light.
another. Shover has a Although there is an
legal right to the lateral increasing need for solar
support of his land. energy, that
consideration does not
apply to this case.

Unreasonable Interference
CASE BRIEFING CHART
Our Case Blum
Facts Greenleaf’s architectural firm Blum owns a hog farm adjacent to a
specializes in solar design. His landfill. The noise and vibration of the
firm is located in a building that defendant’s trash-hauling trucks caused
he retrofitted with solar panels the conception rate of the sows to
and uses to promote his business. decrease from eighty percent to thirty
The billboards block the sunlight, percent.
causing an increase in utility bills
and a potentially significant
decrease in clients.
Holding The use of the defendant’s property was
unreasonable and constituted a
nuisance.

59
Reasons Reasonableness is determined by the
and effect one’s use of property has on
Policies neighboring property. Courts will also
balance the interests of each landowner.
In this case, the defendant’s use of the
land essentially destroyed Blum’s hog
farm and was therefore unreasonable.

b.

Right to the Free Flow of Light


ANALYSIS CHART
Why Element is Met Why Element is Not Met
(Our Client’s Case) (Opponent’s Case)
Facts The rule in Shover is broad enough Greenleaf’s claim that shielding the
to include the right to the free flow sun from his building will have a
of light. Cassells is distinguishable negative effect on his business is
because it concerned the shielding similar to the hotel owner’s claim in
of the sun from beaches, not solar Cassells that a ten-story addition to
panels. Also, the damage to another hotel would shade the beaches
Greenleaf is much greater than the and other areas where his guests
damage suffered in Cassells. In sunbathed. The court rejected this
that case the hotel owner claim because here is no legal right to
anticipated potential economic loss the free flow of light.
and inconvenience to its guests. In Shover is distinguishable because it
this case Greenleaf’s business concerned the right to the physical,
might be destroyed because lateral support of land.
potential customers can see that
solar energy will not work if
neighbors can block the sunlight.
Even if it survives, it will be
“greatly devalued” like the
property in Blum

60
Reasons The traditional rule that there is no The rule in Cassells is the traditional
and right to the free flow of light was common law rule. The reasons for the
Policies formulated before the need to rule are just as valid today. Requiring
develop alternative energy sources that every property owner allow the
and should not be applied to this free flow of sunlight onto neighboring
case. The court in Cassells property would make building
recognized the increasing need for construction in high density areas
solar energy, citing Cohen v. economically unfeasible and would
Andrus. In Cohen the court infringe on individual property rights.
recognized a right to the free flow
of light in a case where a proposed
construction would block the
sunlight from a neighbor’s solar
collector. Cassells did not involve
the use of solar energy, however,
and the court decided to follow the
traditional rule.

Unreasonable Interference
ANALYSIS CHART
Why Element is Met Why Element is Not Met
(Our Client’s Argument) (Opponent’s Argument)
Facts Just as the landfill owner’s trucks in The interference with Greenleaf’s
Blum essentially destroyed the solar panels is an inconvenience, but
neighboring hog farmer’s business it is not a destruction of his business,
by causing the sows’ conception as was the case in Blum.
rate to decrease from 80% to 30%,
the billboards are essentially
destroying Greenleaf’s business by
making his promotional appeal less
attractive and increasing his cost of
doing business substantially.
Reasons In balancing the interests of the It is unreasonable to ask Elliot to
and respective landowners, Greenleaf remove his property and risk losing
Policies has a greater interest in saving his substantial business.
business than Elliott does in
erecting billboards on the adjacent
land. As the Blum court noted, an
action that amounts to a destruction
of a business is per se unreasonable.

c. Greenleaf’s position regarding the right to the free flow of light is weak because
of the common law rule explained in Cassells. He would have to convince the
court to reject an existing widely recognized rule and formulate a new one that

61
d. only one court has adopted. His public policy argument that the courts should
promote the use of alternative energy sources is not likely to overcome his weak
legal position. Many of our students, on the other hand, believe Greenleaf’s
position is amply justified and want him to prevail. Although some of us believe
Greenleaf ought to prevail, we use this part of the exercise to emphasize the
importance of dispassionate analysis.

Greenleaf’s position regarding unreasonable interference is probably stronger


than Elliot’s. In balancing the interests of the landowners, the scales tip in
Greenleaf’s favor because his entire business is at stake. The court may also
consider that Greenleaf’s solar energy promotion has more social utility than
Elliot’s billboards.

After class discussion of this exercise, you may wish to hand out the annotated
sample discussion in Appendix I.B of this manual. It illustrates how to combine
the answers to the questions into an organized discussion.

Additional Exercise

This exercise focuses on factual analogy and distinction and further illustrates the
importance of the policies underlying the rule.
One approach to the first issue, whether the pond is a natural or artificial condition, is to
explain the close connection between the reason for the rule and the facts. The reason courts
impose no duty of reasonable care for natural conditions is that children are presumed to
understand the dangers they pose. The question then becomes whether the alterations to the
Mayfields’ pond so changed its character that children would not appreciate the danger because
it was no longer natural.
Another approach is to discuss the difficulty of specifically defining the scope of the
term “natural condition.” How many and what kind of alternations are necessary before a body
of water becomes artificial? Similarly, the second issue hinges on the scope of the duty. What
precautions must a landowner take in order to protect trespassing children? These questions
cannot be definitively answered. The only reasonable approach is to ascertain the parameters set
by the courts in prior cases and consider whether the facts of your case fit within those
parameters.

Exercise 8-D

Paul and Judy Mayfield own a house and two-acre lot adjacent to an elementary school.
When they bought the property, it contained a small pond, used by the previous owners as a
watering hole for cattle. They turned it into an ornamental pond by excavating the sides to
create a gradual slope, planting various species of water lilies, and stocking it with goldfish.
They installed a recirculating pump and built a dock that runs from the edge of the pond to the
center. The pond is roughly circular and about forty feet in diameter. It is six feet deep at the
center and tapers to a depth of about eight inches at the edges. Paul and Judy use the dock for
sunbathing, entertaining guests, and servicing the pump when necessary. After the renovation
of the pond was completed, they installed a four-foot-high cedar picket fence around the one-
half acre in the backyard where the pond is located.

62
Earlier this year, several children from the elementary school, including Alan Finch,
climbed over the fence, went down to the pond, and walked out onto the dock to watch the
goldfish. Judy Mayfield told the children to leave her property and stay away from the pond.
After that incident, she put up a sign along the fence that read “DO NOT CLIMB FENCE.”
Several weeks later, on a day when the Mayfields were not at home, Alan Finch and two
friends decided to visit the pond again. Alan had some goldfish food and wanted to feed the
fish. At one end of the yard, they found a gate standing slightly ajar, entered through the gate,
and went down to the pond. The gate through which they entered is the only gate. It has a latch
but no lock. Alan went to the end of the dock, slipped, and fell into the pond. Since Alan could
not swim, his friends jumped in and attempted a rescue. After several minutes, they succeeded
in pulling him onto the dock. Alan survived but suffered brain damage because of the
deprivation of oxygen to his brain.
Alan’s parents, Joseph and Carol Finch, have asked you whether the Mayfields are
liable for Alan’s injuries. You have found the following cases from your state:7

Vernon v. Earnhart (1984)

This wrongful death action arose out of the drowning of five-year-old


Leroy Vernon in a pond located on the property of Max and Sarah Earnhart.
James and Rosalyn Vernon, who live next door to the Earnharts, were outside
working in their garden when they noticed that Leroy was missing. One of their
other children said that Leroy had gone next door to look at the ducks on the
Earnharts’ pond. The Vernons rushed over to the pond but were too late to save
Leroy. He had apparently waded into the pond, lost his footing, and drowned.
The Vernons maintain that the pond is an attractive nuisance and that the
Earnharts were negligent in failing to anticipate and guard against the danger it
presented to children. The trial court dismissed the Vernons’ suit against the
Earnharts for failure to state a cause of action. We affirm.
A possessor of land is liable for injuries to children trespassing on the
land only if the injuries were caused by an artificial condition on the land and
the possessor failed to use reasonable care to eliminate the danger or otherwise
protect the children. A possessor of land is not liable for natural conditions on
the land. This distinction between artificial and natural conditions is based on
the premise that children are presumed to appreciate the danger of conditions
which occur in nature.
We agree with the trial court that the pond, which was constructed by the
Vernons for the purposes of swimming and boating, is not an artificial
condition. Changes in natural environments do not create an artificial condition
where the affected terrain duplicates nature. The pond is seven feet deep at the
center and has a sandy beach and a sloping sandy bottom. The only artificial
characteristic is the cement walkway. We cannot characterize the entire pond as
artificial based on such a minimal change in its natural condition.

7
The cases are drawn from Johnson v. Washington County, 506 N.W.2d 632 (Minn. Ct. App. 1993), aff’d.,
518 N.W.2d 594 (Minn. 1994); Adams v. Atlantic Faith Memorial Church, Inc., 381 S.E.2d 397 (Ga. Ct. App.
1989); Gregory v. Johnson, 289 S.E.2d 232 (Ga. 1982); and Watts v. Murray, 43 So. 2d 303 (La. Ct. App.
1949).

63
Ward v. Kirlin (1992)

Steve and Grace Kirlin appeal from a decision by the trial court finding
them liable for the death of a four-year-old child who wandered onto their
property from the neighboring property and drowned in their swimming pool.
The child’s parent, Sally Ward, brought an action in negligence against the
Kirlins for maintaining an attractive nuisance. The issue on appeal is whether the
Kirlins owed a duty to the trespassing child to protect him from the danger
posed by the swimming pool and if so, whether there was sufficient evidence
that they breached that duty.
The Kirlins’ house is on a corner lot in a residential area. The pool is in
the side yard, visible from the street, and is equipped with a diving board and
slide. The Kirlins had failed to erect a fence or other barriers around the pool,
even though they had been warned that the presence of the pool and its
playground-like equipment was likely to attract children.
The rule, long observed in this state, is that the attractive nuisance
doctrine does not apply to bodies of water such as rivers and streams. Requiring
landowners to alter land in its natural state for the benefit of trespassers
would place an unacceptable burden on them. The rule, however, has not been
considered in the context of a swimming pool. We hold that the attractive
nuisance doctrine does apply to landowners who maintains a swimming pool on
their property and that they have a duty to exercise reasonable care to guard
against harm to trespassing children who cannot appreciate the danger posed by
the artificial condition. There is no doubt that the deceased child in this case was
a trespasser, since he came upon the property uninvited, and that he was too
young to appreciate the danger. The failure of the Kirlins to take any steps, such
as building a fence, to protect children who might foreseeably come upon their
property amounts to a breach of the duty of reasonable care.
Affirmed.

Saylor v. Renfro Art Museum (1999)

The trial court awarded summary judgment to Renfro Art Museum in a


negligence claim filed against it by Phillip Saylor, whose six-year-old son
drowned in a small ornamental pool located in a fenced, wooded area owned by
the museum and adjacent to the apartment complex where the Saylors live.
Saylor appeals, claiming that the museum maintained an attractive nuisance and
failed to exercise reasonable care to eliminate the danger to children posed by
the pool. The pool is located in a formal garden underneath a fountain. It is
made of concrete and is approximately four feet deep and eight feet in diameter.
The pool and garden are hidden by woods. A chain link fence surrounds the
museum property and another chain link fence surrounds the garden. The Saylor
child entered the museum property without permission. While wading in the
pool, he apparently slipped, hit his head on the concrete, and drowned
We find Saylor’s claim that the museum failed to exercise reasonable
care to be without merit. By erecting two fences, it did all that could reasonably
be expected to prevent unauthorized entry. Despite the tragic nature of this

64
occurrence, we cannot place upon a landowner the burden of taking precautions
against every conceivable danger to a child who ventures uninvited onto the
property.
Affirmed.

1. Identify the rule(s) and elements of the rule(s) in this problem.


2. For each issue, how do the facts support the Finches’ case? the Mayfields’ case?
3. For each issue, how do the reasons and policies of the cases support the Finches’
case? the Mayfields’ case?
4. Evaluate the strength of the Finches’ position on each issue.

Suggested Approach to Exercise 8-D

1. A landowner is liable for injury to (1) trespassing children (2) if the injury is caused by
an artificial condition on the land, and (3) the landowner failed to use reasonable care to
eliminate the danger or protect the children. Element (1) is given. Elements (2) and (3)
are issues.

2/3. Issue 1: Is the ornamental pond a natural condition on the land?

Case support for the Finches

Vernon is distinguishable. In that case, the cement walkway was the only artificial
addition to a pond that duplicated nature. The Mayfields’ pond has a dock, a
recirculating pump, water lilies, and goldfish. Taken as a whole, these additions
changed the pond from its natural state.

The purpose of the attractive nuisance doctrine is to protect children. Vernon. The pond,
like the swimming pool in Ward with its “playground-like equipment,” is attractive to
children because of the goldfish and water lilies, whereas an ordinary, unaltered pond
would not be. Therefore, children are unlikely to appreciate the danger.

Case support for the Mayfields

Like the pond in Vernon, the Mayfields’ pond is a natural condition on the land. In
Vernon, the pond, even though constructed by the landowners, duplicated nature and
thus was not an artificial condition. The Mayfields’ pond was not even constructed by
them, merely reconfigured, and the additions to the pond, such as fish and water lilies,
are things commonly found in ponds and do not change its nature. Also, the dock and
pump, like the cement walkway in Vernon, should be considered minimal changes.

Issue 2: Did the Mayfields exercise reasonable care to eliminate the danger and
protect the children?

Case support for the Finches

This case is similar to Ward, where the court found a breach of the duty of reasonable
care when the landowners failed to erect a fence around their swimming pool. A fence

65
with an open gate is similar to no fence at all because the children had free access to the
yard where the pond was located. Although Judy Mayfield had warned the children to
stay away, she did not prevent their re-entry. The sign she erected did not forbid entering
through the gate, merely climbing the fence. Saylor is distinguishable because in that
case the owner installed two fences with no openings in them, and the pool was hidden
from view by trees, not visible to passing children like the Mayfields’ pond.

Case support for the Mayfields

The court in Ward implied that a fence is all that is necessary, and the court in Saylor
found that the owners exercised reasonable care by erecting fences. The Mayfields not
only erected a fence but also warned the children to stay away. After they had climbed
the fence previously, Judy Mayfield put up a sign saying “Do Not Climb Fence.” A
landowner is not required to take “precautions against every conceivable danger to a
child who ventures uninvited onto the property.” Saylor.

4. The Mayfields are likely to prevail on the first issue. Although they altered the natural
appearance of the pond more than did the owners in Ward, the changes are probably not
enough for a court to consider the pond artificial.

The Finches’ position as to the second issue is slightly stronger because of the open
gate. The Saylor case is strong support for the Mayfields, however. They had taken the
precautions of warning the children to stay away and not to climb the fence and were
not expected to guard against every eventuality.

66
9
Statutory Analysis

Changes from the Sixth Edition

There are no major changes from the sixth edition.

Teaching Notes

As with Chapter 8, some teachers find it helpful to use this chapter in conjunction with
Chapter 10 (Reaching a Conclusion). Others encourage students to use elements charts and
statutory analysis charts together as a pre-writing technique. The example in Appendix III.D of
this manual shows a chart built on a statutory rule as interpreted by two cases.
Consider asking your class to find the rules of statutory construction for the state in
which you are located. Some of us also hand out a summary of the principles explained in this
chapter, such as the outline below, or distribute a handout that describes some of the most
common canons of statutory construction.

Selected Rules of Statutory Construction

Legislative intent. Giving effect to legislative intent is the primary goal in statutory
construction and is “trumps” over every other rule. Courts use the following rules and
presumptions in trying to ascertain legislative intent.

1. Plain meaning. If the meaning of the statute is plain, the courts’ task is to apply it as
written. They will not stretch the meaning of a word, phrase, or definition beyond
its ordinary meaning since to do so would be judicial legislation.

2. Ambiguity. If the statute is ambiguous, courts will interpret it according to their


understanding of the purpose of the legislation. They will also make certain
presumptions, also known as rules of construction. For example:

a. Courts will presume that the legislature was aware of existing statutory and
common law on the same subject and will interpret the statute in light of prior
law.

b. Courts will presume that the drafters understood the commonly accepted rules
of grammar and punctuation and will interpret the statute according to those
rules.

c. Under the rule of ejusdem generis, courts will presume that a general term
following a specific list of items refers to the same type of items as those in the
list.

d. Courts will presume that the legislature did not intend an unjust or absurd result.

67
You might also point out that courts use the same general principles in interpreting
constitutional provisions. They will strive to give effect to the intent of the drafters and will first
look at the plain language of the provision in question. Courts will interpret amendments so as to
harmonize them with existing provisions and existing common law, if possible, and will not
construe a provision so as to render it purposeless or in conflict with other provisions. However,
punctuation, grammar, and sentence structure do not play as large a role in constitutional
interpretation as they do in statutory interpretation.

Suggested Approach to Exercises Exercise 9-A

The factual and legal materials for this exercise are also used in Exercise 14-D.

As the elements chart on page 135-136 of the text indicates, a public body exists when (1)
there is a state or local legislative body (2) that is empowered by law to exercise governmental or
proprietary functions. The first element focuses on what that body is and the second focuses on
what that body does. Thus, the discussion in the text for the second element focuses on whether
the four Conservative members could exercise governmental functions based on their majority
status even if they do not constitute a quorum. The discussion under the first sub-issue must focus
on whether they can be understood to constitute the council or a committee of the council. An
outline of the analysis might look like this:

 Determine how the language of the statute, and the facts of any cases
interpreting the statute, support Smith’s position. The statutory language is in Section 2(b),
and there are no cases interpreting that language. The statute applies to “any state or local
legislative body, including a board, commission, committee, subcommittee, authority, or
council.” Because the statute applies to the City Council, Smith would argue, it surely applies to
the majority of the Council. In the alternative, Smith might argue that the four Conservatives
constitute an informal committee of the Council.

 Determine how the language of the statute, and the facts of any cases
interpreting the statute, support the Conservatives’ position. The Conservatives would argue
that section 2(b) applies to the entire council, which must be represented by at least a quorum of
its members, or a committee of the Council. Because they do not comprise a quorum or a formal
committee, they are not a legislative body. The Conservatives could add that the McPhee case
involved meetings attended by an entire board of education.

 Determine how the policies of the statute, and the policies of any cases
interpreting the statute, support Smith’s position. Smith’s policy arguments would be similar,
in part, to those under the governmental functions sub-issue. Meetings of a working majority are
intended to accomplish “public business” and would probably have that effect. In some ways,
Smith’s most effective position may be based on the purposes of the committee language. The
statute’s inclusion of committees in its definition of public bodies recognizes that parts of a
council may have a profound impact on public business. If committee meetings must be made
public, and committees may not even include enough members to constitute a majority of the
council, then surely the meeting of the four Conservatives should be made public. Besides,
they constitute a de facto budget committee when they decide to operate in this way. Smith could

68
also focus on the McPhee rationale concerning the importance of opening preliminary meetings to
the public.

 Determine how the policies of the statute, and the policies of any cases
interpreting the statute, support the Conservative’s position. The Conservative’s arguments
would also parallel those made under the governmental functions sub-issue. The statute
recognizes the importance of informal meetings among council members. That is why it applies
only to the entire council or formal committees. It makes no sense to say that a legislative body
exists without a quorum. Committees ordinarily exist when the council itself appoints them, and
they are ordinarily bipartisan. A party caucus is different; there is no committee when the
members of one party meet to discuss their position, regardless of whether they constitute a
majority.

 Evaluate the strength of Smith’s position. The Chapter 10 analysis looks


something like this:

 Little or no extension of the law. The Conservatives probably have a slight edge
here because the four members do not comprise a quorum or a formal committee.

 Purposes of the law. Smith probably has an edge here because the statute is
intended to reach committee behavior and because this group is acting like a
committee.

 Fairness to the parties. Smith’s position is that the Conservative caucus wants to
exclude him from discussion that he was elected to participate in. The
Conservatives would argue that political parties cannot do business if their
meetings must be public. Smith could respond that this isn’t just a party meeting;
it is more like a committee meeting because it is intended to develop a specific
position on the budget that is likely going to be passed by the Council. Smith
probably has an edge here too.

As a result, Smith is more likely than not to prevail on this sub-issue.

Exercise 9-B

The strongest argument for the motel owner is based on the plain meaning rule. Because
the statute is clear on its face, there is no need to use a rule of construction such as ejusdem generis
to ascertain its meaning. The ninety-day statute of limitations applies to recovery of “personal
property . . . left at a hotel or other public lodging.” The car and trailer fit within this category.
They are personal property and they were left at a motel.
An alternative argument is that even if the language is ambiguous, it should be read to
include vehicles because a traveler is likely to have a vehicle with him when he stays at a motel.
The statute does not limit the category to property brought into a room. It includes any personal
property left “at” a public lodging. This interpretation comports with the implicit intent of the
statute—to limit liability to a period of time within which the traveler discovered or could
reasonably have discovered that his property was missing.

69
The second argument is considerably weaker than the first. If the court rejects the plain
meaning argument and engages in statutory construction, it is likely to find the construction
described above somewhat less credible than the construction applying the ejusdem generis rule
described on page 141 of the text.

Exercise 9-C

1. The phrase “such action as will effectuate the purpose of the Act, including, but not limited
to” is quite general, and reasonable minds could construe it differently. The broadest
construction is that it includes any remedy that furthers the purpose of the Act, which is to
eliminate employment discrimination. The narrowest construction is that it is limited to
putting the employee in the same employment situation he or she would have been in but
for the discrimination.

2. The ejusdem generis rule would apply here, and Jade Enterprises should argue for its
application. The remedies listed in section 6 are those that would restore employment
benefits the employee lost because of discrimination. Under ejusdem generis, the
Commission would only be authorized to order the employer to provide remedies of the
same kind or class. It would not be authorized to order the employer to provide other
kinds of remedies such as compensatory damages for emotional distress.

Apart from ejusdem generis, Jade would argue that the legislature is presumed to be aware
of the common law, including existing common law remedies. If the legislature intended
to replace or supplement these remedies, it would have said so.

The Commission could invoke the plain meaning rule. Section 6 authorizes it to order
actions “including, but not limited to” those that are listed. The Commission would argue
that this statutory language expressly defeats the ejusdem generis presumption. This
language gives the Commission the express authority to fashion other remedies.

A court is likely to presume that the legislature did not intend an unjust result. The
Commission could use this presumption to argue that it would be unjust in this situation to
award no compensation in light of the judge’s finding that Jade had engaged in sexual
harassment. The theory is that the legislature did not contemplate situations in which the
victim has no remedy except for compensatory damages; if it had, it would have provided
such a remedy.

In a real situation, both parties would research common law construing this and similar
statutes to see whether they could base an argument on the judicial presumption described
on page 141 of the text that the legislature was aware of relevant common law and prior
statutes when it enacted the current statute.

Both sides will also argue that legislative intent supports their respective positions. (See
answer 3.)

3. The stated purpose of the Act, “to eliminate discrimination .by providing effective
remedies” to victims of discrimination, supports the Commission’s position because a

70
remedy is effective only if it discourages the employer from practicing discrimination.
Unless Jade pays compensatory damages to Preston it will pay nothing. Preston was a
temporary employee who worked for the full term of her employment. She lost no wages
or other employee benefits and she is not entitled to reinstatement. Allowing Jade to
practice sexual harassment without consequence defeats the purpose of the Act.

The Act is also remedial in nature. One of its purposes is to compensate the victim. The
only way to compensate Preston in this case is to allow compensatory damages.

4. Jade’s position is that the purpose of the Act is to restore to the victim any employment-
related compensation and benefits of which she was deprived because of discrimination.
In other words, the statutory purpose is to put her in the same position she was in before
the illegal acts occurred. In this case, Preston has suffered no loss of pay or benefits. She
is already in the same position she was in before. Although another purpose is to discourage
discrimination, the legislature did not intend to accomplish this purpose by requiring the
employer to pay to the victim anything other than what the victim has lost. Otherwise, it
would have given the Commission specific authority to award compensatory, and
perhaps even punitive, damages.

5. Although both sides have strong legal and policy arguments, Jade’s position is stronger.
The primary concern of the court will be to give effect to legislative intent. To find for
the Commission, the court would have to read the statutory language more broadly than
the legislature probably intended.

Exercise 9-D

The factual and legal materials for this exercise are also used in Exercises 14-E and 23-D.
When using this problem, make sure that the students are looking carefully at the wording of the
statute and are considering how the case law has interpreted or modified the statute.
1.

Application to Intervene
ELEMENTS CHART
Element Facts of Our Case Element Met?
The application was timely. The League’s application would Maybe
be filed six months after the city
denied Swift’s proposal for a
zoning amendment and one week
after the city indicated that it
would not represent the League’s
interest.
The applicant’s interest is or The League has an interest in Maybe
may be inadequately low-income housing. The mayor
represented. and city attorney have indicated
that they will no longer support
the low-income housing plan.

71
The applicant will be bound by The League has asked the city to Maybe
a judgment, which the court has set aside the eighteen-square-
defined as having an interest block area for low-income
that will be substantially housing, but it does not own or
affected. occupy any portion of the
property.

2. a.

Timely Application
CASE BRIEFING CHART
Our Case Halsey
Facts The League’s application would The application for intervention was
be filed six months after the city filed three weeks after the rezoning
denied Swift’s proposal for a suit was filed.
zoning amendment and one week
after the city indicated it would no
longer represent the League’s
interest
Holding The application was timely.
Reasons There was no unreasonable delay in
and filing the application.
Policies

Inadequate Representation of Applicant’s Interest


CASE BRIEFING CHART
Our Case Halsey
Facts The League represents welfare The applicants for intervention were
recipients and has proposed the owners of homes within 300 feet of
development of low-income the parcel that the appellee had
housing in the area in question. petitioned the Village to rezone. The
The city has indicated that it is no village did not purport to represent the
longer interested in the League’s applicants.
proposal.
Holding The applicants would be inadequately
represented.
Reasons The burden of satisfying this
and requirement is minimal. The
Policies applicants met the burden because the
village had not indicated any support
for their interest.

72
Applicant Will Be Bound By a Judgment
CASE BRIEFING CHART
Our Case Halsey
Facts The League has proposed the The application was filed by neighbors
development of low-income whose homes were within 300 feet of
housing in the area. It has the area.
invested a significant amount of
planning and negotiation in this
project on behalf of its members,
all of which will be nullified by a
decision in Swift’s favor.
Holding Applicants would be bound by the
judgment.
Reasons The applicants would be “bound by”
and the decision because their ability to
Policies protect their interests would be
substantially affected.

b.

Timely Application
ANALYSIS CHART
Why Element is Met Why Element is Not Met
(the League’s Position) (Swift’s Position)
Facts This case is similar to Halsey. In The intervenors in Halsey filed their
that case the intervenors filed suit application within three weeks after
three weeks after suit was brought. the suit was filed, and the court
In this case the League filed its decided that their application was
application only a week or two after timely. In this case, the League
the attorney general’s statement that waited six months to file its
the city might be wrong. application.
Reasons The application was timely because Because the League waited six
and it was filed a week or two after the months to file its application, the
Policies League learned that it would not be requirement that the application be
adequately represented. timely has not been met.

Inadequate Representation of Applicant’s Interests


ANALYSIS CHART
Why Element is Met Why Element is Not Met
(the League’s Position) (Swift’s Position)
Facts As an organization representing The League has not established that
welfare recipients, the League has its interest is or may be inadequately
an interest in the development of represented.
low-income housing.

73
Reasons The League’s interest is no longer In Halsey, the court noted that the
and adequately represented in the action village did not purport to represent
Policies because the city attorney and mayor the interests of the landowners. In
have now indicated that the city this case the city has the
may no longer be supporting the responsibility to represent the
low-income housing plan. interests of its people, including the
members of the League. The city has
not indicated that it has abandoned
the plan advocated by the League,
only that it will not represent narrow
special interest groups.

Applicant Will Be Bound By a Judgment


ANALYSIS CHART
Why Element is Met Why Element is Not Met
(the League’s Position) (Swift’s Position)
Facts The court in Halsey interpreted The League would not be bound by
“bound by” to mean that the an adverse judgment because the
intervenor’s “ability to protect their court is not being asked to adjudicate
interest may be substantially the League’s rights or
affected.” Like the property owners responsibilities.
in Halsey, the League’s interest in
developing low-income housing
would be adversely affected
Reasons A decision to allow an auto salvage In Halsey, the intervenors were
and yard in the area is incompatible with nearby property owners whose
Policies the residential plan that the League quality of life and property values
has been working on. A decision in would be adversely affected by the
Swift’s favor is also likely to result proposed rezoning. The League
in other commercial development in neither owns nor occupies the
the eighteen-block area. property involved. Therefore, its
interest would not be substantially
affected.

c. Students may conclude either way, but they should be able to explain why they have
decided that one side’s argument is stronger than the other. It may be helpful to work
through the Chapter 9 factors for at least one of these sub-issues.

74
Exercise 9-E

1. a. The relevant rule is in section 31(a), which prohibits:

 anyone other than a campaign committee member


 from contributing more than $2,500
 to the campaign of the winner of a primary election for the office of governor
 for any purpose
 after the date of such primary election.

2.

Election Campaign Finance Act Sec. 31(a)


ELEMENTS CHART
Element Facts of Our Case Element Met?
Anyone other than a campaign Rolf Odegaard was not a member of Yes
committee member Francine Odegaard’s campaign
committee.
contributed more than $2,500 Rolf contributed $41,995. Yes
to the campaign of a winner of a Francine won the general election. Maybe
primary election for the office of
governor
for any purpose The contribution was used to retire Yes
Francine’s campaign debt after she
won the general election.
after the primary election The contribution was made after Yes
Francine won the general election.

a. The following elements of Sec. 31(a) are given:

 Rolf Odegaard was not a member of his granddaughter’s campaign


committee.
 He contributed more than $2,500 to her campaign.
 The contribution was made after the primary election.

b. All of the elements except one are givens, as shown by the chart. The only sub-
issue is whether the statute applies, since the contribution was made after Francine
Odegaard won the general election for governor, not after the primary election.

3. Sec. 31(b) exception provides that:


• contributions
• by a member of the candidate’s immediate family
• to the candidate’s campaign committee
are exempt from the limitations in section 31(a).

75
Sec. 31(c) defines “immediate family” as including “a spouse, parent, brother,
sister, son, or daughter.

4.

Sec. 31(b) Exception to Election Finance Act Prohibition


ELEMENTS CHART
Element Facts of Our Case Element Met?
contribution Rolf contributed $41,995, Yes
by a member of the candidate’s Rolf is Francine’s grandfather. He Maybe
immediate family. raised her after her parents died when
she was very young, but he never
formally adopted her.

to the candidate’s campaign The contribution was made to the Yes


committee campaign committee after the general
election.

5. a.

Member of Immediate Family Sec. 31(c)


CASE BRIEFING CHART
Our Case Toland
Facts Rolf is Francine’s grandfather. He Raymond Swanson handled the finances
raised her after her parents died when of his father’s campaign for governor. He
she was very young, but he never deposited the funds he received, including
formally adopted her. a check from Alphonse Toland, into his
personal bank account and then wrote
checks to pay the campaign expenses.
Holding Toland violated the Election Campaign
Finance Act.
Reasons The “immediate family” exception to the
and Act does not apply because the son was a
Policies mere conduit of the funds. The purpose of
preventing corrupting influence is not
served when the donors are third parties
and the family member merely collects
the donations.

76
b.

Member of Immediate Family


ANALYSIS CHART
Why Element is Met Why Element is Not Met
(Odegaard’s Position) (Committee’s Position)
Facts Although Rolf never formally adopted The Act defines immediate family as a
Francine, he raised her from early “spouse, parent, brother, sister, son, or
childhood after her parents died. He is daughter. Because a grandfather is not a
also her grandfather, not an unrelated listed family member, Rolf Odegaard
adoptive parent and is thus part of her does not qualify for the exemption.
“immediate family” under section Therefore, he is not exempt from the
31(c). Toland is distinguishable. In $2,500 limit under section 31(b).
that case, the candidate’s son
contributed money he had received
from others, not his own money.
Reasons The policy of regulating political Section 31 of the Act is intended to
and activity set out in section 2 is regulate campaign financing, and to do
Policies inapplicable here because Odegaard so without jeopardizing candidates’
did not make this donation for any ability to run effective campaigns.
political favors. The policies behind Applying the section 31 restrictions will
the immediate family exception not adversely affect Francine
recognized in Toland apply here Odegaard’s ability to run an effective
because he stands in the place of the campaign because she has already won.
candidate’s parents. It would not be Although the policy of improving the
fair to deprive Francine Odegaard of integrity of the election process may not
her grandfather’s support and yet be at issue, the appearance of integrity of
allow other candidates the benefit of the election process is, as it is with any
the support of their parents. large donation.

Analysis of each side’s position under the statute and under the cases:

Rolf Odegaard is unlikely to prevail on the issue concerning section 31(a) spending
limits. The concerns regarding campaign impropriety in light of the court’s statement that
one of the purposes of the act is to “improve the integrity, and the appearance of integrity,
of the election process.” Alberts. Allowing a large contribution from one who is not an
immediate family member may impair this purpose, although Rolf could argue that the
stated exemption for family members eliminates any potential appearance of impropriety
under these facts.

Students may decide the “immediate family” issue either way. This issue comes down to
the “letter of the law” versus the underlying policy. Below is a summary of the arguments
for each side.

77
Language of the Act supporting the committee:

 Rolf Odegaard is a person other than a campaign committee member. His $41,995
contribution exceeds $1,000. He meets the statutory requirement that the contribution
be “for any purpose” because he contributed the money to retire a campaign debt.

 Rolf Odegaard is not a spouse, parent, brother, sister, son, or daughter. Because he is
not a member of Francine Odegaard’s immediate family, he is not exempt from the
$1,000 limit under section 31(b).

Facts of the cases supporting the committee:

 The court in Alberts indicated that the language “for any purpose” includes the purpose
of retiring campaign debts. Just as the contribution in Alberts was intended to retire a
campaign debt, Odegaard here obviously was attempting to retire his granddaughter’s
campaign debt.

 The Toland court said that the immediate family exemption would ordinarily apply to
the candidate’s son, one of the types of persons specifically identified in section 31(b).
Because a grandfather is not a listed family member, Rolf Odegaard does not qualify
for the exemption.

Policies supporting the committee:

 Section 2 of the Act is intended to regulate campaign financing, and to do so without


jeopardizing candidates’ ability to run effective campaigns. The section 31 restrictions
will not adversely affect Francine Odegaard’s ability to run an effective campaign
because she has already won.

 The committee will find additional policy support from the cases. Although the policy
of improving the integrity of the election process may not be at issue, the appearance
of integrity of the election process noted in Alberts is applicable here, as it is with any
large donation.

 Students may identify other policies not specifically stated in the Act or cases. The Act
purports to put all candidates on a level playing field, for example. Thus, any exceptions
to its application should be narrowly construed.

Language of the Act supporting Rolf Odegaard:

 Section 31(a) applies to the winner of a primary election, not to the winner of a general
election. This type of argument suggests, as students will see from the difficulty they
have with this part of the question, that it is hard to make an argument for Rolf
Odegaard based on the statutory language.

 Although he never formally adopted Francine, he raised her from early childhood after

78
her parents died. He is also her grandfather, not an unrelated adoptive parent. Rolf
Odegaard is thus part of her “immediate family” under section 31(c).

Facts of the cases supporting Rolf Odegaard:

 Section 31 has never been applied to contributions made after a general election. Thus,
this situation is distinguishable from that in Alberts. The reasoning in that case—that
large contributions near the end of a campaign can gain an improper level of influence
for the contributor—is not applicable when the election has already been won.

 Toland is not applicable here. In that case, the candidate’s son contributed money
he had received from others. The court held that the immediate family exemption did
not apply when a family member was merely a “conduit” for other people’s money. In
this case, by contrast, Rolf Odegaard donated his own money.

Policies supporting Odegaard:

 The policy of regulating political activity set out in section 2 is inapplicable here
because Odegaard did not make this donation for any political favors. Because he is
the candidate’s grandfather, the policies articulated in Alberts regarding the integrity or
appearance of integrity of the election process are also inapplicable. Further, the
policies behind the immediate family exception recognized in Toland apply here
because he stands in the place of the candidate’s parents. It would not be fair to deprive
Francine Odegaard of her grandfather’s support and yet allow other candidates the
benefit of the support of their parents.

79
10
Reaching a Conclusion

Changes from the Sixth Edition

There are no major changes from the sixth edition.

Teaching Notes

In our experience, students find it easier to compare and contrast than to draw
conclusions. This chapter is intended to help solve that problem. We have tried to provide
guidance without being mechanical.
Some teachers find it useful to teach this chapter in conjunction with Chapters 8
(Common Law Analysis) and 9 (Statutory Analysis). The fifth step in each of those chapters
requires students to draw a conclusion and defend it and specifically urges students to refer to
Chapter 10. One approach to classroom teaching is to work through each factor in this chapter
with the textual problems in Chapters 8 and 9.

Suggested Approach to Exercises

Most of the exercises in the book can be used to teach this chapter. We also use closed
memorandum assignments that the students are working on or have just completed.

Exercise 10-A

The factual and legal materials for this exercise are also used in Exercises 7-A, 12-A,
13-A, and16-A. Below are suggested answers. There may be more here than you can or want to
use. We include all of it to provide a reasonably complete analysis of the problem.

First principle: A position is stronger to the extent that it involves little or no extension
of existing law.

Issue 1: Was there a valid offer?

McKay: No. I didn’t intend to make an offer. I was only joking and any
reasonable person would have known it was a joke to offer a $100,000
yacht for $10,000. Green admitted as much when he said, “You can’t be
serious.” This case is analogous to the offer in Derek, which the court
found invalid because the offeree knew or should have known the offer
was a practical joke.

Green: Yes. I had no reason to know that McKay was joking. I expressed
surprise when I said “You can’t be serious” but not disbelief that the
offer was genuine, because immediately after that I asked for time to
consider the offer and raise the money. This case is different from Derek
because in that case the offeree knew the offeror’s reputation as a
practical joker. I did not know McKay had such a reputation.

80
Resolution of this issue depends on a straightforward application of the rule in Derek.
Did Green know or should he have known that the offer was not serious? The facts can be
read to favor either side. Derek is distinguishable, however, because the offeree in that
case knew the reputation of the offeror. For this reason, the first factor tips slightly in
Green’s favor. A decision for McKay would call for a slight extension of existing law.

Issue 2: Was there a valid acceptance?


McKay: No. Green accepted the alleged offer on the condition that I include my
golf clubs, a condition that was not part of the original offer. Because the
acceptance did not mirror the offer in every respect, it is invalid. In this
respect, this case is similar to Anselm, where the court held that a buyer’s
reply to a seller’s offer agreeing to the seller’s terms but changing the
delivery dates was a counteroffer, not an acceptance.

Even if the acceptance was valid in form, it was not sufficiently


communicated. When Green made his oral acceptance over the
telephone, I did not hear it because I was unconscious. This case differs
from Anselm in that regard. In Anselm the court considered delivery of a
written acceptance sufficient even though the offeror had not read it. The
offeree had done all that was required of him by delivering the written
acceptance to the offeror’s secretary. Green did not do all that was
required of him. He should have realized when I did not respond that I
had not heard his part of the conversation.

Green: Yes. I called McKay and said the words “I accept” right after he
answered the phone. That was my acceptance. After that, I asked him to
throw in his golf clubs, just to see how far he would go. Unlike the
offeree in Anselm, I did not make the golf clubs a condition to accepting
the yacht contract. It was just a suggestion.

Also, under the Uniform Commercial Code, which most states have
adopted, there is no mirror-image rule. My suggestion about the golf
clubs would be a proposal for an addition to the contract, not a
counteroffer or rejection.

My acceptance was also sufficiently communicated. In Anselm, the court


said that placement of written acceptance on the offeror’s cluttered desk
was sufficient. The offeree need not ensure that the offeror read it.
Similarly, I gave my acceptance plainly over the telephone. I had no duty
to ensure that McKay heard it.

McKay must argue that Green’s golf club “suggestion” is really a condition, and it is
phrased as such because it includes the conjunction “but.” Neither McKay’s argument
that the golf club reference is a condition, and thus a counteroffer, nor Green’s argument
that it is a mere suggestion requires an extension of existing law. Both interpretations are
possible according to the rules in Anselm, but the interpretation offered by McKay is

81
more plausible. Green’s argument for adoption of the UCC rule would be a change in
the law and weakens his position. As to this sub- issue, McKay’s position is stronger.

Green’s argument that his communication was sufficient according to Anselm requires
some extension of the law. A court might be reluctant to equate a telephone
conversation with a drunken acquaintance to a properly delivered written
communication to a business associate, despite some similarities. (In each case, the
offeree delivered the message, but the message was not received.) McKay has a slight
advantage here because Green’s argument requires the court to stretch the Anselm case
to cover quite different facts.

Issue 3: Was there sufficient consideration?


McKay: No. The court in Derek stated the rule that the consideration must not be
“so grossly disproportionate that it would shock the conscience of the
court to enforce the contract.” A payment of ten percent of the value of a
$100,000 yacht would shock the conscience of the court.

Green: Yes. The accepted rule is that any consideration, no matter how small,
will support a contract.

The initial problem here is not whether one side requires an extension of existing law,
but what the law is. McKay relies on dicta in the Derek majority opinion. Green relies
on a statement in the dissenting opinion in Derek in which the judge cites a treatise on
contract law. Assuming the majority stated the existing rule, McKay’s argument fits
squarely within it and his position would be quite strong.

Second principle: A position is stronger to the extent that it furthers the policies or
purposes of the law.

The policy arguments made here could apply to any of the issues or sub-issues in this exercise.

McKay: The purpose of contract law is to enforce serious agreements freely


entered into by the parties, not to turn a joke between friends into
an enforceable contract. I didn’t freely enter into this contract
because I made the offer in jest.

Green: The purpose of contract law is to give effect to the reasonable


expectations of the parties. My expectations were reasonable under the
circumstances.

Although both parties have stated valid purposes for judicial enforcement of a contract,
McKay’s position is weak because the test in this jurisdiction is one of reasonableness.
Therefore, his subjective intent is irrelevant. Furthermore, we know from the Derek case
that the court will enforce an agreement that is also a practical joke if the requirements
for a valid contract are met.

82
Third principle: When the law does not require a particular result, a position is stronger
to the extent that it involves a fair or just outcome for the parties.

The fairness arguments made here could apply to any of the issues or sub-issues in this
exercise.

McKay: Enforcement of this alleged contract would be unfair because I would be


losing $90,000 and Green would gain an unfair and undeserved
benefit—a yacht for one-tenth of what it is worth.

Green: Enforcement of this contract is fair because McKay initiated the


negotiations for sale by calling me and offering to sell his yacht. He
can’t complain now about unfairness because he led me to believe he
was serious.

One view of justice is that McKay carried the joke too far, raised Green’s expectations
unnecessarily, and should now follow through with his part of the bargain. Another
view is that Green would be getting a windfall if McKay is forced to sell Green his
yacht for a fraction of its value. Both views are defensible, but McKay’s position is
probably stronger because Green has not suffered any monetary detriment.

According to this summary, McKay’s position is stronger than Green’s. Green has a
less than even chance of proving all three elements of a contract, and neither the purposes of
the law nor the considerations of fairness and justice clearly favor either side.

Exercise 10-B

The factual and legal materials for this exercise are also used in Exercises 7-B, 12-B,
13-B, and16-B.

Some arguments that students may raise under each guideline follow, but these are not
meant to be exhaustive. Many of the arguments are described in this manual for Exercise 7-B.
You may find it helpful to appoint half the class to represent one side and half to represent the
other and then discuss how each of the factors would apply. When students are given a “client”
they sometimes find it easier to see the arguments involved.

Issue 1. Does the exemption set out in section 5(a) apply to the police department
records regarding the CLO?

First principle: A position is stronger to the extent that it involves little or no


extension of existing law.

Banner-Patriot: These records do not deal with the “detection and investigation of
crime.” Even if they are maintained for the Police Department’s internal
use “in matters relating to law enforcement,” they are analogous to the
reports the court found not to be privileged in Wheeler. In Wheeler, the
court held that a police department could not withhold reports
concerning the offense committed, surrounding circumstances,

83
witnesses, and investigating officers. The CLO records contain similar
kinds of basic information.

Police Dep’t: The exemption set out in 5(a) applies to the CLO files. The information in
the CLO files is analogous to the Supplementary Offense Reports in
Wheeler. to disclose this information would jeopardize the ongoing
effort to investigate criminal activities as well as potentially revealing
the names of informants. The paper is not seeking information about
specific crimes, as was contained in the Offense Reports in Wheeler, but
rather is seeking general information about an organization that may or
may not be involved in criminal activity.

This factor probably favors the Police Department because these records are
“internal records and notations” of the Department’s internal use “related to law
enforcement.” In addition, the records are more analogous to the records that the
Wheeler court said could be withheld.

Second principle: A position is stronger to the extent that it furthers the policies
and purposes of the law.

Banner-Patriot: The policy behind the Freedom of Information Act is to ensure that the
press and the public can know what activities their governing bodies are
engaged in. We do not live in a society where the government is allowed
to conduct its business in secret. This information was gathered by
public servants and belongs to the public. The CLO is potentially a very
destructive force in the community, and it derives some of its power
from its secrecy. The community needs to be informed of its activities
and the individuals involved.

Police Dep’t: The public’s access to information must be balanced with the
Department’s need to keep certain information confidential. The
Freedom of Information Act was not intended to hamper law
enforcement efforts or jeopardize the safety of police informants. That is
why the exception in 5(a) was included in the statute.

The Police Department’s argument that disclosure would interfere with an


ongoing investigation is likely to outweigh the newspaper’s public information
argument.

Third principle: When the law does not require a particular result, a position is
stronger to the extent that it involves a fair or just outcome for the parties.

Banner-Patriot: The press has a responsibility to keep its readers informed about
important matters in their community. The CLO may be a threat to
individuals or the community as a whole, but a story about the CLO
is difficult to write because of the secrecy with which the
organization conducts its affairs. The police files might contain
information that the paper will not be able to obtain any other way

84
and which is crucial to any story it may write.

Police Dep’t: The Department has invested two years in investigating the CLO, an
investigation that is not concluded as yet. to disclose the information
sought would place all of the effort expended to this point in jeopardy. If
and when arrests are made, the paper will receive the pertinent
information.

The fairness argument comes down a question of timing. So long as the


investigation is continuing, and there is a reasonable suspicion of illegal activity (as
there is here), withholding the information is probably the fairest result at present.

As a result, we think the Police Department could justify refusing to disclose the
records under section 5(a). We recognize that students might draw a contrary conclusion,
reasoning, for example, that the Police Department “investigation” is really just an ongoing
monitoring activity of a suspicious organization. Those students should be asked to explain,
however, why such an investigation doesn’t increase the likelihood of detecting criminal
activity that the organization directly or indirectly encourages.

Issue 2. Does the exemption set out in section 5(d) apply to the police department
records regarding the CLO?

First principle (existing law):

This factor probably tips in favor of the Police Department because of the
sweeping nature of the newspaper’s request and because the investigation is not yet
completed. Investigations like this usually involve people who are likely to be guilty of
wrongdoing as well as people who are suspected but are ultimately innocent. If it
received the Department’s files, the newspaper would have information about people
who have not committed any crime and who do not intend to do so; using such
information might constitute an invasion of their privacy. In addition, the police have
obtained information about membership of the CLO. Mere membership in an
organization does not ordinarily make that person a criminal, but disclosure of that
information is likely to be embarrassing, although probably not a “clearly unwarranted
invasion of privacy.”

Second principle (policies and purposes):

The purpose of the section 5(d) exemption is evident from its adoption only two
years after the Holcombe case. The newspaper will argue that those who voluntarily
join a paramilitary organization with racist views are in a radically different position
from that of a rape victim. On the other hand, without knowing who has been
investigated (other than the mayor’s wife), and what information is in the files, it is
impossible to say that only bad or guilty people will turn up in the police records. For
that reason, this factor also favors the Department.

85
Third principle (fair or just outcome):

Not disclosing the information is fair to the innocent people who are being, or
who have been, investigated. This result gives the Police Department an opportunity to
finish a proper investigation. Presumably, the people who have committed crimes will
be charged, so the fact that their names and activities are not now disclosed is
unimportant.

Thus, the section 5(d) exemption applies. This may be surprising to students, and there
may be some discussion about whether the files could somehow be divided. Some students
may see this as one of those problems where you reduce the scope of a sweeping request to get
at least some of what you want. A professor could generate an interesting discussion by asking
how that could be done effectively.

Exercise 10-C

The factual and legal materials for this exercise are also used in Exercises 7-C, 12-C, 13-
C, and18-B

Issue 1: Whether the contract complies with the statute of frauds


a. Whether the contract states with reasonable certainty and accuracy the terms and
conditions of all promises.

First Principle: A position is stronger to the extent that it involves little or no


extension of existing law.

The contract was in writing, as required by the statute of frauds, but it does not
fully comply with the requirement in Treacher that a contract must include all terms
and conditions. The written offer included the name of the property and the purchase
price, but it did not include a description of the property and the payment terms and
dates. The court in Treacher interpreted the law strictly, and Hughes’s position would
require some extension of existing law.

Second Principle: A position is stronger to the extent that it furthers the policies
and purposes of the law.

According to the court in Treacher, the statute of frauds serves two purposes: to
ensure that there is proof of the agreement and that the parties have thought carefully
about the proposed transaction before agreeing to it. Zoeller’s brief statement written on
a napkin probably does not satisfy either requirement. Because the statement lacked
certain terms and conditions, it does not serve as proof of the specific agreement. Also,
there was little time for either of the parties to carefully consider the transaction. This
principle therefore favors Hughes.

86
Third Principle: When the law does not require a particular result, a position is
stronger to the extent that it involves a fair or just outcome for the parties.

Both parties can argue that a decision in that party’s favor would be fair. Zoeller
could claim that an offer discussed by both parties several hours earlier and then agreed
to should not be invalidated. Hughes could claim that she was taken advantage of
because of her inebriated state or, alternatively, that she believed Zoeller was joking.
Hughes’s position is somewhat stronger. This reasoning also applies to the sub-issue of
whether the contract was signed.

b. Whether the contract was signed by the parties to be charged.

First Principle (existing law):

Zoeller wrote the offer, Hughes orally agreed to it, and her agreement was
recorded. However, neither party actually signed the contract, Because of the strict
interpretation of the statute in Treacher, this principle weighs heavily in Hughes’s
favor.

Second Principle (policies and purpose):

As stated earlier, the purpose of the statute is to ensure proof of the agreement
and careful consideration by the parties. The recording of Hughes’s spontaneous oral
acceptance might satisfy the first purpose because it proves that she agreed to the offer.
Her immediate agreement, however, does not satisfy the second purpose. As stated
above, it is likely that she did not give the offer serious consideration.

Third Principle (fair or just outcome):

See the explanation under the third principle above.

Issue 2: Whether the contract is unenforceable based on fraud.


a. Whether Hughes was justified in relying on Zoeller’s representations.

First Principle (existing law):

This principle favors Zoeller. Unlike the plaintiff in Divine, who was justified in
relying on the dance school representations because of her lack of education and
knowledge, Hughes was probably not justified. When she questioned the validity of the
figures and Zoeller admitted that there might be some mistakes, she did not make
further inquiries. Unlike the plaintiff in Divine, who had an eighth-grade education,
Hughes has a graduate degree in business and accounting and was capable of checking
the accuracy of Zoeller’s business records.

87
Second Principle (policies and purpose):

The purpose of the law is to protect persons who have been taken advantage of.
Although Hughes did have a chance to verify the figures that Zoeller presented, this
principle probably favors her. The court in Divine mentioned a lack of the exercise of
reasonable judgment as a consideration. Hughes was drinking when Zoeller made the
representations to her and intoxicated when she accepted. Therefore, she probably
lacked the capacity to exercise good judgment.
Third Principle (fair or just outcome):

This principle also favors Hughes. Zoeller took advantage of an intoxicated


patron by misrepresenting the facts. Although Zoeller could argue that he gave Hughes
the opportunity to think about the offer before he wrote it on a napkin, Hughes position
is probably stronger.

b. Whether Hughes suffered damage as a result of her reliance.

First Principle (existing law):

The sale price exceeded the market value of the property by $40,000, but the
tavern has made a profit of $25,000 each year for the last three years and profits may
double or triple in the near future. If the court takes future profits into account, it will be
difficult for Hughes to prove that she has suffered damage. If the court considers only
the difference between the sale price and the actual value of the property, this principle
favors Hughes.

Second Principle (policies and purpose):

The purpose of this requirement is to ensure that a person cannot challenge the
validity of a contract if that person has not suffered actual damage. If Hughes can prove
that she suffered damage because of the $40,00 difference between the sale price and
the market value of the property, this principle would weigh in her favor.

Third Principle (fair or just outcome):

The profit that Hughes would earn from operation of the tavern is unknown. The
profit may increase, but it could also decrease. For this reason, a decision that Hughes
suffered damage would be a just outcome for her. Although Zoeller would consider it
unjust, he would not be unduly harmed because he would still own the tavern and
receive the profits from it.

88
Part C
Basic Concepts of Legal Writing

89
11
Organization

Changes from the Sixth Edition

There are no major changes from the sixth edition.

Teaching Notes

The chapters in Part C build in a logical order to Chapters 14 (Drafting the Analysis)
and 15 (Revising and Editing). Some teachers may find it effective to teach these chapters in
sequence, and some may prefer to skip over some chapters and then return to them. We have,
for example, taught Chapters 11, 14, and 15 early in the year, and then returned to Chapters 12
and 13 later. Chapters 11 (Organization) and 14 (Drafting the Analysis) can be taught together
in a longer class. Chapter 15 (Revising and Editing) can be used very effectively at any point
during the course, by asking students to edit a piece of work in class, either individually or as a
group activity. Students can work on a sample that you provide, or you may ask them to
perform a peer- or a self-edit related to the writing project on which they are working. You can
also return to these chapters when students are working on other projects later in the year.
We wrote this chapter in response to recurring problems in student papers. It is the first
in Part C because it provides a structure within which the lessons in the other chapters fit.
Because good organization is fundamental, many teachers find this chapter and its exercises
worth emphasizing.

Suggested Approach to Exercises

Exercise 11-A

The factual and legal materials for this exercise are also used in Exercise 16-C.

Students find this problem interesting because it offers a choice of rules. The will could
be invalid under the Steffans test, the Kendall test, or both. It is also a good problem for getting
students to identify facts relevant to the elements in each test and to make arguments based on
these facts.

1. Students should have no difficulty identifying the two rules and their elements from the
cases. Because both rules involve undue influence, the best descriptive labels for the
rules may be based on the case names (e.g., Kendall rule).

90
Steffans Rule
ELEMENTS CHART

Element Facts of our case Elements met?


Susceptibility to undue Colonel was vice-president of Maybe
influence company. Before he wrote his second
will, company financed foreign trips for
him, allegedly to learn foreign land
development strategies.

Opportunity to influence Colonel went on frequent fishing trips Yes


with company’s general counsel.
General counsel drafted the second will.

Disposition to influence General counsel worked for the Maybe


company.

Coveted result Second will gave Colonel’s entire estate Maybe


to the company, but general counsel
may have wanted to benefit personally.

You may want to add a fourth column to this chart showing the relevant facts for each
element in the Steffans case. You might point out that although the court is far from
explicit in its holding on each element, you can infer the holding on specific elements
from the facts.

Kendall Rule
ELEMENTS CHART

Element Facts of our case Element met?


Confidential relationship Person who wrote second will was Maybe
between testator and one Colonel’s personal attorney, and was
alleged to have exercised general counsel for the company
under influence (that is, where Colonel was vice-president.
whether the confidant
controlled or influenced
drafting of the will)

Suspicious circumstances In his second will, Colonel gave his Yes


surrounding making of will entire estate to the company. General
counsel went on frequent fishing trips
with Colonel. Not clear how or
whether general counsel benefited
personally.

91
Again, you might want to add a fourth column showing the relevant facts from the Kendall
case.
a. Using this analysis, the issues and sub-issues break out like this:

(1) Whether the general counsel exercised undue influence (under the Steffans
rule).

a. Whether the Colonel was susceptible to undue influence.


b. Whether the general counsel had a disposition to influence.
c. Whether the general counsel obtained the coveted result.

(2) Whether there was a confidential relationship between the general


counsel and the Colonel that establishes undue influence (under the
Kendall test) (that is, whether the general counsel controlled or
influenced drafting of the will).

b. There is one given under each rule:

 The company’s general counsel went on fishing trips with the Colonel,
establishing the opportunity to exercise influence under the Steffans rule.

 The “suspicious circumstances” requirement under the Kendall rule is likely


met since the Colonel left his entire estate to the company. Even though the
Colonel was estranged from his son, the second will was drafted by the
company’s general counsel and was made only after the Colonel went on
several trips financed by the company.
2. a.

Kendall Rule
ANALYSIS CHART

Element Why Element is Met Why Element is Not Met


Confidential relationship Person who wrote second will was Colonel was not at all
between testator and one Colonel’s personal attorney, and senile. General counsel
alleged to have exercised was general counsel for the received no apparent
undue influence (that is, company where Colonel was vice- personal benefit from the
whether the confidant president. They were also close will, especially since he left
controlled or influenced personal friends, fishing together the company a year ago.
drafting of the will) several times each month.

92
Suspicious circumstances In second will, Colonel gave his Given.
surrounding making of will entire estate to the company, after
going on several international trips
financed by company. Disinherited
only surviving, albeit estranged,
child. General counsel frequently
went fishing with Colonel. Not
clear how or whether general
counsel benefited personally.

b. Since the facts can support a conclusion for either party, there is no “right”
answer to this question. But students should be prepared to explain why they
have concluded that a particular outcome is more likely.

c. Here is one approach to outlining the analysis:

Court will probably hold the will is valid under the Kendall test.

1. General counsel probably did not control or influence drafting of the


will. Drafter of second will was Colonel’s personal attorney as well as
general counsel for company. Unlike the testator in Kendall, Colonel
was active, not senile. Fact that general counsel drafted will that
benefited his company does not mean that the Colonel didn’t make this
decision.

2. However, there are some suspicious circumstances surrounding making


of the will. Analogy to Kendall: drafting attorney there was personal
attorney for mother of beneficiary who received entire estate rather than
the natural beneficiary – the testator’s husband, while here will drafted
by general counsel for company that received entire estate rather than
natural heir – testator’s son.
3. a.

Steffans Rule
ANALYSIS CHART
Element Why Element is Met Why Element is Not Met
Susceptibility to undue Colonel was vice-president of Colonel not at all senile.
influence company. Before he wrote his Nothing to indicate he
second will, company financed was unduly influenced by
foreign trips for him, allegedly to going on trips financed by
learn foreign land development company.
strategies.

93
Opportunity to influence Colonel went on frequent fishing Given
trips with company’s general
counsel. General counsel drafted
the second will.

Disposition to influence General counsel worked for the General counsel


company; may have been in his apparently received no
personal financial interest to personal benefit under the
have company receive large will and left the company
bequest. a year ago.
Coveted result Second will gave Colonel’s No personal benefit to
entire estate to the company, but general counsel apparent
general counsel may have and he has since left the
wanted to benefit personally. company.

1. Since the facts can support a conclusion for either party, there is no “right”
answer to this question. But students should be prepared to explain why they
have concluded a particular outcome is more likely.

2. Here is one approach to outlining the analysis:

The court would probably not hold the second will invalid under the Steffans
test.

a. Opportunity to influence: Attorney and Colonel had much contact. Steffans


is distinguishable because there was minimal contact with paperboy.

b. Disposition to influence: General counsel knew company would


benefit. Steffans is distinguishable because paperboy had no idea he
would benefit.

c. Coveted result: General counsel probably wanted company to benefit


because benefit to company would also benefit him as general counsel.
Steffans is analogous, even though benefit is not as direct.

d. Susceptible to undue influence: Colonel probably not susceptible. Like


the will in Steffans, his will left his estate to someone other than his
natural heir. But Colonel was not senile, was estranged from son who
would have benefitted from the first will, and appears to have been
devoted to the company.

Exercise 11-B

The factual and legal materials for this problem are also used in Exercises 16-D, 18-C,
and 18-G.

94
1. There are three relevant rules: the regulation, nuisance, and trespass. Some students
want to argue that the farmers should sue the Department of the Environment under
section 11 for not promulgating a more protective regulation, especially when they find
that the regulation does not apply. But the problem focuses on possible causes of action
against the company, not against the agency. As in the previous exercise, an elements
chart will help clarify the analysis under each rule.

Section 14 allows any person to bring an action in the appropriate trial court to enforce
the regulations. An elements chart for the regulation might look like this:

Regulation
ELEMENTS CHART
Element Facts of our case Element met?
Powerhouse with capacity of Powerhouse has capacity of 500,000 No, although
more than 500,000 pounds of pounds of steam per hour. students will want to
steam per hour. say “maybe.”
Burns fuel that exceeds 1.0% Burns coal containing 0.9% to 1.2% Yes, at least some of
sulfur content by weight. by weight. the time.

Many Students will look for reasons the first element should apply. Some may argue,
for example, that the powerhouse could not possibly have a capacity of exactly 500,000
pounds per hour, using technical reasons. Based on the facts in the problem, however,
the regulation is not applicable.

An elements chart for nuisance based on the Peters case might look like this:

Nuisance
ELEMENTS CHART
Element Facts of our case Element met?
Defendant landowner All-Rite owns land on which factory Yes
is located or a lease-hold estate in the
land.
Unreasonably Although there is interference, All- Maybe
Rite employs 490 people and provides
much income for the community. Its
coal has a lower sulfur content than
coal burned in other states.
Interferes with use and Sulfur dioxide produced by the Yes
enjoyment factory reduces value of farmers’
crops by 5%, reduces value of their
property by $5,000-$10,000, and
probably increases their respiratory
ailments.
Of the plaintiff’s land Farmers’ land is affected. Yes

95
Notice that the facts under the first element are inferred from the problem. This might
be a useful place to talk about when it is permissible to draw inferences from the factual
situation since any other inference is not very plausible.

As in Exercise 11-A, you might want to develop a fourth column for the facts of the
Peters case for comparison.

An elements chart for trespass based on the Neely and Jacobs cases might look like this:

Trespass
ELEMENTS CHART
Element Facts of our case Element met?
Physical invasion - some Sulfur dioxide particles from the Maybe
tangible object air settle on fields, whitening
leaves of farmer’s plants.
by the defendant All-Rite emits sulfur dioxide. Yes
on the plaintiff’s land Sulfur dioxide settles on farmers’ land. Yes
Unless privileged by There are no facts to show that it is Maybe
necessity economically unfeasible for All-Rite to
further reduce emissions.

Because there are two relevant cases, it may be worthwhile to prepare case briefing
charts for the two sub-issues in this problem.

a. The problem thus suggests the following issues and sub-issues: Under the
nuisance rule:
b.
Whether All-Rite’s interference with the use and enjoyment of the
farmers’ property is unreasonable and therefore constitutes a nuisance.

Under the trespass rule:

Whether All-Rite’s sulfur dioxide emissions trespass on the farmers’ land.

1. Whether the sulfur dioxide emissions are tangible and therefore


physically invade the farmers’ land.
2. Whether All-Rite’s emissions are privileged by necessity.

c. The following are givens under each rule:

Under the nuisance rule the first element is a given since All-Rite is the owner
of the factory that produces sulfur dioxide emissions. Further, the third and
fourth elements are given since the farmers are land owners whose use of their
land is being interfered with, as shown by the decrease in the value of their
crops and the increase in their respiratory ailments.

96
Under the trespass rule only the second and third elements are given since All-
Rite emits the sulfur dioxide and the white powder settles on the farmers’ land.

2. a.

Trespass
ANALYSIS CHART
Element Why element is met Why element is not met
Physical invasion - Sulfur dioxide emitted into the Particles of sulfur dioxide are
some tangible air settles on plants in the not the same type of tangible
object fields, turning them white. physical invasion required in
prior cases.
by the defendant All-Rite emits sulfur dioxide. Given
on the plaintiff’s land Sulfur dioxide settles on Given
farmers’ land.
Unless privileged There are no facts to show that it The power plant provides jobs
by necessity is economically unfeasible for and financial benefit for the
All- Rite to further reduce community, thereby potentially
emissions. establishing the necessity of its
continued operation.

Trespass
CASE BRIEFING CHART
Our Case Neely Jacobs
Facts Sulfur dioxide emissions Light from outdoor Defendant’s car stopped
from defendant’s power theater and car in plaintiff’s front yard
plant settle on plaintiffs’ headlights disturbed after leaving the road to
crops turning them white adjoining landowner’s avoid colliding with
and decreasing their enjoyment of his home. another vehicle.
value. Also negatively
affect plaintiffs’ health.
Holding No trespass. No trespass.
Reasoning Light is not a physical Although defendant’s car
And invasion by a tangible invaded plaintiff’s
Policy object. property, the invasion
was privileged by the
necessity of avoiding the
collision.

b. As stated earlier, it is more difficult for students to draw conclusions than to


make arguments for either side. This is a good place to reinforce the three
factors from Chapter 10 (Reaching a Conclusion).

c. Here is one approach to outlining the analysis. The “givens” are listed first.
They should be explained before addressing the elements in dispute.
97
All-Rite’s emissions are trespassing on the farmers’ land.

1. The invasion is by All-Rite, the defendant.

2. The emissions invade the farmers’ land.

3. The sulfur dioxide emissions constitute a physical invasion of the


farmer’s land because they settle on the farmers’ alfalfa fields. Neely is
distinguishable because it involved light, unlike a chemical compound
(albeit a gaseous one) that can settle on leaves.

[Students can argue this sub-issue either way and based on how they
characterize sulfur dioxide. The problem does not state whether sulfur
dioxide is a gas or a particle. Some students, of course, will provide
answers based on their knowledge of chemistry or environmental
pollution. You might use this circumstance to explain the importance of
further factual research in real-world legal situations.]

4. The invasion is probably not privileged by necessity. The car driver in


Jacobs was trying to save his own life. In this case, there is no claim that
All-Rite’s emissions are driven by economic necessity, much less the
necessity of saving lives.
3. a.

Nuisance
ANALYSIS CHART
Element Why element is met Why element is not met
Defendant landowner All-Rite owns land on which factory Given
is located or a lease-hold estate in the
land. Factory emits sulfur dioxide.
Unreasonably No facts show that defendant cannot All-Rite employs 490
reduce emissions to reduce the people and provides
impact on the farmers. Further, significant income for the
damage to the farmers outweighs community. Its coal has a
economic benefits to community, lower sulfur content than
coal burned in other
states.
Interferes with use and Sulfur dioxide residue reduces the Given
enjoyment value of the farmers’ crops by 5%,
reduces value of their property by
$5,000 to $10,000, and probably
increases their respiratory ailments.
Of the plaintiff’s land The farmers’ land is affected. Given

98
b. As stated earlier, it is more difficult for students to draw conclusions than to
make arguments for either side. This is a good place to reinforce the three
factors from Chapter 10 (Reaching a Conclusion).

c. Here is one approach to outlining the analysis. The “givens” are listed first.
They should be explained before addressing the elements in dispute.

All-Rite is causing a nuisance.

1. All-Rite, the defendant, is a landowner; it either owns the property in fee


simple or it owns a leasehold estate in the property.

2. The farmers who would be the plaintiffs are also landowners.

3. All-Rite has interfered with the use and enjoyment of the farmers’ land
because the powerhouse’s emissions have reduced the value of their
crops
by 5%, reduced the value of their property by $5,000 to $10,000, and
probably increased their respiratory ailments. This interference is
analogous to, but more serious than, the disturbances caused by the
saloon’s patrons in Peters.

4. All-Rite’s interference is unreasonable. Peters is distinguishable because


it involved only occasional loudness or rudeness by the saloon’s patrons.
All- Rite’s interference reduces crop and property values as well as
increasing the likelihood of respiratory ailments. In Peters, the court said
the defendant should do what she could, but suggested that her patrons
were not under her control. In this case, by contrast, All-Rite’s emissions
are entirely under its control. The facts do not suggest that All-Rite is
doing everything it could; it might, for example, increase the height of
its stack or use lower sulfur coal. All-Rite’s apparent compliance with
the regulation and the jobs and money it contributes to the local economy
do not mean it is doing everything it can.

Exercise 11-C

1. The main rule is subsection (a) of State Code § 492, which makes it unlawful for an
employer to discriminate based on age, among other things. Subsection (b) provides an
exception to the rule, the bona fide occupational qualification (BFOQ), where the
qualification is reasonably necessary to the operation of the employer’s business. It is
applicable only if subsection (a) has been satisfied. Because the relevance of the
exception depends on whether the rule applies, it is logical to discuss the rule first.

The court in Hawkins defined discrimination as follows:

(1) the complainant is a member of a protected class,


(2) the complainant is qualified for the position,

99
(3) the employer made an adverse decision concerning the complainant, and
(4) but for the complainant’s protected status, the adverse decision would not have
been made.

The court in Alders clarified the BFOQ exception:

(1) age is not a BFOQ if the person is capable of performing the work, and
(2) economic justifications do not satisfy the BFOQ exception.

General Discrimination Rule from Hawkins


ELEMENTS CHART
Element Facts of our case Element met?
Complainant is a member Rhee is over 18 and under 70 years of age Yes
of a protected class
Complainant is qualified Rhee worked for company for 17 years Maybe
for the position but only as an office manager; she asserts
she can and knows how to do the physical
labor required because she maintains her
own garden.
Employer made an Rhee was the only employee terminated. Yes
adverse decision
concerning complainant
But for the complainant’s Currier told Rhee she should retire and Maybe
protected status, adverse enjoy life. Also said she had no
decision would not have experience with physical labor involved
been made and he no longer needed her services as
office manager

Bona Fide Occupational Qualification Rule from Alders


ELEMENTS CHART
Element Facts of our case Element met?
Discrimination was Available jobs involved physical labor like Maybe.
based on a bona fide mowing, pruning, digging. Currier asserts
occupational Rhee had no experience with such work. Rhee
qualification asserts she was terminated because her wages
were higher than the younger employees who
had not been employed for as long as she had.
Reasonably necessary Currier runs a landscaping business. Physical Yes.
to the operation of the requirements and knowledge of landscaping
employer’s business. and lawn maintenance are necessary.

100
But Alders clarified the BFOQ rule by defining what is not a sufficient justification for
the exception. Students may make a separate chart for these elements, or add to the end
of the chart above:

Element Facts of our case Element met?


Age is not a BFOQ if Rhee is 59 years old but maintains Maybe. No facts indicate
the person is capable her own lawn and garden. Rhee Ferguson is not capable of
of performing the work asserts she is capable of doing doing the physical labor
physical labor and knows how because of her age but
based on personal experience, but new job demands will
her previous employment was as an probably be more
office manager. strenuous.

Economic justifications Currier retained younger employees Maybe. Although Currier


do not satisfy the BFOQ who had been with the company a did not assert financial
exception shorter period of time and whose reasons for retaining
wages were lower than Rhee’s. lower paid employees, the
facts could support that
conclusion.

. a. Elements (2) and (4) and the BFOQ exception as expanded upon in Alders,
are issues that could be outlined as follows:

1. Did Currier discriminate against Rhee because of her age?


a. Was Rhee qualified for the position?
b. Would Currier have retained her but for her age?

2. If so, does the bona fide occupational qualification exception apply?


a. Does the position require that a person be under the age of 59?
b. Did Currier refuse to retain Rhee for financial reasons?

b. Elements (1) and (3) in the Hawkins rule are given.

2. a.

General Discrimination Rule


ANALYSIS CHART
Element Why element is met Why element is not met
Complainant is a member Rhee is over 18 and under 70 Given
of a protected class years of age

101
Complainant is qualified Rhee worked for company for 17 Rhee worked for company for
for the position years and knows the operation. 17 years but only as an office
Asserts she can and knows how manager; no work experience
to do physical labor required in physical labor required
because she maintains her own
garden.
Employer made an Rhee was the only employee Given
adverse decision terminated.
concerning complainant
But for the complainant’s Currier told Rhee she should Rhee had no experience with
protected status, adverse retire and enjoy life. Kept on less physical labor involved and
decision would not have experience workers who earned Currier no longer needed her
been made lower wages. services as office manager

b. As stated earlier, it is more difficult for students to draw conclusions than to


make arguments for either side. This is a good place to reinforce the three
factors from Chapter 10 (Reaching a Conclusion).

c. Did Currier discriminate against Rhee because of her age? Students should be
prepared to explain how the law supports their conclusion. They should state the
two givens in the introduction and then focus on the two elements at issue.
Some suggested points to discuss:

Rhee’s position:

1. Rhee had worked for the company for seventeen years and is well
acquainted with the requirements of the job. She is also capable of doing
the manual work required in the new position because she maintained her
own private lawn and garden.

2. Currier terminated her employment because, in his words, she was


“getting up there” and hired several younger employees. This case is
similar to Hawkins, where the union provided employment opportunities to
other union members who were younger than the complainant.

Currier’s position:
1. Rhee is not capable of performing some of the tasks required for the new
position. That she was qualified for the position that will now be filled by
the owner of the business is irrelevant.

2. The main reason for Rhee’s termination is not her age but that her position
as office manager no longer exists because Currier will handle that job
himself. This case differs from the situation in Hawkins, where there were
jobs available for which the complainant was qualified.

102
3. a.

Bona Fide Occupational Qualification Rule


ANALYSIS CHART
Element Why element is met Why element is not met
Discrimination is Available jobs involved physical labor Rhee asserts she is capable of
based on a bona fide like mowing, pruning, digging, doing physical labor and
occupational Currier asserts Rhee had no knows how based on personal
qualification experience with such work. Rhee experience, but her previous
asserts she was terminated because employment was as an office
Currier told her she should retire and manager. Currier asserts she
her wages were higher than the has no experience in the type
younger employees who had not been of work required.
employed for as long as she had.
Reasonably Currier runs a landscaping business. Given
necessary to the Physical requirements and knowledge
operation of the of landscaping and lawn maintenance
employer’s business. are necessary.
Age is not a BFOQ Rhee is 59 years old but maintains her No facts indicate Rhee is not
if the person is own lawn and garden. capable of doing the physical
capable of labor because of her age.
performing the work
Economic Currier retained younger employees Although Currier did not
justifications do not who had been with the company a assert financial reasons for
satisfy the BFOQ shorter period of time and whose retaining lower paid
exception wages were lower than Rhee’s. employees, the facts could
support that conclusion.

b. As stated earlier, it is more difficult for students to draw conclusions than to


make arguments for either side. This is a good place to reinforce the three
factors from Chapter 10 (Reaching a Conclusion).

c. Does the bona fide occupational qualification apply? Students should be


prepared to explain how the law supports their conclusion. Students should
state the one
given at the outset and then organize the discussion around the remaining
elements. Since the Alders rule defined two factors that do not satisfy the BFOQ,
they should follow the discussion of the rule. Some suggested points to discuss:

Rhee’s position:

1. Persons fifty-nine years old and older are capable of the mowing, pruning,
spraying, and digging required in the new position.

2. Currier retained and hired new younger employees for lower wages. As
the court stated in Alders, the employer cannot rely on the BFOQ exception

103
when the discrimination is based on economic justifications.

Currier’s position:

1. Even if there was discrimination according to subsection (a), the BFOQ


exception is applicable because the new position requires someone
younger and stronger than Rhee.

2. Rhee was terminated for legitimate business reasons, not for financial
reasons. The decision to expand the business and move to a new location
required a new allocation of employment resources. Alders is
distinguishable because in that case the employer refused to hire anyone
over a certain age because of the impact on pension and disability plans –
a purely economic justification.

104
12
Describing the Law

Changes from the Sixth Edition

There are no major changes from the sixth edition.

Teaching Notes

Deciding how much information to include about a case or statute and whether to quote
or paraphrase are often difficult decisions for beginning law students. Some of us hand out an
outline, similar to the one below, that summarizes these and other topics discussed in this
chapter and offers additional examples drawn from the examples and exercises in the text or
from a recent assignment.

1. Be accurate.

Accuracy is essential in legal writing because without it you have no credibility. There
are four kinds of inaccuracy:

(a) Misstating the law or the facts. An obvious error is representing a statement that
is dicta as the holding of the court. Another is representing a rule that the court
rejected as the rule in the case. Careful reading of the cases and checking your
case briefs for congruency (Chapter 3) will eliminate these kinds of errors.

(b) Giving an incomplete rule of law, as illustrated by Answer B in the first example
in Part A and Answer C in the second example. In Answer B, the writer left out at
least one requirement of the remedy of specific performance. In Answer C, the
writer omitted important information regarding the notice requirement. It is
always better to quote rather than paraphrase the relevant portions of a rule.

(c) Lack of precision in stating the law, as also illustrated by Answer B in the first
example. The writer said “fair” consideration, but the rule laid down by the
court said “adequate.” These two terms may not have the same meaning or legal
significance. Avoid elegant variation in legal writing.

(d) Failing to show how various elements relate to each other, also illustrated by
Answer B. The element of inadequate remedy at law includes unascertainable
damages and unique items. They are not three separate elements.

Example: Assume that in your state, burglary is defined as “breaking and entering the
dwelling of another with the intent to commit a felony therein.” Consider this
description of the law:

One who breaks into the dwelling of another and has the intent to commit
a felony has committed burglary.

105
This description demonstrates three of the four kinds of inaccuracy.

 It is incomplete. Two elements, “entering” and “therein,” are missing.

 It is imprecise. The writer might have intended the term “breaks into” to include
both breaking and entering, but we cannot know for sure. This is an example of
altering the language of a rule unnecessarily.

 It does not show how the elements relate to each other. Two relationships are not
included: (1) One must have, at the time of the breaking and entering, an intent to
commit a felony. The intent element is not met if the actor forms the intent after
breaking and entering. (2) There must be an intent to commit a felony inside the
dwelling, not somewhere else.

2. Describe only the relevant law.

Whether you are using a statute or common law, include only the parts directly relevant
to the issue under discussion. For example, if the Lopez case on page 186 addressed all of the
requirements for specific performance, and you were discussing only the element of
inadequacy of a remedy at law, you would describe only the facts, holding, and reasoning
related to the inadequacy issue. Similarly, if you were analyzing the offer issue in Exercise 7-A
and describing Derek v. Beir, you would not include the statements in the majority and
dissenting opinions regarding consideration in that section of your paper.

3. Describe the law in enough detail.

There is no set rule regarding how much detail to include. A good rule of thumb is this:
The reader should not have to read the law (the case or statute) to follow your discussion and
understand your logic. to put it another way, your paper should be self-contained.
*Beginning legal writing students sometimes overlook this rule because the professor
gives them the law they are to use, so they assume the reader (the professor) is familiar with it;
therefore, they can just refer to it. Remind them that in real life, the reader may not have read the
cases or statutes and should be able to rely on the writer’s description.

4. Summarize the law whenever appropriate.

How much information—when you must include the facts, holding, reasoning of a
case— depends on what you are using the case for.

(a) A bare citation with no facts, holding, etc. is appropriate if you are citing the
case for an undisputed legal principle.

(b) A full description of key facts, holding, and reasoning is necessary if the case is
central to your analysis. Suppose you are analyzing the acceptance issue in
Exercise 7-A. Since you will use the Anselm facts, holding, and reasoning in
your analysis, you would include them in your description of the law.

106
5. Synthesize the law whenever necessary.

If several cases support a particular point, case synthesis results in a more coherent and
cohesive picture of the law than a series of case summaries. By showing what the cases have in
common, you can more easily compare them to or distinguish them from the facts of your case.
Your synthesis should begin with a thesis statement. In the example below, assume that you are
analyzing the falsity element of the libel issue in Chapter 8:

Minor or technical errors in a published statement are not enough to


render the statement false. According to the court in White v. Ball, the test is
whether “the gist, the sting, of the matter is false.” In White, the court held that a
customer’s statement accusing employees of stealing his watch was
“substantially accurate” even though the watch had been stolen by an
employee’s friend, who had been present at the work site. Similarly, in Willow
v. Orr, a statement by a woman that her husband had kidnapped their four-year-
old son, while technically inaccurate, was “true in substance.” The husband’s
act of taking the child without the wife’s consent and traveling with him to
another state was not kidnapping in the legal sense because the husband had not
violated any custody orders, but the essence of the charge was true. [Following
would be a discussion comparing these cases to Dooley’s case.]

When the facts of the cases do not require much explanation, you may use a simplified
synthesis consisting of a series of case citations with explanatory parentheticals. 8 Here is an
example (preceding is a discussion of Avery v. Goth, (1985), in which the court rejected the
common law rule that a landowner has no duty to prevent natural vegetation on his property
from obscuring the view of motorists on an adjoining street or highway):

Several courts have adopted the Avery rule. See Costa v. Lockhart (2002)
(foliage blocking view of street); Daily v. Compton (1995) (tree near shopping
mall exit); Graves v. Hoffman (1987) (weeds on shoulder of road).

Suggested Approach to Exercises

Exercise 12-A

The factual and legal materials for this exercise are also used in Exercises 7-A, 10-A,
13-A, and16-A.
We include thesis sentences here to provide a structure for the description of each of the
rules. You may want your students to do the same. After a thesis sentence or two stating that
the contract is or is not enforceable, the description of the law would follow.

8
The cases in this example were suggested by Sprecher v. Adamson, 636 P.2d 1121 (Cal. 1981); Whitt v.
Silverman, 788 So. 2d 210 (Fla. 2001); and Langden v. Rushton, 360 N.W.2d 270 (Mich. Ct. App. 1984).

107
Rule regarding “effective offer”:

Following are two examples of how to describe the law relevant to the “effective offer”
issue based on different conclusions. In the first, the conclusion is that the offer was effective.
In the second, the conclusion is that it was not. Notice that the only difference between the two
suggested answers is the way the writer introduces the law. The lesson for students is that
regardless of the conclusion, the writer must describe the law fairly and objectively. The
support for the conclusion will come when the writer applies the law to the facts of the case.

Example 1

McKay made an effective offer to sell his yacht to Green because Green
reasonably believed McKay intended to make a binding offer, even though
McKay may not have actually had that intent. Although an offer is effective in
this state if the offeror intended to make a binding offer, intent is defined not by
what the offeror actually intended, but by an objective standard: “[W]hether a
reasonable person in the offeree’s shoes would believe that the offeror intended
to make a binding contract.” Derek. In Derek, the offeror told the offeree, his
neighbor, that for five dollars he would build a wall between their lots so that he
would never have to look at the offeree’s “ridiculous” face again. The offeree
agreed and paid him five dollars, but the offeror never built the fence. The
offeree in that case knew about the offeror’s reputation for playing practical
jokes on his neighbors. Under those circumstances, the court held that there was
no binding offer because the offeree either knew or should have known that the
offeror was not serious.
[The next paragraph would distinguish Derek and show why Green’s
belief was reasonable.]

Example 2

McKay’s offer to sell his yacht to Green is not enforceable because the
offer was not effective. An effective offer is one in which the offeror intends to
make a binding contract. Derek. Intent is defined by an objective standard,
rather than by what the offeror actually intended: “[W]hether a reasonable
person in the offeree’s shoes would believe that the offeror intended to make a
binding contract.” Derek. In Derek, the offeror told the offeree, his neighbor,
that for five dollars he would build a wall between their lots so that he would
never have to look at the offeree’s “ridiculous” face again. The offeree agreed
and paid him five dollars, but the offeror never built the fence. The offeree in
that case knew about the offeror’s reputation for playing practical jokes on his
neighbors. Under those circumstances, the court held that there was no binding
offer because the offeree either knew or should have known that the offeror was
not serious.
[The next paragraph would analogize the McKay case to Derek and
show why Green should have known the offer was not serious even though he
did not know of McKay’s reputation as a practical joker.]

108
Rule regarding valid acceptance (law from Anselm):

The example below illustrates an introductory conclusion that the acceptance is not valid. Like
the examples above, students holding the contrary view would begin with a conclusion stating
their position, but the objective summary of the law would be the same.

Green’s acceptance of McKay’s offer is probably not valid. A contract is


not enforceable unless there is valid acceptance of the offer. Anselm. Although
the offeree does not have to ensure that the offer is read or understood, the
acceptance must not differ in any way from the offer. Anselm. In Anselm, the
offeree delivered a written message of acceptance to buy bales of cotton to the
seller’s office. Although the secretary placed the message on the defendant’s
desk, he did not see it for several weeks and sold the cotton to another buyer.
The court held that it would be unreasonable to require an offeree to ensure that
his written acceptance is read after it is delivered. However, the court held there
was no valid contract since the written acceptance changed the date of delivery,
noting that an acceptance that does not mirror the offer in every respect is a
counteroffer and does not constitute an acceptance of the original offer.
[The next paragraph would analogize the McKay case to Anselm and
show why the acceptance was not valid because Green’s acceptance specified
the addition of McKay’s golf clubs, thereby creating a counter-offer instead of a
valid acceptance.]

Exercise 12-B

The factual and legal materials for this exercises are also used in Exercises 7-B, 10-B,
13-B, and 16-B
As in Exercise 12-A, you may want students to include thesis sentences for the question
of whether the paper can obtain the information by filing a lawsuit in order to provide a
structure for the discussion.

Rule regarding the acquisition of the information by request or by legal action:

Sections 3 and 4 as well as the definitions of the statute need to be summarized to


establish the method by which the information may be requested and by which a suit may
subsequently be filed.

Rule regarding the law enforcement exception to the statute:

Section 5(a) should be quoted and the Wheeler case explained.

Rule regarding the privacy exception to the statute:

Section 5(d) should be quoted and the Holcombe case explained.

109
Exercise 12-C

The factual and legal materials for this exercise are also used in Exercises 7-C, 10-C,
13-C, and18-B.
As in the previous exercises, you may want students to include thesis sentences for
overall question of whether the statute is enforceable in order to provide a structure for the
discussion.

Rule regarding Statute of Frauds:

Students should quote State Rev. Laws § 60.1(4) to establish the requirements under the
Statute of Frauds and then use Treacher to set out the 3 elements a valid contract must contain
under the statute. The students should then summarize Treacher to explain the policy
underlying the statute and why the court concluded that contract was not valid.

Rule regarding misrepresentation:

Students should use Divine to set out the 6 elements of misrepresentation and define the
elements of “material fact” and “justifiable reliance.” Students should then explain the Divine
case to illustrate the application of these elements.

Exercise 12-D

The factual and legal materials for this exercise are also used in Exercises 13-D and 17-
C.

Students should first give the common law rule and then explain the exceptions in the
context of the case law. Example:

Philip and Julia Langford have no property interest in the dogwood trees they
planted in their backyard because the trees were part of the real estate they sold
to Wilbur and Marcie Cliff. The common law rule is that trees growing in the
soil are part of the land. Dinesen v. Stafford Nat’l Bank. There are two
exceptions to this rule.

Rule regarding personal property:

The first exception is that crops are considered personal property if the owner so
intended. Id. In Dinesen, the court held that cranberry bushes were not crops
because the owner intended to harvest only the berries, not the bushes
themselves. Therefore, the bushes were part of the realty.

[The next paragraph would analogize Dinesen and explain why the trees would
probably not be considered crops.]

110
Rule regarding “stock in trade”:

The second exception is that trees or plants that are stock in trade are the
merchant’s personal property. Updike v. Teague. Stock in trade is defined as
“goods or chattel that a merchant obtains for the purpose of resale.” Id. In
Updike, the court held that young trees in a nursery were personal property,
characterizing them as goods offered for sale that a merchant must temporarily
store in the ground in order to keep them alive.

[The next paragraph would distinguish Updike on the basis that the trees were
on private property and were not offered for sale at the time of the real estate
transaction.]

111
13
Applying the Law

Changes from the Sixth Edition

There are no major changes from the sixth edition.

Teaching Notes

Students often find it difficult to fully and completely explain how the relevant law applies
to the specific facts of a new problem. It is not uncommon to see fairly good descriptions of the
law followed by reasoning that is brief, superficial, and conclusory. Some professors use non-legal
examples to help their students learn how to draw specific analogies and distinctions. One
professor always begins with the classic comparison between apples and oranges, setting one of
each on the desk at the beginning of class. Students very quickly discover that apples and oranges
have at least as many similarities as they have differences.
Encourage your students to compare and contrast all the relevant aspects of each case they
read with the other cases and with the problem they are trying to resolve. You can create lists of
similarities and differences on the board, or have students develop detailed case briefing charts
and analysis charts, as shown in Chapters 8 (Common Law Analysis) and 9 (Statutory Analysis).
Then, when students begin to explain their analysis of a legal question, they will have the specific
information necessary to draw strong analogies and distinctions and explain the basis for their
position on an issue.
Remind students that they must both make the point of comparison explicit, and explain
its significance. The phrase “Like Smith . . .” is unclear because the writer has not identified the
point of comparison between Smith and the present case. Statements such as “Like the defendant
in Smith . . .” or “Like the search conducted in Smith . . .” identify the specific point of
comparison and logically lead to the explanation of how the comparison supports the writer’s
analysis that follows. The significance of the comparison must be explained to show how it
supports the writer’s conclusion. For example, the statement: “Like the search conducted in
Smith, the search of Defendant’s apartment was conducted before the warrant was issued.” points
out a specific comparison, but it leaves the reader hanging as to why it matters. The last step of
legal reasoning requires an explanation as to why the comparison is significant: “Thus, the
evidence is probably inadmissible as the product of an unconstitutional search.”

Suggested Approach to Exercises Exercise 13-A

The factual and legal materials used for this exercises are also used in Exercises 7-A, 10-
A, 12-A, and16-A. This exercise works best as a follow-up to Exercise 12-A, since these answers
continue the discussion begun in Exercise 12-A. The first answer concludes that there was an
effective offer; the second concludes that there was not.

Example 1

McKay made an effective offer to sell his yacht to Green because Green
reasonably believed McKay intended to make a binding offer, even though

112
McKay may not have actually had that intent. [Here a description of the law in
Derek as suggested in the answer to Exercise 12-A.]
Unlike the offeree in Derek, Green did not know of McKay’s reputation as
a practical joker and so had no reason to doubt that the offer was genuine. His
response at the end of the conversation that he needed time to think about the offer
and raise the money shows that he took the offer seriously. Unlike the situation in
Derek, Green’s attitude was reasonable because McKay gave him no reason to
doubt the sincerity of the offer. Green expressed incredulity at the beginning of
the conversation when he said “You can’t be serious” after McKay offered to sell
his yacht at one-tenth of its value. But McKay continued the conversation after
that and never intimated that the offer was a joke. Green’s belief that McKay
intended to make a binding offer was therefore reasonable under the
circumstances.

Example 2

McKay’s offer to sell his yacht to Green is not enforceable because the
offer was not effective. [Here a description of the law in Derek as suggested in the
answer to Exercise 12-A.]
In Derek, the offeree sought to enforce a contract based on a practical
joke. The court rejected that attempt, holding that the offeree knew or should have
known that the offer was made in jest. This case is similar. As a joke, McKay
offered to sell his $100,000 yacht to Green for $10,000. Even though Green did
not know that McKay had a reputation as a practical joker, he should have known
that the offer was a joke. He knew the true value of the boat and exclaimed “You
can’t be serious” when McKay made the offer. The court in Derek refused to
enforce a contract when the offeror promised to build a wall between their
adjoining lots for five dollars. In McKay’s case, the difference in value between
the subject of the offer and the price is also greatly disproportionate. Green may
have actually believed that McKay intended to make a binding offer; because he
requested time to consider the offer and raise the money. If he had that belief,
however, it was not reasonable under the circumstances.

Rule regarding valid acceptance (law from Anselm):

Green’s acceptance of McKay’s offer is probably not valid. [Here a


description of the law in Anselm as suggested in the answer to Exercise 12-A.]
Like the acceptance in Anselm, that was sufficiently communicated even
though it was not seen by the offeror for several weeks, Green’s acceptance was
sufficiently communicated to McKay even though he had passed out and did not
hear it. However, like the acceptance in Anselm that changed the terms of the
agreement by changing the delivery date, Green changed the terms by asking for
McKay’s golf clubs to be included. Since Green’s acceptance did not mirror
McKay’s offer in every respect, it was a counteroffer and not an acceptance of the
original offer.

113
Exercise 13-B

The factual and legal materials used for this exercises are also used in Exercises 7-B, 10-B,
12-B, and 16-B. This exercise works best as a follow-up to Exercise 12-B.

Rule under which the information is requested or sought by court action:

Students should first explain all of the “givens,” one by one, as established under section
2. This explanation leads to the discussion showing that the newspaper complied with section 3
and that an action to compel disclosure under section 4 is appropriate.

Rule regarding the law enforcement exception:

Students should explain why the text of section 5(a) does or does not apply to this case.
Students should then discuss how the Wheeler Publishing case is analogous to or different from
the facts of this case by comparing the information Mickel believes to be in the police files with
the information the court contrasted in the “Offense Report” and “Supplementary Offense Report”
in the Wheeler Publishing case.

Rule regarding the privacy exception:

Students need to explain why the text of section 5(d) does or does not apply to the Banner-
Patriot’s request in light of the Holcombe case. Students should discuss how the information
regarding the mayor’s wife is similar to or different from the information disclosed in Holcombe
and the court’s likely decision based on the reasoning regarding information that is already public.

Exercise 13-C

The legal and factual materials used for this exercise are also used in Exercises 7-C, 10-
C, 12-C, and18-B. This exercise works best as a follow-up to Exercise 12-C.

Although students will likely conclude that the Statute of Frauds was not met, they might
reach different conclusions as to whether the contract is enforceable under each of the two rules
presented.

Rule regarding Statute of Frauds:

Students will likely conclude that the Statute of Frauds as set out in State Rev. Laws §
60.1(4) and defined by the 3 requirements set out in Treacher has not been satisfied. Although the
napkin was imprinted with the name of the seller, “Blue Goose Inn,” It did not specify both parties
to the contract. Further, the contract was not signed by Hughes, the party “to be charged therewith,”
even though Hughes’ acceptance was recorded. The students should consider the above elements
in light of the policies recognized in Treacher.

114
Rule regarding misrepresentation:

Students may reach differing conclusions as to the outcome under this rule. Students should
consider each of the 6 elements set out in Divine and explain that elements 1, 2, 4, and 6 are givens:
(1) the value of the Blue Goose is a material fact to a purchaser; (2) Zoeller told Hughes it was
worth $400,000 when in reality the books showed it was only worth $280,000; (4) Zoeller
intended Hughes to rely on the information in deciding whether to buy the Inn; and (6) Hughes
will suffer damages if she has to buy the Inn for $320,000 when it is only worth $280,000. Thus,
students should analogize and distinguish the facts of Divine to analyze the remaining elements:
(3) whether Zoeller knew the figures were false or really believed they were the result of an error
and (5) whether Hughes justifiably relied on the information in light of her actual knowledge of
the books. The latter element should be considered in light of the court’s definition of “justifiable
reliance.”

Exercise 13-D

The factual and legal materials used for this exercise are also used in Exercises 12-D and
17-C. This exercise works best as a follow-up to Exercise 12-D.

Students might reach different conclusions when they analyze the issue under the crop
theory or the stock-in-trade theory. They might decide, for example, that the trees are not crops
but they are stock in trade. Even if they reach the conclusion that the trees are neither, and therefore
no longer the property of the Langfords, it is important to explain the law and its application under
both theories.
As to the first rule, whether the trees were crops, the likely conclusion is that they were
not. The court in Dinesen pointed out that the cranberry bushes were not crops because only the
berries, not the bushes, were harvested annually. Trees are also not harvested annually. Students
might disagree about whether this issue a “given.” Clearly, trees are not “crops” in the ordinary
sense of the word. However, the trees in this case were intended to be harvested, in the sense that
they were to be dug up and sold to customers. Students could therefore make a legitimate argument
that Dinesen is distinguishable and the trees are similar to plants traditionally considered crops.
The second rule presents a closer question. The facts in this case are virtually identical to
the facts in Updike—trees intended to be sold were temporarily stored by being planted in the
ground. A reasonable conclusion is that, like the trees in Updike, they were stock in trade. The
important difference is that the Langfords’ trees were in the backyard of a private residence, not
in a commercial nursery. Students should explain the relevance of this fact.
In analyzing the facts under each rule, students should recognize the importance that the
courts placed on the owner’s intent. The court in Dinesen stated that the owner must intend that
plants growing in the soil are crops to be harvested. The court in Updike stated that stock in trade
is property that the merchant obtains for the purpose of resale. If other considerations are evenly
balanced, intent is often the deciding factor.

115
14
Drafting the Analysis

Changes from the Sixth Edition

This was Chapter 15 in the Sixth Edition. Although this chapter still includes the step of
including transitions to lead the reader from one point to another, much of the chapter on
Signposting that was Chapter 14 in the Sixth Edition has been integrated into Chapter 16
(Revising and Editing). There are no significant changes in the text or exercises of this chapter
from the Sixth Edition.

Teaching Notes

This chapter synthesizes the most important lessons of Parts B and C by showing students,
step by step, how to put together a Discussion. For that reason, we try to teach this chapter before
the first formal memo is due, even if we have to skip other chapters to do so. We return to those
chapters later.
As indicated in the text, legal writing professors may use slightly different approaches:
CRAC, IRAC, etc., but the process of explaining the law before applying it to the facts of the
problem is the same. Teachers should emphasize this does not advocate a “fill in the blanks”
approach to legal analysis, but rather provides an easy way for the students to see the overall
structure and remember each step during the drafting process.
You might ask your students what changes they would make to the two discussions (the
libel issue and the intent-to-publish sub-issue on pages 234-235 and the falsity sub-issue under
Answer A on page 236) if they combined them. The question is worth discussing in class
because it addresses the problem of overall organization and provides a nice transition to the
exercises.
To combine the two discussions, at least two changes are necessary. First, the thesis
paragraph for the libel issue needs to include a sentence or two concerning the falsity of the fliers.
The revised paragraph is shown below, with the new material:

Dooley probably did not commit libel. Libel is the “intentional publication of
false statements about a person that humiliate that person or subject him to the
loss of social prestige.” White v. Ball. Two elements of this rule have been
satisfied. The fliers were published when they were released by accident. The
fliers also subjected Fremont to a loss of social prestige because they accused him
of having a criminal record. Two elements require more discussion—the falsity of
the flier and Dooley’s intent. The fliers are false because Fremont has no criminal
record. The intent element has likely not been met, however, because Dooley did
not intend his fliers to be read by others.

Second, the discussion of the falsity sub-issue should go immediately after this paragraph,
followed by the discussion of the intent sub-issue. Some teachers believe that the sub-issues that
support your conclusion on an issue ought to be located before the sub-issues that support an
opposing position. Under that approach, the intent sub-issue should be discussed before the falsity
issue. Although this approach is acceptable, we believe that a good thesis paragraph for a

116
memorandum generally provides sufficient guidance for the reader. As Chapter 21 (Structure of
an Argument) states, however, briefs must be written differently.

Suggested Approach to Exercises

Exercise 14-A

The factual and legal materials used for this exercise are also used in Exercise 19-A.

This exercise gives students an opportunity to see the world from the teacher’s point of
view. That is, it asks them to read three discussions of the same issue, decide which is most
effective, and explain why. In our experience, students like this type of exercise because it puts
them in the position of praising and criticizing, rather than being praised and criticized. from a
pedagogical point of view, the exercise reinforces the lessons of Chapter 13 by having students
apply those lessons to someone else’s writing.
One approach is to divide the class into groups of 3 or 4 and have each group grade one
of the answers, using the grade criteria sheet in Appendix II.B of this manual. Then ask each
panel to point out the strengths and weaknesses according to the criteria in this chapter. This
approach gives the students valuable insight into how we assess the quality of their papers.
Another approach is to ask the class to evaluate the three answers according to the eight
steps explained in the chapter. Before getting into details, you might ask students for their overall
impressions—which they like better and why. Following is a commentary on the three answers.
Answer C is the best. The thesis paragraph explains the writer’s conclusion on the overall
issue in terms of the three elements and the facts of this case. It then states the rule in a way that
makes the three elements clear.
The second paragraph begins with a topic sentence explaining the writer’s conclusion on
the first element (propensity toward violence), explains the law by synthesizing the two cases, and
then explains how the law applies. It also answers a contrary viewpoint (violent hockey playing
does not necessarily mean violence off the rink) by explaining that Deemer’s behavior is violent
even by hockey’s standards.
The third and fourth paragraphs analyze the knowledge issue in two parts—actual
knowledge (third paragraph) and constructive knowledge (fourth paragraph). The second element,
of course, requires that the employer knew or should have known. The third paragraph also
synthesizes the holdings of the two cases.
The next paragraph addresses another contrary viewpoint (this case is different from the
other two because Deemer had no criminal record for the employer to be aware of) by focusing
the reader’s attention on the rule itself (knowledge of propensity toward violence).
The last two paragraphs address the third element—the employer had the employee work
in a place open to the public. The writer uses the analogous case and distinguishes Hersh. The
discussion closes with a conclusion on the element and the overall issue. For this sub-issue, like
the other two, a teacher can show how the writer addressed steps 2 through 7.
Answer A is the worst of the three. The thesis sentence does not incorporate the overall
rule or the reasons for the writer’s conclusion. The second sentence describes the Tyus case without
first providing a topic sentence identifying the relevant sub-issue or the writer’s conclusion.
The first five paragraphs contain a simultaneous discussion of two sub- issues—propensity
toward violence and knowledge. You might use that to reinforce an important point from Chapters
7 (Identifying and Selecting Issues) and 11 (Organization): If students do not clearly separate

117
elements, their discussion of sub-issues is likely to be confused.
The first and fourth paragraphs contain separate descriptions of the two cases. On both sub-
issues, the writer could have synthesized the holdings and facts. Note the repetition this creates in
the second and fifth paragraphs.
The discussion of the public place sub-issue in the last two paragraphs is the best part of
Answer A, although it is not without problems.
Answer A contains other significant organizational problems. The fourth paragraph lacks
a topic sentence, for example. The counterargument in the third paragraph comes before the
writer has discussed the reasons for his conclusion on that point.
Answer A also contains several instances of vague editorializing that are not compatible
with a precise analysis of the problem. The topic sentence of the last paragraph, for example, says
the employer in the Hersh case “did the right thing.” The next sentence explains what that means,
but Answer A would be better if it did not include that phrase and continued directly into the next
sentence. The comments about the Bolshoi ballet and “dangerous and frightening people like
Deemer” add little to the analysis.
Finally, the conclusion is wrong. The issue is whether Watson is liable for assault, not
whether he is liable for hiring Deemer. Also, if you want to explicitly anticipate Chapter 16
(Objective Writing), you might point out that Answer A says Watson “is liable,” a more certain
conclusion than is warranted here. Answer C, by contrast, says Watson “is most likely
responsible,” a conclusion that is more objective than that in Answer A.
Answer B is better but still contains major problems. The thesis paragraph states a
conclusion on each of the three elements and the facts of this case, and then describes the law as
if it contained only two elements.
The second paragraph starts with a topic sentence on the “propensity” sub-issue but then
describes the law on knowledge and applies that law to the facts of this case.
The third paragraph responds to a potential contrary position on the “propensity” sub-
issue, which is confusing in this context. The topic sentence, which begins “Watson may argue
that,” is wordy and focuses attention on what Watson may argue rather than the position itself.
The last two paragraphs address the public place sub-issue. The first few sentences of the
last paragraph are wordy, however, and that paragraph repeats significant parts of the prior
paragraph. The whole point of discussing your position first, of course, is to make the conclusion
so clear that it does not need to be repeated.

Exercise 14-B

The factual and legal materials used for this exercise are also used in Exercises 8-B, 17-A,
19-B, and 23-C.

Unlike most of these exercises, which involve several sub-issues, this problem involves
only one issue—whether Dr. Farmer’s negligence proximately caused Andrew Quale’s death.
For this reason, it may be a good introductory exercise for writing a discussion. This exercise can
easily be resolved either way; some teachers may see it as a good writing exercise for that
reason. The suggested approach below is based on a conclusion that Dr. Farmer is not liable.

 Step 1 requires the student to state his or her overall conclusion about Dr. Farmer’s
liability and a brief explanation of the legal rules, the “Ci” and “R” of the organizational

118
structure. Dr. Farmer is liable if (1) he was negligent and (2) his negligence proximately
caused Andrew Quale’s death. Although the problem asks students to assume Dr.
Farmer’s negligence and thus establishes that it is a “given,” students should explain
the facts supporting this element - that Dr. Farmer did not examine the back of
Andrew’s head and did not use other standard diagnostic procedures. This establishes
the framework to address the issue of causation.

 Step 2, which requires a topic sentence for this issue, is satisfied by a statement that Dr.
Farmer’s negligence did or did not proximately cause the boy’s death, the “Ci”
statement for this issue. Students are likely to be divided as to the probable outcome.
Note that students must follow Steps 2 through 7 for this issue even though those steps
are written in terms of sub-issues.

 Step 3 requires a description of the law, the “R” for this issue. Answers here may vary a
little, because the cases talk about both “a substantial possibility” and “more likely than
not.” The Mallard court treats them as interchangeable, and that is probably the best
explanation. Students who use this explanation are more likely to conclude that Dr.
Farmer is not liable, but others might rely on the expert’s opinion that Andrew might
have had a slightly greater than 50/50 chance of survival to reach the opposite
conclusion. Students should also include the Mallard court’s policy rationale that doctors
should not be held liable for harm they “did not cause and may not have been able to
prevent.”

 Step 4 requires an explanation of how the law applies to the facts, the “A” of the analysis.
Students who conclude that Dr. Farmer is not liable will most likely use Mallard. Like
the decedent in Mallard, who had a possibility but not a probability of recovery with
prompt surgery, Andrew Quale had an even chance, but not a probability, of recovery if
Dr. Farmer had followed the proper procedures. Like the doctor in Mallard, Dr. Farmer
should not be held liable for harm he did not cause and which he probably could not have
prevented.

 Step 5 requires a statement of the support for contrary conclusion and its resolution. As
noted above, students who conclude that Farmer is liable will argue that a better than
50/50 chance of survival is “substantial” under Moulton and consistent with the “more
likely than not” language in Mallard. Step 6 requires a description of the law on which
the counterargument would be based. Unless students have already explained Moulton,
they would do so here. This explanation would include the facts, the holding, and the
policy that certainty is not required. The counterargument would also be based on a
strict reading of the “more likely than not” language in Mallard.

 Step 7 requires an explanation. Moulton is distinguishable because the decedent in that


case would have survived with surgery. Thus, even though the case talks about a
substantial possibility, the facts involved a substantial certainty. Although the law does
not require certainty in causation, it does require that the doctor’s negligence more than
likely caused the boy’s death. Because that is not what happened here, Dr. Farmer is
probably not liable.

119
 Step 8 requires the final conclusion, the “C” of the analysis, and signposting. Here, since
students must evaluate their own writing in light of Chapter 13, the answers will vary.
This exercise also works well for peer review and for signposting.

Exercise 14-C

The factual and legal materials used for this exercise are also used in Exercises 8-C, 17-
B, and 23-E. This problem is exhaustively analyzed in this manual under Exercise 8-C, and there
is an annotated sample discussion in Appendix I.B of this manual. The explanation here will
therefore be in outline form.

 The Step 1 thesis, or “Ci”, must state the student’s conclusion on the overall issue of
private nuisance and provide a brief explanation. It must describe the elements of the
private nuisance rule, the “R”, and explain the two “givens” (Greenleaf’s injury and
Elliot’s interference), and identify the two sub-issues (legal right to free flow of light
and unreasonableness of interference).

 Steps 2 through 7 should be repeated for each of these sub-issues. Using the CiRAC
acronym may help the students remember each step as they draft their discussion of
each sub-issue. Students will often (though not always) find the interference to be
unreasonable. That satisfies the Step 2 requirement for a statement of the student’s
conclusion on each sub-issue. The Step 3 description of the law will include the facts
and holdings of the Blum and Horton cases. In Step 4, students will show how the facts
of those cases are analogous to the facts of Greenleaf’s case. The Step 5 contrary
conclusion might be based on a claim that the plaintiffs in the two cases were
damaged more than Greenleaf. Students should state this contrary position and deny its
validity in one or two sentences. (Note: There may be other contrary arguments.) The
legal basis for this contrary view under Step 6 might require some additional
explanation of the facts of the cases, particularly concerning the relative amount of
harm the plaintiffs suffered. In Step 7, students would explain that Greenleaf has been
harmed in the same fundamental way as the melon grower in Horton and the farmer in
Blum. Because Greenleaf’s firm specializes in solar design, interference with the solar
panels is equally fundamental and unreasonable to him.

 For the sub-issue on the legal right to sunlight, the Step 2 statement of the student’s
conclusion should be that there is no legal right to the free flow of sunlight. Students
often want to take issue with this conclusion, and some student papers will be written
from this perspective. The Step 3 description of the law might include the legal rule
from Shover and the facts, holding, and rationale in Cassells. The Step 4 explanation
might show how Cassells is analogous to Greenleaf’s case and would also explain that
there is no case in this state holding that there is a legal right to the free flow of sunlight.
Several contrary conclusions are possible for Step 5. One is that Shover is broad
enough to include the free flow of light. Students who use this contrary approach
would include a sentence or two making this statement and denying its
persuasiveness. Under Step 6, students might explain the facts and holding in Shover.

120
Under Step 7, students would explain that the free flow of light and lateral support are
different, and reiterate the lack of any judicial recognition of a free flow of light.

 Students should then signpost the entire discussion under Step 8.

Exercise 14-D

The factual and legal materials used in this exercise are also used in Exercise 9-A. This is
a good initial writing exercise because the analysis for one of the two sub-issues is already set out
in Chapter 9 (Statutory Analysis). The only missing part of the analysis for the governmental
functions sub-issue is a conclusion. If students have done Exercise 9-A, then this exercise mostly
requires that students write what they have already analyzed.
The framework for this analysis should look something like the following. As noted earlier,
although we reach certain conclusions about these sub-issues, you might just as easily reach other
conclusions.

 Step 1 (thesis on overall issue) (Ci) needs to include the student’s overall conclusion
about whether the four Conservative members are a public body and a summary of the
rule defining the legal requirements (R). On balance, we think a court would conclude
that they are. The students should include an explanation of the “givens” (set out in the
chart on page 135-136 of the text), and should also explain that “public body” is used
both in the definition of “meeting” and in section 3. Finally, the students should
identify the remaining sub-issues—whether the four members are a local legislative
body and whether they are empowered to exercise governmental functions. This step
should, in short, provide a framework for the discussion that follows.

 Steps 2 through 7 must then be repeated for each sub-issue. If students have already
done Exercise 9-A, they need only organize and write what they have already analyzed
for the local legislative body sub-issue. Step 2 requires a statement of the writer’s
conclusion (Ci). It is likely that a court would decide that the four Conservative
members constitute a local legislative body. Step 3 requires a statement of the law (R)
—in this case, the relevant statutory language. Here it is essential to show the entire
relevant text from section 2(b): “any state or local legislative body, including a board,
commission, committee, subcommittee, authority, or council.” Step 4 requires an
explanation of the writer’s conclusion as to how the law applies to the facts (A). That
the four members constitute, as a practical matter, a working budget committee is
perhaps the best explanation. Students may also want to argue that the Conservatives
are a council because they comprise a working majority of the Council.

 Step 5 requires a statement of the contrary conclusion and its resolution. In this case,
it is likely irrelevant that the four members do not constitute a formal committee or
subcommittee. Under Step 6, students need not repeat the statute but should describe
the McPhee case. Under Step 7, although the McPhee case applied to the entire board
of education, its rationale that preliminary steps in the decision-making process should
be public probably applies with equal force to meetings of committees or
subcommittees. The statute does not require that the committee or subcommittee be

121
“formal,” in the sense of being appointed by the Council or in any other sense. Step 8
depends on what students write, but they should create at least three paragraphs, one
for Step 1, one for the conclusion (Steps 2 through 4) on this sub-issue, and another for
Steps 5 through 7 (the contrary conclusion). The structure is unchanged even if
students reach the opposite conclusion.

 For the governmental functions sub-issue, Step 2 requires a statement of the conclusion
(Ci). It is likely the court would conclude that the four Conservative members are
empowered to exercise governmental functions because they constitute a majority of
the Council. Step 3 requires a description (preferably a quotation) of the relevant
statutory language (R). Step 4 requires an explanation of how the law applies to the
facts (A). In this case, that a majority can vote whatever budget it wants when the
Council eventually meets, coupled with the McPhee court’s explanation of the
importance of subjecting preliminary meetings to public review, provides that
explanation. Step 5 requires a statement of the contrary conclusion. In this case, it is
likely irrelevant that the four members do not constitute a quorum. Students may skip
Step 6 because there is no additional law to describe. Under Step 7, students will
likely say that the four members can adopt whatever budget they want when the full
Council meets, as it eventually must. Step 8 would depend on the way the discussion is
written, but students should create separate paragraphs for the discussion and the
response to the contrary conclusion. At the conclusion of each sub-issue and at the
end of the overall discussion, students should provide a summary conclusion (C) to
either finish one sub-issue and transition to the next or to summarize the answer to the
overall issue for the reader.

Exercise 14-E

The factual and legal materials for this exercise are also used in Exercises 9-D and 23-D.

Begin with a conclusion (Ci) stated in terms of the issue. For example, if the conclusion is
that the League can probably intervene, the first sentence should say so and briefly say why. Then
follow with a summary of the law (R). Students should quote the statute. If students have concluded
that any of the requirements are “givens,” they should explain those first. They may want to explain
how a court is likely to resolve each sub-issue.
Steps 2 through 7 (the A of the analysis) should be repeated for each sub-issue. Thus, each
sub-issue should begin with a conclusion (Ci), which should be stated in terms of the law
applicable to that sub-issue (R), before the law is applied to the facts relevant to that sub-issue.
These steps suggest answers to one of these sub-issues—timeliness. Thus:

The League’s application would be “timely” as required by the statute.

Following the conclusion on a sub-issue, describe the law. Here, the law will be the
applicable part of the statute, which was already quoted in the thesis, followed by a discussion of
that requirement’s application in Halsey. For example:

In Halsey, the court found that the plaintiffs’ application was timely when it

122
was filed only three weeks after the suit was filed. The court concluded
there was no “unreasonable delay” in filing the application.

Note that this explanation includes the facts, the holding, and the court’s reasoning.

Next, specifically explain how the law applies to the facts (A). Show how the facts are like
or unlike those of Halsey. The student’s analysis should be complete and explicit. For example:

Similarly, here, the League would file its application within a week or two
after the city announced that it no longer represented the League’s interest
in this action. As in Halsey, there would be no unreasonable delay.

Although there is no explicit policy applicable in this example, be sure students include any
policies or reasoning that explain how or why they think the law will be applied in a certain way.
After explaining how the law supports their position, students should turn to the contrary
conclusion:

That the League’s application would be filed more than six months after
the original suit does not make it untimely.

Describe the law on which this conclusion would be based if it has not already been described. In
the example used above, the contrary conclusion would be based on Halsey, which has already
been explained.
Finally, explain why the contrary conclusion is unpersuasive. Reasoning and policy may
also play a role in this step:

The League believed the city would be representing its interests in the
litigation until last week, when the city attorney said the city might be
wrong. Because of this belief, the League did not intervene earlier. The
timing of the League’s application is therefore not unreasonable.

Students should follow Steps 2 through 7 for each of the three elements of the statute, as
noted above and follow each with the final, summary conclusion (C). They should then pull
together the various steps using the signposting materials in this chapter and in Chapter 15.

123
15
Revising and Editing

Changes from the Sixth Edition

This chapter integrated Chapters 14 and 16 from the Sixth Edition, and reorganized the
material around the two steps in the writing process: revision and editing. Part A of the
chapter, “Revision.” addresses large-scale re-organization of points or paragraphs and
ensuring that everything needed has been included, and everything that is not has been
removed. Part B of the Chapter, “Editing,” takes the student through the steps needed for the
smaller-scale sentence and word editing and polishing necessary to produce a final written
product.
The substance of the material and the exercises are substantially the same as in the
Sixth Edition, but it has been organized and streamlined to reflect the logical progression of
the writing process. Thus, this chapter is now easier to use as both a teaching tool and a
student’s reference book.

Teaching Notes

As noted at the beginning of Part C, this chapter lends itself to self-editing exercises
and peer review. In addition to the exercises here, you may want to develop your own exercises
from student papers or your own examples of common mistakes. You may find it helpful to
pull sentences or paragraphs from papers as you grade preliminary assignments and then hand
them out as revising and editing exercises. This provides the opportunity to focus on writing
problems common to a particular assignment. (If you use this approach, be sure to omit any
information that might identify the authors of the examples you have selected.)

Suggested Approach to Exercises

Exercise 15-A

This exercise works best if you tell students in advance what to look for. The discussion
needs a thesis statement, topic sentences for the paragraphs, transitions between cases, and a
transition between the cases and your client’s facts. Some students will rewrite this exercise
entirely unless you limit them to specific types of corrections. You may want a total rewrite,
but you should limit their efforts if you do not, in order to focus the students’ attention. You
may want to create a handout of this exercise, inserting spaces or empty text boxes at the places
in the discussion where paragraph breaks or signposts are needed.

Below is an example of how the specific problems might be corrected.

Although McGraw used a cane that resembled a gun during the


robbery, he did not use it in a manner likely to cause harm. He is therefore
not likely to be convicted of armed robbery. A person commits armed robbery
when, with intent to commit theft, he takes the property of another by use of an
“offensive weapon.” State Code § 365(b). The courts have defined “offensive

124
weapon” as including not only a weapon per se but also anything used in a
manner likely to cause death or great bodily injury. Fann v. State; Meminger v.
State.
McGraw’s cane was not a weapon per se. In both Fann and Choate the
defendants used replicas of guns to commit robberies. The court held in both
cases that the defendants could not be convicted of armed robbery because the
legislature had specifically eliminated statutory language that would have
brought replicas within the purview of the armed robbery statute. Like the
defendants in Fann and Choate, McGraw cannot be convicted of armed robbery
simply because he used something that looked like a gun.
In addition, McGraw’s use of the cane was not likely to cause death
or great bodily injury. For example, in Fann there was no evidence that the
defendant tried to strike the victim; in Choate the defendant was unable to do so
because the victim was seated in an enclosed booth. In both cases the court
noted the lack of any evidence of an intent to harm the victim. Similarly,
McGraw did not attempt to hit the owner, or show any intent to do so.
Although some facts indicate that McGraw may have intended to
strike the coin shop owner, a court is unlikely to conclude that he had that
intent. Unlike the defendant in Fann, who gave no indication of intent to strike
the victim, McGraw did swing the weapon back and forth menacingly and did
threaten to “hurt [the shop owner] bad” unless he cooperated. Also, the shop
owner stated that he was fearful of being either shot or struck, indicating the
weapon’s movement implied this threat. However, the court in Fann did not
infer an intention to harm without substantive indication of such intent. The
court is unlikely to find swinging the cane back and forth sufficient to establish
McGraw’s intent to strike the shop owner when McGraw made no attempt to do
so. Therefore, because McGraw did not use the cane in a manner indicating an
intent to harm the shop owner, he is not likely to be convicted of armed robbery.
The Meminger case, in which the court found that the defendant
showed an intent to cause harm, is distinguishable. In Meminger, the
defendant was convicted of armed robbery when he hit the victim in the head
with a liquor bottle. The court held that because the bottle was used in a manner
likely to cause harm, it was an offensive weapon. McGraw, on the other hand,
did not use the cane in a way indicating an intent to hit the shop owner; he only
swung the cane back and forth and did not swing it at the owner. Thus, because
the cane is not likely to be considered an offensive weapon, McGraw did
not commit armed robbery.

Exercise 15B

Direct your students to edit what is here, not to completely rewrite the analysis. Explain
that there are cases saying that there is no release from negligence unless the release specifically
refers to negligence. Those cases are analyzed in the discussion of the contrary conclusion,
which is not included here. That is why the text states that the issue is whether the word
“negligence” must be specifically included in the release to bar an action for negligence.
This paragraph needs to be divided into several paragraphs. Additionally, it needs a
thesis statement, topic sentences for the new paragraphs, and transitions between the cases and
your facts. In the example below, additions are in boldface and deletions are in brackets.

125
The court will likely conclude that the release Spencer signed bars
any action for negligence against the outfitter, even though it does not
mention negligence. Although the courts of this state have not addressed this
issue, they have held that releases must be interpreted to mean what they
say. The court in Conner, for example, held that a release was invalid and did
not bar a cause of action arising from a motorcycle accident at a racetrack. The
release purported to absolve the defendant from liability, but it mentioned only
“automobile racing” in a list of racing activities included. The court reasoned
that motorcycle racing was not included under the commonly understood
definition of automobile racing.
The court in Anders also indicated that a release must be read
according to its plain language. [In Anders], the court granted summary
judgment to the defendant after the plaintiff sued for damages arising from the
defendant’s negligence in drilling oil and gas wells. The contract between the
parties specifically absolved the defendant from liability arising from its
negligence. However, the court based its decision on the equal bargaining
positions of the parties and the unequivocal language of the contract.
Although both cases interpret releases according to their plain
words, neither Conner nor Anders [did not address] addressed the specific
question presented here because the contract between Spencer and Rocky River
Outfitters (RRO) [however] did not contain the word “negligence.” However,
given the courts’ emphasis on the plain meaning of the language of the
release, it is likely that the RRO release bars any action for negligence since it
relieved RRO from liability for any personal injury or property damage
sustained “as a result of this rafting trip.”
The courts in other states have applied the same plain-language
principle as this state’s courts, holding that broadly worded releases bar
negligence actions even if they do not use that word. For example, the courts
in Sommers and Cabel concluded that specific mention of the word “negligence”
was not necessary if the intent of the parties was otherwise clearly expressed.
The release in Sommers absolved a parachuting school from “any and all claims,
demands, actions . . . whatsoever, in any way resulting from personal injuries . . .
arising from . . . parachute jumping.” Similarly, in Cabel the release covered
“any and all losses, claims, actions, or proceedings of every kind and character . . .
arising directly or indirectly from any activity . . . such as parachuting.” In both
cases, the negligence actions brought by students injured in parachute jumping
were barred by the releases.
The RRO release is similar to those addressed in Sommers and
Cabel. In the release Spencer signed, he specifically absolved [Rocky River
Outfitters] RRO from liability for “any and all claims I have or may acquire
against RRO for any personal injury or property damage I may sustain as a result
of this rafting trip.” Like the releases in Sommers and Cabel, this release
specifically covered all claims that might arise from the specified activity in
language that was obviously intended to be all-inclusive. Because these cases
are consistent with the plain-language principle underlying this state’s
cases, Spencer will probably not be able to recover from RRO for the
injuries he has sustained

126
Exercise 15-C

There is more than one way to edit the following passages to remedy the problems of
imprecision and wordiness. These are samples of the type of editing that is needed:

1. Note that in this passage the phrase “appearance of impropriety” has been retained even
though it could be written without the nominalization. This phrase is a term of art used
when discussing the application of Canon 9.

Canon 9 requires a lawyer to avoid even the appearance of impropriety.


Although this requirement has led several courts to disqualify attorneys
when no actual impropriety existed, the appearance of impropriety is not
always sufficient to disqualify an attorney. For example, in Blumenfeld
v. Fusco, the court held that the marriage of two attorneys employed by
different firms did not create an appearance of impropriety sufficient to
warrant disqualifying the wife. The case involved a challenge to a will,
in which an associate in the firm representing the propounder was
married to a partner in the firm retained to represent the caveator. The
court found that no actual impropriety existed because the husband
worked in the real estate department and had no contact with the
attorneys who were actually representing the caveator. In declining to
disqualify the wife, the court balanced the need to avoid the appearance of
impropriety with the plaintiff’s right to employ the counsel of his choice.
The court concluded that the latter outweighed the former because any
actual impropriety was unlikely.

2. This example contains some nominalizations that are awkward to replace with an
adjective or verb. They have thus been left unchanged. Although the guidelines in this
chapter will generally improve the clarity of your students’ writing, they are
guidelines only and should not be applied mechanically.

No court has refused to consider fraud in an adoption case when there


was evidence of active concealment. For example, In re Baby J., the court
found fraudulent concealment of the truth when the adoption agency told
the parents that the birth mother was an eighteen-year-old unwed mother
who could not care for the baby. In reality, the agency knew that the
mother was a thirty-five-year-old patient in a mental institution and that
the father was presumed to be another patient. The adoptive parents
discovered the truth only after their thirteen-year-old adopted son was
diagnosed as having an inherited mental disorder. Similarly, in Roe, the
court awarded damages when the adoption agency placed three school-
aged children for adoption, telling the prospective parents that the
children were normal, healthy children. However, the agency had test
results and psychological evaluations showing that the children had a
history of dangerously violent behavior and other behavior disorders. In
both cases, the court emphasized that the agencies knew and actively
concealed the truth.

127
Exercise 15-D

Caution your students not to change the meaning of these sentences when editing. If the
meaning of the sentence is not clear, they should decide what they think the writer was trying
to say and rewrite the sentence to express that thought more clearly. These sentences may be
rewritten in a variety of ways. You may find it helpful to have the students work in groups to
rewrite the sentences and then compare the results. Here are some examples of how the
sentences may be rewritten:

1. Because medical malpractice lawsuits can be lengthy and expensive, plaintiffs and their
attorneys should carefully weigh the chances of success before filing a complaint.

2. The defendant’s fraudulent conduct was so reprehensible that the court should impose
the maximum fine allowed.

3. The attorney strenuously objected to the witness’ testimony as immaterial, irrelevant,


and a product of hearsay.

4. In most jurisdictions, the “fireman’s rule” bars any recovery by emergency personnel
for injuries suffered in the line of duty.

5. Because the deadline for answering had passed, the attorney filed a motion to open the
default.

6. The plaintiff alleged that the manufacturer’s warning about the hazardous product was
insufficient.
7. Discrepancies in the defendant’s testimony persuaded the jury of her guilt as to all charges.

8. The plaintiff contended that he should be given a new trial because of the unethical
conduct of the defendant’s attorney.

9. The court will hold the witness in contempt until he discloses the required information.

10. The possible injury to the lake outweighs the benefits of a recreational camp.

128
Part D
Office Memoranda
& the Opinion Letter

129
16
Objective Writing

Changes from the Sixth Edition

There are no major changes from the Sixth Edition.

Teaching Notes

This chapter addresses the objectivity necessary for legal memoranda and opinion
letters just as Chapter 22 (Persuasive Writing Techniques) discusses advocacy in brief writing.
Chapter 16 provides another opportunity to refer back to earlier chapters as students apply
some of the fundamental skills they have already learned in the context of a formal
memorandum. Chapters 11 (Organization) and 14 (Drafting the Analysis) are particularly
relevant, and some teachers work with all three of these chapters at the same time, showing
students how the essential principles of good organization remain constant.

Suggested Approach to Exercises

Objectivity is the hallmark of an effective and complete memo Discussion, but students
often struggle with presenting both sides of and issue. Writing a judicial order or opinion can
help students approach an issue more objectively than when they are given a client for whom to
advocate. Exercises 16-A and 16-B require the students to write trial court opinions deciding a
motion for summary judgment and will help the students develop an objective approach and
tone. The judicial opinion will require students to use the rules from Chapters 11 and 14 to
reach a conclusion and fully explain how the law and facts support that conclusion.
The CiRAC acronym may help the students to remember each step of the analysis.
However, while students must take into account adverse facts or potentially troublesome
aspects of the law, a judicial opinion does not usually separately address the contrary
conclusion to an issue or sub-issue, as discussed in Chapters 11 and 14. When using these
exercises, caution the students against adopting a pompous or condescending tone, which can
emerge when students write from a judicial perspective.
Exercises 16-C asks the students to draft the discussion that would be found in an
analytical legal memorandum, analyzing the merits of a client’s potential claim. The discussion
should use the rules from Chapters 11 and 14; using the CiRAC acronym may help the students
to remember each step of legal analysis. After students have completely explained how the law
applies to the facts to support their conclusion, their discussion should specifically address any
potential contrary viewpoints to consider the weaknesses in their position.
Exercise 16-D asks students to draft a shorter summary-of-the-law memo that only
explains the law relevant to a legal question. This type of discussion should also use the rules
from Chapters 11 and 14, although the shorter memo will not include an application section or
address the contrary viewpoint as no facts have been provided for analysis.

130
Exercise 16-A

The factual and legal materials used for this exercise are also used in Exercises 7-A, 10-A, 12-
A, and 13-A. Using the example in Appendix G of the text, the caption for this case should
look something like this:

CALVERT COUNTY DISTRICT COURT FOR


THE STATE OF CHEASAPEAKE

HUGH GREEN, ) CASE NO. CV 20-671


)
Plaintiff, )
)
v. ) ORDER RE: SUMMARY
) JUDGMENT MOTION
NEIL McKAY, )
)
Defendant. )
______________________)

It would also be helpful to include a short introduction as shown in Appendix G of the


text, that might look something like this:

Defendant Hugh Green, has moved this court for summary judgment against
Defendant Neil McKay in this contract action. Green contends . . . . For the
reasons that follow, Defendant’s motion is . . . .

Thereafter, students should be able to draft the sections specified. Each section should
reflect the student’s conclusion as to the likely outcome and will vary depending on the rule
you have chosen and the student’s decision. The sample trial court order in Appendix G can be
used as a model for the student’s written opinion.

131
Exercise 16-B

The factual and legal materials used for this exercise are also used in Exercises 7-B, 10-
B, 12-B, and 13-B. Using the example in Appendix E of the text, the caption for this case
should look something like this:

HUMBOLDT SUPERIOR COURT FOR


THE STATE OF HURON

STAR CITY BANNER-PATRIOT, ) CASE NO. CIV 121-2020


)
Plaintiff, )
)
v. ) ORDER RE: SUMMARY
) JUDGMENT MOTION
STAR CITY, )
)
Defendant. )
___________________________ )

The students’ opinions should be drafted as indicated above under Exercise 16-A.

Exercise 16-C

The factual and legal materials used for this exercise are also used in Exercise 11-A. If
students have already worked through Exercise 11-A, this exercise requires them to draft a
Discussion from their outline for that exercise. If they have not, the Discussion should be based
on the kind of analysis shown in the suggested response to Exercise 11-A, Questions 2 (c) or 3
(c), in this manual. This exercise works well as a hand-in assignment or as an exercise that
students share with their classmates for peer review.
The answer we outlined for that exercise provides a starting point. The students should
follow the Chapter 14 steps for translating their outlines into a Discussion. The CiRAC
acronym may help the students to remember each step of the analysis. In the context of this
chapter, you might consider these teaching points:

 Objectivity in drawing a conclusion. Problems to look for include waffling,


overconfidence, advocacy, and lack of confidence.

 Objectivity in describing the law. Problems to look for include failure to explain the
elements (particularly the confidential relationship element in Kendall) and
selective use of facts from the cases.

 Objectivity in explaining the analysis. Problems to look for include failure to


consider relevant facts, advocacy, and failure to discuss or reluctance to acknowledge
the strength of potential contrary viewpoints.

132
Exercise 16-D

The factual and legal materials used for this exercise are also used in Exercises 11-B,
18-C, and18-G. If students have already worked through Exercise 11-B, this exercise requires
them to draft a Discussion from their outline for that exercise. If they have not, the Discussion
should be based on the kind of analysis shown in the suggested response to Exercise 11-B,
Questions 2 (c) or 3 (c), in this manual. This exercise works well as a hand-in assignment or as
an exercise that students share with their classmates for peer review.
The answer we outlined for that exercise provides a starting point. The students should
follow the Chapter 14 steps for translating their outlines into a Discussion. The CiRAC
acronym may help the students to remember each step of the analysis. In the context of this
chapter, you might consider these teaching points, which are virtually identical to those for
Exercise 16-C:

 Objectivity in drawing a conclusion. Problems to look for include waffling,


overconfidence, advocacy, and under-confidence. These points are particularly
important for the issues raised by this problem, because students may follow their
feelings.

 Objectivity in describing the law. Problems to look for include failure to explain the
elements (particularly the unreasonable interference element in Peters) and selective
use of facts from the cases.

 Objectivity in explaining the analysis. Problems to look for include failure to


consider relevant facts, advocacy, and failure or reluctance to discuss or
acknowledge the strength of potential contrary viewpoints.

133
17
Elements of a Summary-of-the-Law Memorandum

Teaching Notes

This chapter addresses a memorandum that narrowly addresses a specific legal


question. These summary-of-the- law memos differ from analytical memos not only in length,
but in scope. Often no facts are provided - only the request to “get me the law on_____.” As
indicated in the chapter, the request may sometimes include a fact or two, and the response
should take those facts into account when summarizing the law. But since a full summary of
the facts is not provided, the memo cannot include the application section that is part of the
analytical memo – the section following the law summary that applies the law to the facts and
considers any counteranalysis to the writer’s conclusions. Thus, in short-hand form, the
summary-of-the-law memo includes the CiRC of the CiRAC structure set out in Chapter 14,
but omits the A or application of the law to facts.

This type of document provides a concise summary of the law on a specific point.
Given the fast pace of the practice of law, it is often requested by email or orally, and a
response is expected in a very short time-frame, often by email. Thus, it is critical that the
writer understand the question asked and focus on answering that question directly and
accurately, as quickly as possible. Emphasize to the students that a memo that is shorter or
produced quickly has to be just as complete and accurate as a longer analytical memo – advice
is likely to be given or decisions made on the basis of the information provided. Since these
types of memos are often submitted by email, it would be helpful to review the considerations
for email communications in Chapter 16.

Suggested Approach to Exercises

As indicated in the chapter, the students should write a question presented for each
summary of the law memo, to define the scope of the memo. The Brief Answer should directly
answer the question asked. If the Brief Answer is omitted, the summary of the law should
begin with the answer to the question asked, followed by the rule or rules relevant to the
question. Students may want to end the memo at this point, since the law has been provided.
But as also illustrated in the chapter, including a short summary of 2-3 relevant cases is
necessary for the reader’s understanding of how the law has been applied to specific
circumstances. Without this context, the reader will not be equipped to reach a conclusion
about the client’s situation. Below are examples of memos for each exercise. Students’ memos
may vary, but they should include all of the necessary information.

Exercise 17-A

Students should write this summary-of-the-law memo as an email response, as requested. No


facts are provided and the exercise directs the students to the 2 cases set out in Exercise 8-B.
Thus, the memo should only summarize the law that defines when doctors may be held liable
in a medical malpractice case, as set out in the Moulton and Mallard cases. Note that the cases
focus on the question of proximate cause, not the actual standard of care to determine
negligence. This type of imprecision in a request is not unusual, and provides an opportunity to

134
discuss with the students how to respond to this situation. In real life, one could send a question
to the attorney who requested the information to clear up the confusion. In this exercise, the
students should summarize the law in the 2 cases designated, but should also point out the
distinction between standard of care and proximate cause. This ensures the reader understands
that the cases addressed a different legal question and provides the opportunity to clarify which
question the requesting attorney needs to have answered. A suggested response:

TO: Partner
FROM: Student
DATE:
RE: Physician’s potential liability

Question Presented: When can a doctor be held liable in a medical malpractice


case?

Brief Answer: A doctor may be held liable when his negligence more likely
than not caused the plaintiff harm, which has been defined as preventing a
“substantial possibility” of survival or recovery.

Summary of the Law: A physician may be liable in a medical malpractice case


when the doctor’s negligence “more likely than not” caused the harm to the
patient. Mallard v. Harkin (1999). The court has further defined this standard to
mean that the doctor’s negligence prevented a “substantial possibility” of survival
or recovery. Moulton v. Ginocchio (1996). In Moulton, the plaintiff died after the
defendant doctor misdiagnosed her intestinal obstruction as a “stomach bug.”
The court held the defendant doctor liable since expert testimony established
that the plaintiff would have survived with prompt surgery, thereby meeting the
“substantial possibility” requirement. However, in Mallard the court found no
liability for the defendant doctor whose failure to diagnose and perform surgery
for a degenerative disease left the plaintiff paralyzed. The expert testimony
established that the plaintiff had a “possibility” of recovery with the surgery, but
“probably would not have recovered.” The court noted that the law does not hold
a doctor liable on the mere possibility that the plaintiff would have recovered,
but requires that the harm was “more likely than not” caused by the defendant’s
actions. Id. Thus, a doctor may be held liable when his actions “more likely than
not” caused harm, which has been defined as preventing a “substantial
possibility” of recovery.

Note that these cases address the question of proximate cause in a medical malpractice
action. Your initial question was framed in terms of the standard of care for doctors in medical
malpractice actions. Would you like me to further research the standard of care issue and send
a follow-up memo?

Exercise 17-B

135
This exercise asks students to write a summary-of-the-law memorandum based on the
three cases used in Exercise 8-C. Students’ memos may vary. One example:

Question Presented: What does the law require a plaintiff to show to establish a
private nuisance?

Brief Answer: to recover in an action for a private nuisance, a plaintiff property


owner must show that the defendant landowner has unreasonably used his
property in a manner that interferes with the plaintiff owner’s legal rights to use
and enjoy his property.

Summary of the Law: A private nuisance can be the basis for a tort action by
one landowner against another. A private nuisance arises when one landowner’s
use of his property unreasonably interferes with the legal rights of a neighboring
or adjacent landowner. Blum v. Disposal Systems, Inc. (1997).

To determine whether the defendant’s use was reasonable, the court


balances the interests of the respective landowners and has held that “a use of
property that essentially confiscates or destroys the neighboring property is
unreasonable.” Id. For example, in Shover v. Scott (1889), the defendant’s
excavation of his adjacent property caused the plaintiff’s land to cave in. The
court held the plaintiff was entitled to damages since a landowner has a legal
right to lateral support of his property. However, the court has also recognized
that not every interference with an adjacent landowner’s use of the property is a
nuisance. In Cassells International v. Avery Resorts (2012), the plaintiff sued,
alleging that the defendant’s proposed 10-story building would block the sun
from the beaches and other sunbathing areas of plaintiff’s hotel, rendering them
unfit for their intended purposes. In declining to recognize a legal right to the
free flow of sunlight, the court in Cassels distinguished Cohen v. Andrus, a case
that had recognized such a right. The Cohen court did so in the context of solar
energy collection, noting the increasing need for solar power. The Cassells court
concluded that this policy was not relevant to sunbathing and declined to hold
that the plaintiff had a legal right to sunlight. Id. Thus, the determination of
whether the defendant landowner’s use of the land constitutes a private nuisance
turns on whether the defendant unreasonably interfered with a plaintiff’s legal
rights to use his property.

Exercise 17-C

Note: the example below uses a format that excludes the Brief Answer as a separate
section. Thus, the answer to the question asked introduces the summary of the law, answering
the question at the outset. This, like the introductory conclusion (Ci) discussed in Chapter 14
(Drafting the Analysis), provides a context and prepares the reader to understand the case
discussion that follows.
This exercise asks students to write a summary-of-the-law memorandum based on the
two cases used in Exercise 12-D .Students’ memos may vary. One example:

Question Presented: Are plants growing on property considered personal

136
property or part of the realty?

Summary of the Law: Generally, plants and trees growing on real property are
fixtures and thus, are considered part of the land. However, crops are considered
personal property even though they grow in the soil, because it is the
landowner’s intention to harvest them. Stafford National Bank v. Dinesen
(1958). In Stafford, the court held that cranberry bushes were part of the realty
since only the berries, not the bushes themselves, were intended to be harvested.
Id. However, in Updike v. Teague (1995) the court concluded that trees growing
on the property were not part of the realty because they were planted as part of
the plaintiff’s nursery business. The court concluded the trees were the
plaintiff’s “stock in trade” and thus were personal property, reasoning that they
had been planted in the soil only to keep them alive prior to resale. Id. Thus, the
relevant question to determine whether trees or plants are part of the realty is
whether the owner intended to harvest them.

137
18
Elements of an Analytical Legal Memorandum

Changes from the Sixth Edition

In the sixth edition, the Questions Presented and Statement of Facts for an analytical
memorandum were mentioned only briefly here and were addressed in detail in Chapters 20
and 21. This chapter now consolidates the material from these chapters. This material has been
streamlined and integrated to allow a more unified approach to teaching the elements of a
memo, without having to assign several chapters. They are, however, still presented as separate
sections in the chapter for those who prefer to teach these elements with a more concentrated
focus on specific elements or in a different order.

Teaching Notes

The transition from writing discrete exercises to writing a complete legal document is
an important one for students. Consider using this class to talk about professionalism and the
role that writing will play in your students’ careers. You might talk about the role that good
writing and analysis will play on their law school exams. Some professors, looking beyond the
first year, bring advertisements from the career development office to class, showing students
that virtually all employers are looking for law clerks and associates who have strong research
and writing skills. Learning legal writing and analysis is hard work; it is helpful to remind
students what they are working for.
The material in this chapter is best reinforced by a short lecture outlining the elements
and pointing to the sample memorandum in Appendix C of the text. When you give your
students a memorandum assignment, be sure to tell them whether you will require Brief
Answers (which the text says are optional), or perhaps require them to write their memos in
other ways that differ from the format described here. You should also tell your students
whether you expect them to incorporate recommendations into the Conclusion.
You can use a memorandum assignment as a vehicle to revisit all of the basic concepts
covered in Part B of the text. For example, the techniques in Chapter 6 (Understanding Legal
Rules) and Chapter 7 (Identifying & Selecting Issues for Analysis) will help students develop
the large-scale organization of their memo, while the techniques in Chapters 8 and 9 (Common
Law Analysis and Statutory Analysis) can be used to begin to develop analogies and
distinctions. The lessons in Part C will help students draft accurate descriptions of the law and
clear explanations of their analysis of the issues, and will help them make their memos
complete and reader-friendly.
Many students have trouble writing objectively, especially when they know which of
the parties in a problem is “their” client. To help them understand what an objective question
presented or statement of facts should look like, consider dividing your class into three groups:
plaintiffs, defendants, and law clerks or judges when discussing examples or exercises. You
can also use the approach adopted by some teachers of having the students write their memo
from the standpoint of a judge’s law clerk. This can encourage objectivity since the judge has
no vested interest in which party prevails, unlike the hypothetical representation of a client.
Some first-year law students have trouble writing questions presented or fact
statements because they lack confidence in their ability to identify the legally significant facts
or to accurately identify and define the issues. Consequently, they err on the side of including

138
information lest they inadvertently omit something important, they are easily distracted by
emotional facts that “feel” important but in reality, are not, or they miss the undramatic but
essential fact. The exercises below give you an opportunity to explore these common
problems with your students.

Suggested Approach to Exercises

Exercise18-A

The facts allow students to reach either conclusion. A brief answer reaching the
“probably yes” conclusion is given in Answer B on p. 305. A contrary conclusion could be
written:

Probably not. Although the restraint necessary to prove false imprisonment need
not be physical or forceful, the plaintiff must still establish that he was actually
confined or restrained. Even though the defendant is older and has a reputation as
a bully, she did not prevent the plaintiff from continuing down the sidewalk nor
did she touch him in any way. She was not even present during the duration of
the alleged restraint. Thus, the court would likely there was insufficient evidence
to show actual restraint.

Exercise 18-B

The factual and legal materials used for this exercise are also used in Exercises 7-C, 10-
C, 12-C, and 13-C.

1. Since there are no facts provided in this question, students should write a question
presented that only states the legal question posed; they should not assume that the
factual context is that set out in Exercise 7-C. One example is provided below, but other
approaches may work equally well.

Under what circumstances does one party’s misrepresentation allow for a


contract to be set aside?

2. The issue is whether the contract complies with the statute of frauds. Answering this
question may begin with an in-class discussion, of the rule and the elements charts for this
exercise as shown in the materials for Exercise 7-C in this manual.

Rule: The statute requires that a contract (1) for the sale of land or a legal
interest in land (2) must be in writing and (3) must be signed by the
persons to be charged with the contract. In the Treacher case, the court
interpreted the second element to require that the writing identify with
reasonable certainty and accuracy (a) the parties to the contract, (b) the
subject matter to which the contract relates, and (c) the terms and
conditions of all promises, including payment conditions.

Students can eliminate several “givens” from detailed discussion. This contract was for
the sale of land or a legal interest in land. The napkin stated the parties to the contract

139
and the subject matter. Thus, the issue involves two sub-issues:

1. Whether the contract states with reasonable certainty and accuracy the
terms and conditions of all promises.
2. Whether the contract was signed by the persons to be charged.

Legally significant facts for each sub-issue include the following:

1. Whether the contract states with reasonable certainty and accuracy the
terms and conditions of all promises.
 The offer on the napkin describes the property offered for sale
as the Blue Goose Inn
 The offer on the napkin states the purchase price.
 The offer on the napkin does not state payment conditions.

2. Whether the contract was signed by the persons to be charged.


 The seller, Zoeller, wrote the offer.
 The buyer, Hughes, did not sign the contract written on the
napkin.
 Hughes orally accepted the offer and her oral agreement was
recorded on a tape recorder.

Students should draft the Questions Presented based on the rule and the facts outlined
above. Some students may find this challenging either because the issue includes sub-
issues or because there are a number of relevant facts. Encourage them to be concise
and choose only the most important facts. Another alternative is to write separate
questions for each sub-issue. Two possible answers are shown by the examples below.
The first Question Presented in the memo in Appendix C in the text also illustrates the
latter approach.

Examples:

I. Does a contract for the sale of a tavern comply with the statute of
frauds, which requires the contract to state the terms and
conditions of the agreement with reasonable certainty and be
signed by the person to be charged, when the seller wrote the
name of the tavern and sales price on a napkin but specified no
terms for payment, and the buyer did not sign the agreement but
her oral acceptance was recorded on an audio tape?

Or:

I. Whether a contract for the sale of a tavern complies with the


statute of frauds.

A. Whether the contract states with reasonable certainty and


accuracy the terms and conditions of all promises when it

140
included only the name of the tavern and purchase price
and not the conditions of payment.

B. Whether the contract was signed by the person to be


charged when the buyer did not accept in writing but her
oral agreement to the contract was recorded on tape.

Exercise 18-C

The factual and legal materials used for this exercise are also used in Exercises 11-B,
16-D, and 18-G.

1. An analytical memo applies the law to a set of facts. Thus, for this question the students
should consider the facts presented in Exercise 11-B. The process for identifying
relevant rules, elements, issues, and sub-issues for this exercise is illustrated in the
answer to that exercise in the Chapter 11 materials. It identified the following issues
and sub-issues:

Whether All-Rite’s sulfur dioxide emissions trespass on the farmers’ land.

1. Whether the sulfur dioxide emissions are tangible and


therefore physically invade the farmers’ land.
2. Whether All-Rite’s emissions are privileged by necessity.

2. The legally significant facts for each issue and sub-issue are drawn from the charts used
to answer Exercises11-B and include the following:

 Sulfur dioxide is emitted into the air and settles on the farmers’ alfalfa
fields, turning them white.
 Sulfur dioxide emissions reduce the value of the farmers’ crops by 5%,
reduce the value of their property by $5,000 to $10,000, and probably
increase their respiratory ailments.
 All-Rite may be able to do more to reduce emissions even though the coal
used by All-Rite has lower sulfur content than coal burned in other states.
 All-Rite employs 490 people and provides income for the community.
 It may be economically feasible for All-Rite to further control emissions.
 All-Rite’s emissions are not necessary to save human life or health.

Drafting the Question Presented requires the students to put the relevant facts listed above in
the context of the elements at issue. For example:

I. Do plaintiff farmers have a cause of action for trespass because sulfur


dioxide particles emitted by the defendant’s factory settle on their crops
causing respiratory ailments among the farmers and a decrease in the
value of their crops, where the law requires a plaintiff to show a physical
invasion of the plaintiff’s land by a tangible object that is not privileged
by necessity in order to bring such an action?

141
Students may also choose to write separate questions for each sub-issue, as illustrated in the
example above for Exercise 18-A.

Exercise 18-D

Give each group a sufficient amount of time to write an objective statement of facts for
this exercise, then compare the results and discuss their differences. Emphasize that while the
Statement of Facts must be objective and must include all legally relevant facts, the students’
drafts of this section will vary to some extent and may change as the students begin to analyze
the facts in light of the law.

1. Since lawyers are rarely confronted with a neat summary of the relevant facts, this
exercise presents students with the challenge of compiling facts from different sources
and determining which are relevant to the legal issue presented. The legally significant
facts for this issue are those that affect whether the property was fit for human
habitation. Students will often focus initially on the facts showing the property was
unfit, but those showing the apartment had been renovated to some extent are also
relevant. Thus, the legally relevant facts include:

 Farley leased the apartment.


 The toilet did not work.
 There was no hot water.
 There were two rats in the apartment.
 Cockroaches were everywhere.
 There was a pile of dirty dishes in the sink.
 There were large holes in some of the walls outside the kitchen area.
 The floors were grimy.
 Several windows were broken.
 The kitchen area had just been renovated.
 There was a new refrigerator, stove, and microwave, all in working order.
 The rental agent refused to rescind the lease.

These facts all concern the habitability of the apartment itself. If students want to assert
that facts regarding conditions outside the apartment are also relevant to its habitability,
have them explain why. (In fact, you might respond to every assertion that a certain fact
is legally significant by asking why.) Those facts are:

 There is a run-down park across the street with overgrown bushes that could harbor
muggers.
 There was garbage spilling out of dumpster in the alley.
 There was a boarded-up building next door.
 A man was asleep on the front steps.
 The hubcaps on Farley’s car were stolen while he was in the apartment.

Some students will argue that these facts are relevant to the warranty of habitability.
You can use this question to emphasize the cumulative effect of the facts in evaluating

142
the habitability of the apartment. If you remove one of those facts, does the outcome
change? Two of the facts? Most of the time, the answer will be no.

You can also use this question to show that some of the facts are more important than
others, even facts about the habitability of the apartment. It probably matters that
Farley’s hubcaps were stolen and that the overgrown bushes in the park could harbor
muggers. These facts, cumulatively, could be used to show the neighborhood is unsafe
However, the overflowing dumpster and boarded-up building, while unattractive, likely
do not contribute much to the habitability question.

The issue is whether the warranty of habitability was breached and the rule states that
the warranty is implied from the leasing of the property. Thus, even though the
representations about the apartment in the ad and statements by the rental agent have
considerable emotional importance and may raise questions of fraud or
misrepresentation, they are not relevant to the warranty issue.

2. Basic background facts: Farley had just moved to the city, was looking for an
apartment, answered a newspaper ad, leased the apartment before he looked at it based
on the agent’s representations, and then discovered the condition of the apartment. These
facts, which can be explained in a little more detail, tell the story and provide a context.

3. The Statement of Facts should begin with a context sentence and then set out some of
the background facts. That is, Farley moved to the city, was looking for an apartment,
answered a newspaper ad, and leased an apartment based on the rental agent’s
representations before he looked at it. It should then include a detailed description of
what he saw when he visited the apartment and conclude with a statement that the agent
refused to rescind the lease.

Exercise 18-E

The question here is not whether Ms. Wilkins is violating the covenant not to compete;
that is established by the facts. The question for students to consider is whether the covenant
not to compete is reasonable.
Students will likely focus on two issues that determine whether the covenant is
reasonable: (1) whether the covenant is narrowly tailored in geographic scope, and (2) whether
the covenant is narrowly tailored in duration. Many students, however, fail to recognize the
additional requirement that the covenant’s restrictions must be designed to protect the
employer’s legitimate interests and therefore must be narrowly tailored to achieve that goal. If
the scope of the restrictions on an employee exceed the employer’s legitimate interests, the
covenant is not reasonable. A logical approach to this exercise is to have students work on one
legal issue at a time.

1. Legally significant facts are those that affect the reasonableness of the two types of
restrictions, including their relevance to the legitimate interests of the employer. These
facts include:

Legitimate interests of the employer

143
 The Agreement prohibits an employee from performing or offering speech and
hearing therapy services for a specific time and within a specific area.
 The Agreement prohibits an employee from providing speech and hearing therapy
services to any of Sterling Speech & Hearing Center’s clients or patients.

Two questions often arise when discussing the legitimate interests of the employer.
First, it is not entirely clear whether the 18-month and 15-mile restrictions apply to the
second prohibition listed above, or whether that prohibition is open-ended. Students
should understand that although the law will apply differently depending on how the
second prohibition is interpreted, the prohibition itself is legally significant either way.

Second, students may want to discuss the prohibition in part (a) of the Agreement,
which prohibits an employee from taking Sterling Speech & Hearing Center’s patients,
clients, and confidential records. Although there are facts in the file that appear to raise
this as an issue, the employer herself says that she “has no actual proof” that the
employee violated this part of the covenant and the employee denies that she did.
Absent some proof, this part of the covenant will probably not be litigated and facts
related to that issue are not legally relevant

Geographic scope

 The Agreement establishes a 15-mile restriction.


 Sterling Speech & Hearing Center had a contract to serve students in the Ross City
school district.
 Ms. Wilkins’s case load as a Sterling employee included students in the Ross City
school district.
 Ross City Speech Clinic has the new contract to work with the Ross City school district.
 The Ross City Speech Clinic is 14.5 miles from the Sterling Speech & Hearing Center.

Duration
 The Agreement establishes an 18-month restriction.
 Ms. Wilkins declined to renew her contract with Sterling Speech & Hearing Center
in May 2013.
 Ms. Wilkins began working for Ross City Speech Clinic in August 2013.

2. Basic background facts include:

 Ms. Wilkins worked for Sterling Speech & Hearing Center for approximately one year.
 She married Justin Brill during that year.
 Brill is the director of the Ross City Speech Clinic.
 Dr. Brandt liked Ms. Wilkins’s work and her rapport with students in the Ross City
school district.

The exercise also contains a number of distracters, emotional facts, and irrelevant facts.
Consider asking students to list those and explain why they are not either legally
significant facts or background facts. In the alternative, ask individual students to each
categorize one of the emotional or irrelevant facts and explain their choice.

144
3. At least a couple different approaches could be taken to the Statement of Facts. Each
approach should start with a context sentence that identifies Ms. Wilkins as a speech
therapist and states that her former employer, Sterling Speech & Hearing Center, is
seeking to enforce a non-compete agreement signed by Wilkins.

Thereafter, a chronological fact statement would explain that Ms. Wilkins entered into
an employment contract with Sterling Speech & Hearing Center, quote the relevant
language from the Agreement Not to Compete, and describe Ms. Wilkins’ work with
the Ross City school district during her year of employment. The narrative would then
explain her marriage to Mr. Brill, the director of the Ross City Speech Clinic, end with
the events between May and August 2013: Ms. Wilkins’s declining to renew her
contract with Sterling Speech & Hearing Center, her employment at Ross City Speech
Clinic, and the change in the Ross City school district contract.

In the alternative, but perhaps more challenging to write, a student could follow the
introductory sentence with background information – i.e., that Ms. Wilkins took a job
with Sterling Speech & Hearing Center, that the Center’s director is Kathleen Sterling,
that Ms. Wilkins was married in February 2013, and that her husband is director of Ross
City Speech Clinic – then shift to the legally significant facts. This part of the Facts
would begin with the language from the non-compete agreement, then develop the
legally significant facts relevant to the requirements of the covenant that are implicated
by the facts.

Additional Exercises

The following exercises can provide additional opportunities for your students to
practice writing Questions Presented and/or Statements of Facts. Both are related to other
exercises that you may be using.

Exercise 18-F

The factual and legal materials used for this exercise are also used in Exercises 12-D,
13-D, and 17-C. Identify the relevant rules, issues, and significant facts in Exercise 12-D and
draft the Questions Presented.

Suggested Approach:

This exercise presents a challenge because the issue is composed of alternatives rather
than elements or factors. The broad question is who owns the trees. The answer depends on
how the trees are characterized. They are real property or personal property, and personal
property only if they are crops or stock in trade. This exercise might work best as a class
exercise rather than as a take-home assignment.
Here is an example of how the questions could be drafted. Because all of the significant
facts are relevant to each sub-issue, they are grouped together in the main question:

145
Whether young trees planted in the backyard of residential property belong to
the purchasers of the property or to the previous owners of the property when the
previous owner purchased the trees for resale in their nursery, temporarily
planted the trees in the yard because there was no space in the nursery, and did
not remove them prior to selling the property.

A. Whether the trees belong to the purchasers of the property according to


the common law rule that plants growing in the soil are part of the real
estate.

B. Whether the trees belong to the sellers because they are either

1. crops, which are defined as plants placed in the soil that the owner
intends to harvest and which are the personal property of the owner
who planted them, or
2. stock in trade, which is defined as goods or chattel obtained by a
merchant for the purpose of resale and which are the personal
property of the merchant.

Exercise 18-G:

The factual and legal materials used for this exercise are also used in Exercises 11-B,
16-D, and 18-C, above.
This example illustrates how to draft a Statement of Facts for teachers who use Exercise
11-B and illustrates another use of the charts described in Chapters 7, 8, and 9. The significant
facts of this case should be those identified in those charts. In this case, the significant facts are
drawn from the elements charts used to help answer Exercise 11-B:

Regulation issue:

 Powerhouse has capacity of 500,000 pounds of steam per hour.


 Powerhouse burns coal containing 0.9% to 1.2% sulfur by weight.

Nuisance issue:

 All-Rite owns land on which the factory is located or a leasehold estate in the land.
 The factory emits sulfur dioxide into the air.
 All-Rite employs 490 people and provides much income for the community.
 Coal has lower sulfur content than coal burned in other states.
 Sulfur dioxide emissions reduce value of farmers’ crops by 5%, reduce value of
their property by $5,000 to $10,000, and probably increase their respiratory
ailments.
 These crops are grown on the farmers’ land.

Trespass issue (facts not already listed):

 Sulfur dioxide settles on farmers’ alfalfa fields.

146
 It may be economically feasible for All-Rite to further control emissions.
 All-Rite’s sulfur dioxide emissions are from a short smokestack.
 All-Rite’s emissions are not necessary to save human life or health.

Key background facts might include:

 All-Rite factory makes chemicals, inks, and dyes for many commercial products.
 Source of information about damages (e.g., tax assessor, physician), and details
about alfalfa damage (e.g., whitens leaves).
 All-Rite has ignored the farmers’ complaints.

Students may want to include the newspaper’s advocacy on behalf of the company, but
these additional facts are not legally or (for the most part) emotionally significant.
Students may find summarizing the legally relevant facts in this problem a bit
challenging, because it does not lend itself to a chronological account of the facts. The third
principle under the statement of facts section (Organize the Facts Intelligibly) provides a useful
starting point, however. The statement is more intelligible if the facts concerning the factory’s
operations and emissions are described before the farmer’s complaints, for example, because
the complaints make no sense otherwise.
Because of the use of charts in identifying facts, moreover, students may be tempted to
organize the statement according to issues and sub-issues. Students may need to be reminded
that they must tell a story. For example, each of the three legal rules involves facts concerning
the plant’s emissions. Students should, of course, group those facts rather than state them in
three separate places.
The fourth principle (Describe the Facts Accurately and Objectively) will be a problem
for some students because they will characterize the facts or generalize about them. This may
occur because students have strong feelings about what the outcome should be or because they
are not thinking precisely.

147
19
Opinion Letters

Changes from the Sixth Edition

There are no major changes from the sixth edition.

Teaching Notes

This chapter includes guidelines for writing an opinion letter, sometimes called an
advisory letter since these letters are generally written to answer a client’s question or clarify a
client’s position with regard to a specific legal matter. Since the information provided to a
client is the layman’s version of the same information included in an office memorandum, it
may be helpful to refer back to the chapters regarding specific sections of the memo when
teaching the corresponding sections of the opinion letter. For example, in deciding which facts
must be explained in the opinion letter, the chapter on writing the Statement of Facts for the
memo may be helpful.
Since opinion letters are written to communicate with clients who are not law-trained,
the law and its application must be explained in a straightforward manner, without using legal
terminology. For this reason, it may be helpful to have students draft a client letter while they
are working on, but have not finalized, a memo. This exercise can ensure that students have
identified the relevant facts and synthesized the law during the memo writing process. Since
the opinion letter is shorter and must explain the law and its application in terms a layperson
can understand, it can be a good way to check whether the students have adequately analyzed
both the law and the facts in order to reach a solid understanding of both. As the students’
understanding of the law and its application become more well-defined, the opinion letter will
need to be revised. An opinion letter may also be included as part of the memo assignment but
will again need to be revised after you provide feedback on the memo, as any analytical
problems in the memo will also be reflected in the advisory letter.

Suggested Approach to Exercises

Exercise 19-A

The factual and legal materials used for this exercise are also used in Exercise 14-A.

This exercise is based on the materials from Exercise 14-A in the text and the
corresponding material in the Teacher’s Manual. The letter to Allen Worthington should:

 Answer Mr. Worthington’s question as to whether he is likely to recover damages


to compensate him for his injuries. While this answer should be phrased in terms of
likelihood rather than guaranteeing any specific outcome, it should nevertheless
provide Mr. Worthington with a concrete answer.

 Summarize the facts clearly. Since the facts provided are short and straight-
forward, this section may only take a few sentences. But make sure the students

148
include the facts showing both Deemer’s propensity towards violence and those
showing Watson’s awareness of those facts.

 Explain the law, in layman’s terms, regarding an employer’s liability for the
intentional torts of an employee. This explanation should not include any references
to specific cases and should avoid legal terms like “tort” or “propensity toward
violence requirement” that are appropriate in a legal memorandum.

 Objectively explain how the legal requirements apply to Mr. Worthington’s


situation. This explanation should not only include how Mr. Worthington may be able
to establish the elements necessary to recover, but also how Mr. Watson may be able
defend against this action.

 Communicate in a conversational but professional tone.

Exercise 19-B

The factual and legal materials used for this exercise are also used in Exercises 8-B, 14-
B, 17-A, and 23-C.

This exercise is based on the materials from Exercise 8-B in the text. Refer to the
discussion of these materials in the section on Chapter 8 in the Teacher’s Manual. The letter to
Mary Quale should:
 Answer Ms. Quale’s question as to whether she is likely to succeed in suing Dr.
Farmer for malpractice. This answer should be phrased in terms of the likelihood of
success, but should nevertheless provide Ms. Quale with a concrete answer.

 Summarize the facts clearly. Since this case will turn on the question of causation,
the specific facts regarding Andrew’s chance of surviving must be included, as well
as those showing Dr. Farmer’s negligence.

 Explain the legal standard for recovery in a medical malpractice action in light of
both relevant cases. Although phrases like “more likely than not” that define the
legal standard are appropriate, legal terms of art like “proximate cause” or specific
references to the cases should not be used.

 Objectively explain how the law applies to these facts. The explanation should
include both the facts/analysis that support Quale’s recovery of damages and the
facts/analysis that provide a potential defense for Farmer.

 Communicate in a conversational but professional tone.

149
Part E
Briefs and Oral Argument

150
20
Elements of a Brief

Changes From the Sixth Edition

In the sixth edition, this chapter only briefly mentioned the statement of facts for a
brief, which was covered separately in Chapter 26. The material from Chapter 26 has been
streamlined and incorporated into this chapter, to allow a more unified approach to teaching the
elements of a brief. The elements are still broken out into separate sections, however, to allow
teachers to present this material in a different order if they prefer.

Teaching Notes

As with the elements of a memorandum, the elements of a brief are perhaps best
introduced with a short lecture. This can be an opportunity to review the hierarchy and
jurisdiction of courts (discussed in Chapter 2), and to briefly explain the difference between
trial and appellate court briefs and how each fits into the process and procedure of the judicial
system. These differences are explored in more detail in Chapter 23 (Write to Your Audience).
There are at least two ways to teach the elements of a brief. The first is based on the
elements contained in this chapter. The second is to present the elements defined in this chapter
in the context of the relevant court rules in a particular jurisdiction. Many court rules
specifically identify the elements of a brief and the order in which those elements must appear.
The first approach is simple and straightforward. An advantage of the second approach is that it
prepares students for what they will have to do in practice.
Some teachers find it helpful, when introducing briefs, to list the elements of a memo
on the board alongside the elements of a brief, and then compare and contrast individual
elements in each column. This works particularly well with trial court briefs. (The chart on
page 336 in the text compares the elements of trial and appellate briefs with one another.)
Appendixes E, F, H, and I in the text are very helpful in teaching the elements of a
brief. First, they show entire briefs to trial and appellate courts using the format described in
this chapter. Second, they show how opposing sides might handle each element and thus
provide a basis for more detailed study of individual elements. This is particularly helpful for
elements that are not separately addressed in detail (e.g., Questions Presented, Standard of
Review).
Some professors have used the following technique to demonstrate the principles
discussed in this part of the text. Your students are probably working on a persuasive writing
project of some kind while you are teaching these chapters, often a trial brief and then an
appellate brief. Regardless of the writing assignment that your students are doing, consider
obtaining copies of the briefs filed by the parties in one of the cases that your students are using
for their assignment. If, for example, they are working on an assignment involving a
defendant’s Miranda9 rights, obtain copies of the principal briefs in the case and select
examples from each one for comparison. Study the examples from the case as you reach each
new chapter in the text. For example, in connection with Chapter 21 (Structure of an
Argument), compare the parties’ argument on a certain point and analyze their handling of a

9
Miranda v. Arizona, 384 U.S. 436 (1966).

151
key case or compare the point headings for that issue. You can also consider the differences
between the parties’ description of a particular incident in the Statement of Facts. By using a
familiar case that is relevant to their assignment, you can show students how the principles
described in the text are actually used—or perhaps not used—in a context that is immediately
relevant to their own work.
This chapter can also be reinforced with exercises similar to those suggested in Chapter
22 (Structure of the Argument): return to an objective memorandum that you assigned earlier
in the course and draft several versions of the questions presented or statement of facts. One
approach is to convert an objective question presented or statement of facts to a persuasive one.
Begin with a fairly good student-written example from the objective memorandum and have
students work in small groups to draft new questions or fact statements from the perspective of
one of the parties in the case.

Suggested Approach to Exercises

Exercise 20-A

Shannon’s Statement of Facts will be almost a mirror image of Magaro’s, which is


described in the text. Shannon will emphasize its view of the facts and emphasize facts that
Magaro seeks to downplay, while attempting to neutralize the facts that Magaro emphasizes.
From Shannon’s point of view, this is a story about a good company that sets high
standards for itself and respects its employees. Shannon will therefore want to emphasize its
prompt and effective response when it became aware of Dudley’s unacceptable conduct and its
repeated offers to rehire Magaro. Shannon will also want to draw attention to the role that
Magaro played in creating and perpetuating the problem that Shannon is now trying to rectify:
Magaro failed to report Dudley’s conduct until after she resigned, giving Shannon no
opportunity to intervene earlier, and she has refused Shannon’s offers of reemployment.
Shannon can emphasize its favorable emotional facts by focusing on the company’s
ethical conduct and concern for the welfare of its employees. For example, Shannon promptly
investigated Magaro’s complaints as soon as it became aware of them an took a “zero-
tolerance” approach to conduct that it considered immoral, even though Dudley was a
satisfactory employee who had a longer history with the company than Magaro did. Shannon
will also emphasize its refusal to discharge an innocent employee in order to meet Magaro’s
demands.
Shannon can neutralize Magaro’s emotional facts by telling the whole story and
describing the facts that Magaro will downplay in sufficient detail. Magaro was emotionally
and romantically involved with Dudley – a married man – for approximately one year. Within
four months of the affair’s end, she had resigned and sued Shannon, without ever lodging a
complaint about Dudley’s behavior or otherwise giving the company an opportunity to resolve
the situation. Moreover, although she is a single mother with no savings and two children to
support, she has refused multiple employment offers from Shannon because the position
offered is at a lower level than the job she left. Shannon will want the facts to raise questions
about Magaro’s motives and lead the reader to conclude that it would be unfair to hold
Shannon responsible for difficulties that Magaro created and exacerbated herself.
A chronological organization of the facts might work very effectively for Shannon. A
chronological organization will take the reader through both “sides” of the story as it actually
unfolded, allowing the reader to see the stark contrast between what was happening and how
little Shannon actually knew at the time. This approach will help the court to see that Shannon

152
was prevented from taking prompt and effective action that would have made Magaro’s
resignation unnecessary. This approach will also allow Shannon’s attorney to highlight the
weakness of Magaro’s position on the conciliation issue that is at the heart of the Knowles
decision.

Exercise 20-B

Because the problem requires a choice between two legal rules, the legally significant
facts involve the applicability of each rule. Under the Nokavich case, parties are bound by the
terms of their settlement. Thus, the terms and circumstances of the settlement agreement
between Ellen Brummer and Ivan Pearce are legally significant. The exact language
concerning “bodily injuries, known and unknown, and which have resulted or may in the future
develop” is particularly important. The circumstances would include the fact of the accident,
who was involved, and the eventual settlement.
Under the Brooks case, a party can open a personal injury settlement for an injury of
which neither party was aware when signing the settlement. Legally significant facts would
include the specific injuries Nancy Brummer suffered that were known when the agreement
was signed (facial lacerations, chip fracture in her nasal bones), the subsequent
electroencephalogram showing her severe brain damage, and the special help or treatment she
will require as a result of her brain damage (for her potential seizures, for her impaired reading
and hand-eye coordination, and for her education).
Emotionally favorable facts for Brummer include her promising musical career and
details about the injury. Emotionally favorable facts for Pearce include details about what Ellen
Brummer could have learned about Nancy’s condition before signing the settlement, and her
apparent understanding of what she signed. Important procedural facts include Ellen
Brummer’s attempt to persuade the trial court to reopen the settlement and the trial court’s
refusal to do so.
In outline form, the Statement of Facts for Brummer should begin by describing twelve-
year-old Nancy Brummer as a promising violinist. The specific details of the accident and the
original settlement will follow. Then the subsequent test, coupled with her injuries and required
care and treatment, should be described. The statement should end with an explanation of Ellen
Brummer’s unsuccessful effort to reopen the settlement.
In outline form, the Statement of Facts for Pearce should begin with the accident and
settlement. The details of the settlement would follow, including the fact that Ellen Brummer
read the settlement before signing it. The statement would then describe the subsequent test,
the results, and the care and treatment that will be required. It would include the doctor’s
statement that an electroencephalogram immediately after the accident would have disclosed
the problem. It would conclude with the trial court’s refusal to reopen the settlement and the
court’s statement that a settlement means what it says.
A teacher could contrast the two statements in several ways. One is to show students
copies of opposing statements written by other students.

Additional Exercise

Students often draft their Statement of Facts before their argument is fully developed.
It can be easier to begin the writing process with the Facts because telling a “story” feels more
familiar, and students may believe that they already know what is important. Experienced
lawyers understand, however, that a Statement of Facts should be written – or at least

153
substantially revised – after the Argument is complete. Only then does the writer fully
appreciate which facts matter the most, which must be emphasized, which must be de-
emphasized, and which organizational approach allows you to tell your client’s story most
persuasively. For this reason, it can be very effective to have students draft a Statement of
Facts after exploring a problem in depth. The following exercise is based on a problem that is
introduced in Exercise 21-B. If you have time, you can assign Exercise 21-B when you work
through that chapter, then use the following additional exercise to revisit the Statement of
Facts.

Exercise 20-C

This exercise is based on the same facts and law as Exercises21-B. The legally
significant facts in this problem include all facts that are relevant to the statute’s requirement
that drivers drive on the right side of the road and the exception that comes into play when an
obstruction requires the driver to drive on the left side of the road. These include facts that are
analogous to and distinguishable from the facts in the precedent cases. (E.g., the fact that
Hardy swerved into the left lane to avoid the potential risk that the truck would go over the
embankment or crash and explode is analogous to the facts in James; the facts regarding the
pumpkin spill are distinguishable from the bumpy pavement in Yerrick, and the alert,
experienced hardy is distinguishable from the drunk driver in Merkhof). Other legally relevant
facts include the cargo being carried, the characteristics of the right shoulder of the road and
the steep drop-off, the driver’s experience, the time span involved and the fact that Hansen’s
car was not visible, and the fact that some cars drove over unbroken pumpkins without harm
after the incident. Facts that establish Hansen’s injuries are also relevant. Hansen’s attorney
will emphasize the facts that focus on the basic rule, while Tri-State’s attorney will focus on
the facts that support the position that the exception came into play in this case.
Emotional facts that favor Hansen include the nature and extent of his injuries and the
fact that cars were able to navigate the spilled pumpkins without difficulty. The suddenness of
the accident is also something that Hansen might focus on, to emphasize his inability to take
any preventive action to avoid injury. In contrast, Tri-State’s attorney will emphasize the need
for an experienced driver to make a split-second decision to avoid potentially catastrophic
harm. The attorney will also point out that cars successfully navigated the road after the
accident; it was Hardy who was directly behind the pickup truck when the spill occurred
immediately in front of her.
The procedural facts are simple: Hansen sued, the trial court found that Tri-State was
negligent per se, and Tri-State has appealed.

1. Tri-State’s attorney will want the reader to feel what Hardy felt as she was confronted
with the need to make a split-second decision while transporting highly dangerous
cargo. The Facts, therefore, will be drafted to keep the reader in the moment, so the
reader can experience what Hardy experienced. The facts might begin by describing
Hardy’s years of experience, the dangerous load with which she was entrusted, and the
condition of the road itself – the truck in front of her, the narrow right shoulder, the
steep drop-off. The crisis she faced when the pumpkins spilled in front of her would
follow, including the risks of each choice she had. The fact that she could not see any
drivers in the left lane when she swerved is also important. The Facts would end with
the trial court’s conclusion that Tri-State is negligent per se.

154
2. The Statement of Facts for Hansen will probably be shorter than the facts for Tri-State.
Hansen’s attorney will want to emphasize the suddenness of the accident and nature
and extent of Hansen’s injuries. The fact that cars were able to navigate the spilled
pumpkins without any problem will be emphasized to create a contrast with Hardy’s
decision and create the impression that Hardy exercised poor judgment and panicked
unnecessarily. The facts will end with the trial court’s determination that Tri-State was
negligent per se and Hansen entitled to damages for his injuries.

155
21
Structure of an Argument

Changes From the Sixth Edition

This chapter now consolidates the material that was previously in Chapter 24 (Structure
of an Argument) and Chapter 25 (Point Headings). Streamlining and consolidating this
material allows the structure of the argument to be taught in the context of developing point
headings and emphasizes the integral role point headings play in both defining and organizing
an argument.

Teaching Notes

Exercises 21-A and 21-B involve very similar facts and law. If you need a simple
exercise, Exercise 21-A will suit your needs. If you want a more challenging exercise, Exercise
21-B is a better choice. Most teachers chose one exercise or the other, depending on their
needs. If you have more time, however, Exercises 21-A and 21-B can be taught in sequence.
The following notes will help you decide how to use these exercises.
Exercise 21-A is a good starting point for constructing arguments as to why the law
supports your position. The issue is easy to formulate: Do Jones’s actions fit within the
judicially created exception to the statute? Exercise 21-B is more complicated because it
requires students to progress from the broad issue to narrower issues, as illustrated by the
inverted pyramid below:

Negligent per se?

Violation of statute?

Action within exception?

Obstruction?

Necessity?

Exercise 21-B thus presents more of a challenge regarding how to present the law and
the facts most favorably for your client. Taken together, the two exercises illustrate how the
nature of the relevant law often dictates how you organize and present your arguments.
If you use both exercises, encourage your students to identify the specific question
posed by each exercise. Notwithstanding the similar law and facts, the question in 21-A is not
whether Jones violated the statute but whether the exception applies. In 21-B, however, the
question is whether Hardy violated the statute.

156
A useful exercise to focus on point headings is to have students turn the Questions
Presented in an office memo they have written into point headings for a brief. To illustrate the
differences in objective and persuasive writing, you might ask questions such as: Would you
change the order of the issues? Are there any issues you would not include because the
argument would be weak? How would you change the description of the issues and significant
facts? To continue this exercise, assign half your class to write for each party in the case. You
can then discuss how differences in emphasis, organization and word choice can be used to
make a point heading more persuasive.

Suggested Approach to Exercises

Exercise 21-A

The factual and legal materials used for this exercise are also used in Exercise 22-A.
This exercise works extremely well in classroom discussion as an introduction to
advocacy. Some of us outline the argument for each side on the board as we go through class,
so that students can see the competing arguments side by side at the end.
This exercise also works well as an exercise in synthesizing cases. Basically, each side
is obliged to interpret the cases in a way that supports its position. Although the statutory rule
contains no exceptions, and the earliest (Cain) case said there could be no exceptions, the court
of appeals has created an exception. The problem involves a choice of law (the rule or the
exception) and a question of how the exception should be defined.
You might ask the following questions to develop each side’s argument: Will your side
rely on the exception or the rule? What is the legal basis for the exception/rule? How would
you say that law applies here? What counterargument will your opponent make? What law will
your opponent rely on? What is the source of that law? What is your answer? At the end, you
might get something like this:

Jones State
The charge should be dismissed. The The charge should not be dismissed. The
public necessity exception (McKinney, statute specifically requires drivers to
Gordon dicta) applies here because keep their trucks in the right lane.
Jones avoided a potentially serious
accident with his fuel truck when he Gordon and the rationale in Cain are
swerved to avoid hitting an unexpected applicable.
ice patch.
Jones swerved into left lane because he
Because this was a real emergency, was trying to protect valuable fuel, and
Gordon is inapplicable. thus Shoop applies.

This is not about financial self-interest, There was no emergency here because
and Shoop is thus not applicable. Jones should have been watching for ice
patches. McKinney and Gordon dicta do
Cain does not apply because the law has not apply.
changed.

157
Notice the marginal value of Cain, even to the state. Although it said there could be no
exceptions, the court has subsequently created exceptions.
This problem can also be approached in more complicated ways. One teacher, for
example, creates vertical columns for each of the four cases as well as this case. On the left
side of this chart, the teacher writes these factors: danger without lane change, danger with lane
change, type of maneuver, whether the condition was expected or not, whether actual harm
occurred, and whether the defendant’s conviction was upheld. Each case is then analyzed for
these factors. The teacher uses this approach to get a clear understanding of which cases
provide the strongest basis for each side’s position, although the result is about the same as that
described above.

Exercise 21-B

The factual and legal materials used for this exercise are also used in Exercises 22-B,
24-B through -E, and in an additional exercise for Chapter 20, Exercise 20-C.

1. For Tri-State:

(a) Tri-State’s counsel would emphasize the exception. Hardy violated the general
rule by driving on the left side of the road, but Tri-State will argue that she fits
within the exception because an obstruction made it necessary for her to swerve
into the left lane.

(b) Tri-State’s actions fell within the statutory provision that drivers may drive in the
left lane to avoid an obstruction because pumpkins falling from a truck
immediately ahead of Tri-State’s driver created an obstruction.

(c) (1) The pumpkins were an obstruction. There were at least eleven of them, each
weighing from fifteen to twenty pounds, and they fell off a truck immediately
ahead of the Tri-State truck.

(2) The obstruction made it necessary to swerve into the left lane. Hitting the
pumpkins may have caused the fuel tanker truck to go out of control, crash, and
explode. There was a sharp embankment on the right side. Thus, going into the
left lane was the only reasonable alternative to endangering the lives of both
drivers.

[Note that the arguments track the relevant statutory language: (1) an obstruction
(2) making it necessary.]

(d) The counterargument will be that there was no obstruction because several cars
ran over the pumpkins without incident. The refutation is that the pumpkins may
have presented an obstruction to trucks but not to cars. [Several years ago, one
of us had a student who had driven trucks for several years. He told the class that
he would never drive over pumpkins that large if he could avoid it, because of
the danger of the truck skidding out of control.] Also, the Tri-State driver had to
make an immediate decision because the pumpkins spilled out suddenly. As an
experienced driver, she was in the best position to know what action she should

158
take to avoid possible loss of life. [That she had to make an immediate decision
and did not see Hansen’s car also helps to refute the argument that driving into
the left lane and possibly hitting another vehicle was a more dangerous course of
action than hitting the pumpkins.]

(e) Students should outline the argument based on answers to the previous questions.

2. For Hansen:

(a) Hansen’s counsel will emphasize the general rule to show that Tri-State’s
driver was negligent per se.

(b) Tri-State was negligent per se because its driver violated section 98.01 by
driving a truck in the left lane when there was no obstruction in the right lane,
only some pumpkins which several cars subsequently ran over without incident.

(c) (1) The pumpkins were not an obstruction. Some smashed upon impact. Several
cars hit unbroken pumpkins and continued moving forward.

(2) Swerving into the left lane was not necessary. Even if the pumpkins were
considered an obstruction so that it was dangerous to drive over them, the truck
driver created a greater danger by moving into the path of oncoming traffic.

(d) The Tri-State driver had a cargo of highly flammable fuel and had to make an
immediate decision about three dangerous alternatives: (1) to continue on the
right side of the roadway, run over the pumpkins, and risk losing control of the
truck, with the possibility that the truck would crash and explode; (2) to pull
over to the right side and risk falling down the embankment; (3) to swerve to the
left and risk colliding with another vehicle. She chose the alternative with the
least chance of endangering lives. The refutation is that because the pumpkins
were not an obstruction that prevented safe passage, Tri-State’s driver misjudged
the situation and made an ill-considered choice. She should have continued on
the right side of the road and avoided endangering persons properly traveling in
the left lane (the class of persons the statute is designed to protect).

[Some students make assumptions—e.g., that Hardy was following too closely—and
then state them as facts. This gives you an opportunity to explain the difference between
facts reasonably inferred from those given and mere speculation. You might also ask
students what facts missing from this exercise they would try to discover in a real case.
For example: How closely was Hardy following the pickup truck? If she was following
from a safe distance, could she have avoided the pumpkins by stopping? Were there
vehicles behind her that would have been endangered by her sudden stop?]

(e) Students should outline the argument based on their answers to the previous
questions.

The following chart summarizes the arguments for each side and incorporates the case
law and policy considerations relevant to the question in Exercise 21-B.

159
Tri-State Hansen
Thesis: Hardy’s actions fell within the statutory Thesis: Hardy violated the statute and thus was
provision that drivers may drive in the left lane negligent per se.
when necessary to avoid an obstruction.
Language of statute Language of statute
Obstruction No obstruction

15-20 lb. pumpkins fell from truck Some pumpkins smashed on impact
immediately ahead of Hardy
Several cars ran over pumpkins without incident
Pumpkins are slippery when squashed
Similar to Yerrick, where there was only
Thus, they are an obstruction to trucks a bumpy road
though not necessarily to cars, several of
which ran over the pumpkins Different from James, where there was an
actual “obstruction,” a child
Similar to James, where driver swerved
because of danger to child if he continued in Policy: if pumpkins are considered an
the right lane obstruction in this case, the rule would
be swallowed up with exceptions
Different from Yerrick, where there was
no obstruction, only a bumpy and uneven
road
Necessity No necessity

Suddenness of latch opening created Even if the pumpkins were an obstruction,


situation where Hardy had to make an Hardy created a greater danger by swerving–
instant decision truck more likely to explode from hitting
another vehicle than by running over
Flammable oil in truck could explode if pumpkins
Hardy lost control of truck
Unlike situation in James, where driver
Policy: implicit in James, where driver swerved to avoid hitting a child, Hardy
swerved to avoid hitting a child, is that a swerved and thus endangered motorist driving
driver should choose the least dangerous in the proper lane
alternative–the policy behind the exception in
the statute As an experienced driver, should have been
able to control the truck and handle the
Sharp drop on right created danger of situation properly
truck falling down embankment
Swerving to avoid pumpkins is similar to
Experienced driver unaware of oncoming Yerrick (driver seeking to avoid damage to
car– made the best decision under the car) where there was no necessity
circumstances
Policy: purpose of statute is to protect drivers
on the left side of the lane
Conclusion: compliance with statute, so Conclusion: violation of statute, so
not negligent per se negligence per se

160
2. If you assign this exercise as homework, you might ask students to email their answers
to you before class or to bring a hard copy of their work to class. You can then use the
computer or document reader to read and evaluate a selection of answers. In the
alternative, this exercise works well as an in-class project. Assign half the class to
represent each party and have students work in small groups to draft point headings for
their client’s brief on appeal. When the drafts are done, you can evaluate and discuss
each group’s work. You can look first at all of the plaintiff’s headings and then all of the
defendant’s headings, but consider also looking at the headings in pairs – one set of
plaintiff’s headings and one set of defendant’s headings – to highlight the differences in
the major and minor points that each side will emphasize.

The following headings both track the relevant statutory language: (1) an obstruction, (2)
making it necessary for the driver to swerve into the left lane. Hansen’s counsel,
however, will emphasize the first point and argue the second in the alternative, while
Tri-State’s counsel will treat the first point as a threshold issue and emphasize the
second point.

Sample point headings for Hansen:

THE TRIAL COURT PROPERLY FOUND THAT TRI-STATE WAS


NEGLIGENT PER SE.

A. When cars are able to drive over pumpkins in the roadway


without incident, the pumpkins do not create an obstruction in the
road for a tanker truck.
B. The presence of pumpkins in the roadway do not require the
driver of a tanker truck to swerve into oncoming traffic because
the pumpkins are not likely to prevent safe passage of the truck.

Sample point headings for Tri-State:

THE TRIAL COURT ERRED WHEN IT FOUND THAT TRI-STATE


WAS NEGLIGENT PER SE.

C. An obstruction in the road is created when a truck immediately in


front of a driver spills more than a dozen large pumpkins directly
into the driver’s path,
D. The experienced driver of a tanker truck loaded with highly
flammable oil is not negligent when she is forced to drive on the
left side of the road to avoid the obstruction created by the
pumpkins in her path.
1. The tanker truck could not avoid the obstruction by
driving on the right shoulder of the road because the
shoulder was very narrow and dropped off steeply,
creating a risk that the truck would explode.

161
2. The tanker truck did not have time to stop safely in the
right lane before hitting the obstruction.

3. The risk of explosion if the driver of the tanker truck hit


the obstruction and lost control of the truck required the
driver to drive on the left to avoid the obstruction.

Exercise 21-C

Some students identify a threshold issue concerning whether Hinkle’s injury was self-
inflicted. Because self-infliction is not part of the quoted statutory language, and because the
text emphasizes the importance of quoting the language at issue, other students will not
identify this as a threshold issue. If students raise this as an issue, you can discuss it as the most
significant one for the Commission. If the Commission can prove that the injury was self-
inflicted, it wins the case and subsequent arguments about worker’s compensation coverage are
irrelevant. Hinkle, on the other hand, would have to argue that (1) the injury was not self-
inflicted, and (2) even if it was, the term should not include accidental and unintended injury to
oneself during horseplay. Both parties would place this point first regardless of its strength or
significance, because logic demands that a threshold issue be addressed before issues that
depend on its resolution. Thus, the outline of points for each side would be divided into two:

I. The injury was/was not self-inflicted.


II. The injury did/did not arise out of or in the course of Hinkle’s
employment.

One way to teach this problem is to assign Hinkle’s side to half the class and the
Commission’s side to the other half. Then ask each student to turn in point headings for their
client’s argument. A teacher can then copy (or use an overhead projector) to show selected
student-written point headings, ask students which point headings they prefer and why, and
identify good and bad approaches.
Below are sample point headings for the second issue, two from each side, from a
classroom discussion. Although more than four answers were used in this discussion, these
four illustrate some basic points.
The point headings for both parties as to the second issue should follow the same basic
outline. Hinkle’s general statement that the Industrial Commission erred in denying his claim,
and the Commission’s general statement that it properly denied Hinkle’s claim, should each be
followed by specific assertions that correspond to the factors in the Sperry case. The specific
assertions show why the general assertion is true. They should include the reason for the
assertion and the key facts relevant to that assertion.

1. Point headings for Hinkle

H-1

HINKLE IS ENTITLED TO COMPENSATION FOR HIS EYE INJURY.

A. Shooting rubber bands was not a serious and lengthy departure from
Hinkle’s job because it was for a short period of time.

162
B. Shooting rubber bands was not a complete deviation from the
performance of the job because it was a short distraction.

C. Shooting rubber bands has become part of Hinkle’s employment because


it has happened a number of times.

D. The nature of Hinkle’s employment can be expected to include some


horseplay because of the ready availability of rubber bands.

H-2

THE INDUSTRIAL COMMISSION ERRED IN DENYING


HINKLE’S WORKER’S COMPENSATION BENEFITS BECAUSE
HIS INJURY AROSE “OUT OF OR IN THE COURSE OF” HIS
EMPLOYMENT.

A. Hinkle was injured in a minor and brief departure from his job.

B. Hinkle was injured while performing his regular work.

C. Hinkle was injured while engaged in recurring horseplay of which his


supervisor was aware.

D. Hinkle’s employment can be expected to include some horseplay


because of the ready availability of rubber bands.

2. Point headings for the Industrial Commission

I-1

THE TRIAL COURT WAS CORRECT IN


NOT ASSESSING AN AWARD FOR
HINKLE.

A. Hinkle should not receive benefits under Section 45 of the State


Worker’s Compensation Act because his injuries did not occur in the
normal course of his employment with Goff Medical Supply Co.

B. Hinkle does not meet the four factors necessary to establish a claim
under the Worker’s Compensation Act.

1. The injury in this case resulted from an extended amount of time


away from the work area.

2. There was completeness in deviation because the horseplay was


unrelated to his employment as a deliverer of medical supplies.
163
3. The “fights” had occurred on more than one occasion and the
employees had been warned against them in the past; thus, the
horseplay had become a regular occurrence.

4. Although the rubber bands may lend themselves to horseplay, an


eighteen-inch piece of wood hardly constitutes something readily
available for horseplay.

I-2

THE INDUSTRIAL COMMISSION PROPERLY


DENIED HINKLE’S WORKER’S COMPENSATION
BENEFITS.

A. Hinkle was injured during a serious deviation from his job.

B. Hinkle was injured during a complete deviation from his job because,
while he should have been loading a truck, he was engaged in a rubber
band fight with another employee.

C. Hinkle was injured while engaged in horseplay that his supervisor had
discouraged.
D. Horseplay cannot be expected to be part of Hinkle’s employment
because there is no evidence of lulls in activity.

You can begin by discussing which of the “H” point headings the students liked best.
Usually, this elicits strong support for “H-2.” Students’ answers generally tend to correspond to
the principles in the text.
Then ask about specific headings. Which main heading on Hinkle’s behalf is better?
Why? Explain that the heading under “H-2” is better because it frames the question in terms of
the Commission’s alleged error.
Then ask which subheadings under each are better. In “H-1,” students tend to prefer A
and D to the more vague B (“short distraction”) and C (“has happened a number of times”).
Use this example to demonstrate that the more vague the point headings are, the less
persuasive. In “H-2,” students tend to like all the subheadings but often cannot explain how A
differs from B. You can use this to focus on precision and clarity.
When asked about the “I” point headings, most students preferred “I-2.” Their reasons
were generally drawn from the principles in the text. Discuss how the main heading in I-2
differs from the main heading under “H-2” because it doesn’t contain any reason. The main
heading under “H-2” is better because it provides a general reason that connects the statutory
text to the four factors.
There are several problems with “I-1.” The main heading refers to the trial court when
it should refer to the Industrial Commission. Students need to understand that government
agencies often make the initial decision in these cases. In addition, use of the word “assessing”
is inappropriate. Subheading B.3., moreover, appears to argue Hinkle’s position. The main
heading and the subheading marked A should be combined and distilled; they are repetitive as
is. The sentence after B, moreover, is unnecessary. Some of the sentences under the headings

164
marked 1 through 4 are wordy and awkward. These problems should come out in class
discussion, and students gain confidence by successfully identifying problems in someone
else’s work.
Basing an exercise on student papers, of course, requires the teacher to be responsive to
the particular strengths and weaknesses of those papers. To do this properly, the teacher needs
to study the papers beforehand.
The division of the second issue into four subheadings is logical in light of the four
factors adopted in Sperry. Some of our students have thought it more logical, however,
to combine factor one (extent and seriousness of the deviation) with factor two (completeness
of the deviation). They see no difference between them except in degree. The difference is
fairly subtle. The first factor describes horseplay commingled with the performance of duties.
The second factor describes complete abandonment of duties so that the worker can
concentrate all his energies on the horseplay. As the deviation becomes more extensive and
more serious, the line between these two factors tends to blur. A lengthy and serious deviation
could be a complete abandonment.
As the example under principle 3 in the text illustrates, it is sometimes best to combine
two closely related points. The student should not make this decision until he or she has
analyzed the issues, outlined or drafted the argument, and considered the overall strategy of the
brief. Principle 3 suggests several questions you should ask students to consider in this
situation. For example: Are the arguments supporting each point so repetitive that they should
be combined? Are the arguments for each stronger if they are combined? Is it good strategy to
combine the two?

165
22
Persuasive Writing Techniques

Changes from the Sixth Edition

This was previously Chapter 27 (Persuasive Writing), but there are no major changes from text
in the sixth edition.

Teaching Notes

Teaching this chapter in conjunction with Chapter 21 (Structure of an Argument) is a


good way to illustrate the process of writing an argument: (1) Analyze the relevant law and
identify the arguments and counterarguments; (2) write an outline or chart showing a logical
and effective progression of points and counterpoints; (3) write a draft that includes all relevant
rules, facts, and policy under each point; (4) revise and edit, using the persuasive techniques
described in this chapter.

Exercise 22-A

The factual and legal materials used in this exercise are also used in Exercise 21-A.
Students should draft the arguments based on the outlines constructed in response to that
exercise. These types of responses are illustrated in the context of Exercise 22-B below.

Exercise 22-B

The factual and legal materials used in this exercise are also used in Exercises 21-B and 24-B
through -E. This exercise continues the problem presented in Exercises 21-B.

For Tri-State:

1. (1) The pumpkins were an obstruction. James illustrates that an obstruction can be
anything making it dangerous to continue in the right lane. Yerrick is distinguishable
because there was no obstruction precluding safe passage, merely a bumpy and uneven
stretch of road.

(2) The obstruction made it necessary to swerve into the left lane. In James the
court refused to interpret the statute so strictly as to force the driver to choose between
hitting a child and subjecting himself to a negligence suit by driving on the left side of
the road. The Tri-State driver faced a similar choice. In both cases, the drivers were
concerned with saving lives. By contrast, the driver in Yerrick was concerned only with
possible damage to his car.

2. To show obstruction:
 Size of pumpkins and that they are slippery when squashed

[Some students will say that the pumpkins were an obstruction or created a hazard.

166
Point out that these are conclusions, not facts.]

To show necessity:
 Suddenness of latch opening and pumpkins falling out
 Highly flammable oil in Tri-State truck
 Experienced driver
 Sharp drop on right side and extremely narrow shoulder
 Unaware of oncoming car

3. The policy implicit in James is that one should try to protect lives by choosing the least
dangerous alternative.

4. Students should draft the argument based on their answers to these questions and the
questions in Exercise 21-B. You might have students exchange their answers with
students representing Hansen.

For Hanson:

1. (1) Because Tri-State’s driver crossed into the left lane, the company violated the
statute and was negligent per se. Meekhof applied the general rule and stated its purpose
of protecting persons and property on the left side of the road.

(2) The pumpkins were not an obstruction. This case is similar to Yerrick, where the
obstruction was bumps and unevenness in the road. As in Yerrick, the driver could have
continued forward with some possible damage to the truck but with little risk of loss of
life. James is distinguishable because the “obstruction” in that case was a child.

(3) Swerving into the left lane was not necessary. As in Meekhof (driver intoxicated)
and Yerrick (driver seeking to avoid bumps in the road), it was not necessary for the
Tri-State driver to swerve into the left lane. This case presents a situation the exact
opposite of that in James. In James, the driver swerved to avoid killing a person
directly in front of the truck. In this case, Tri-State’s driver swerved to avoid pumpkins
and took a chance of killing or seriously injuring a motorist traveling in the proper lane.

2. To show no obstruction:
 Only eleven pumpkins recovered; others smashed on impact
 Cars hit pumpkins and did not go out of control
 Tri-State’s truck was large and heavy

To show no necessity:
 Experienced driver (could have controlled truck)
 Pickup driver pulled over onto shoulder (implying Tri-State driver could have done
the same)

To show policy of protecting persons on left side of road:


 Hansen severely injured

167
3. The purpose of the statute is to protect persons and property on the left side of the road.
Yerrick; Meekhof. If the pumpkins in this case are considered an obstruction, the rule
will be swallowed up with similar exceptions and the legislative purpose will be
avoided.

4. Students should draft the argument based on their answers to these questions and the
questions in Exercise 21-B. You might have students exchange their answers with
students representing Tri-State.

168
23
Write to Your Audience

Changes From the Sixth Edition

This chapter consolidates former Chapters 28 (Briefs to a Trial Court) and Chapter 29
(Briefs to an Appellate Court), and approaches writing from the standpoint of audience and
purpose. The chapter retains all the material that was presented in the sixth edition specific to
these particular audiences. This more unified approach allows teachers to teach these specific
principles of writing in the broader context of writing to communicate, instead of focusing only
on a single audience.

Teaching Notes

There are no additional teaching notes for this chapter.

Suggested Approach to Exercises Exercise 23-A

The legal and factual materials used for this exercise are also used in used in Exercises
23-B and 24-A, -C, and -D.
Because the problem and cases are discussed so extensively in the text, we will only
highlight some basic points here.
Aaron must argue that Raseen is applicable to this factual situation. The action for
contribution here is part of the plaintiff’s suit, not a separate action. Both Sarasota and Rollins
should be distinguished on this basis. Additionally, Aaron has clearly indicated that settlement
is not likely. These facts show that the two purposes behind the settlement-bar rule, avoiding
additional litigation and promoting settlements, are not applicable here. Specific facts from the
cases need to be compared in order to support these conclusions.
As part of the argument, Aaron should also challenge the overall fairness of the initial
settlement. The $1.5 million Chisolm settled for is but a small percentage of the $10 million
Miller seeks, even though the accident might have been prevented had Chisolm maintained the
plane properly. Financial responsibility for damages, Aaron might argue, should be more
evenly distributed. Raseen can be analogized and Sarasota distinguished on this point. Rollins
is also distinguishable in that both tortfeasors had agreed to their settlement amounts. Again, it
is important to contrast the specific facts of the cases. Even though Aaron has been offered the
same settlement that Chisolm agreed to, Aaron should not be forced to choose between a
settlement it does not believe is fair and potential liability for an amount that far exceeds its
responsibility in this case. Aaron should highlight Chisolm’s negligence in supporting this
point.
The argument should be brief. Students should not need more than three or four double-
spaced pages to make this argument.

Exercise 23-B

The legal and factual materials used for this exercise are also used in used in Exercises
23-A and 24-A, -C, and -D.

169
Because the problem is discussed so extensively in the text, we will only highlight
some basic points here.
Aaron’s brief, like Chisolm’s, should be drafted in view of the standard of review.
Aaron should subtly emphasize that even though the standard is de novo, the trial court’s
opinion should be accorded a great deal of weight because of the role of the trial court in the
fact-finding process.
Aaron would argue the analogies and distinctions of the Raseen, Sarasota, and Rollins
cases as it did in the trial court, but with more emphasis placed on the policies behind the rule.
Aaron’s position should be established as consistent with Raseen and the more recent trend in
the law.
Aaron should capitalize on the reasoning of the trial court’s opinion by showing that the
policies supporting the settlement bar rule would not be furthered here and that its application
is unfair in view of the specific facts of this case. Aaron would also include a more detailed
argument on the potential unfairness of the settlement-bar rule, and the impossible choice that
Aaron is faced with because Chisolm has settled. That is, Aaron must either accept the offered
settlement or risk potential liability for far more than its share of Miller’s damages. The Burns
article offers support for this argument, and students should use it. Students should also
characterize Aaron’s position as representative of the position any joint tortfeasor may face,
because appellate courts are concerned with the precedent this case will set as well as with the
outcome in this particular case.

Exercise 23-C

The factual and legal materials used for this exercise are also used in 8-B, 14-B, 17-A,
and 19-B. These comments should be read in conjunction with the suggested answers to
Exercise 8- B in this manual.
Farmer’s brief should focus on (1) the applicability of the legal rules because Farmer
has a credible argument whether the court applies the “substantial possibility of recovery”
standard or the “more likely than not” standard, and (2) the court’s policy that a physician
should not be held liable for a result that was a mere possibility. Quale’s brief should focus
more on (1) the applicability of the “more likely than not” standard in light of the expert’s
opinion regarding Andrew’s chance of survival, (2) the undoubtedly sympathetic facts of the
case, and (3) the injustice of not holding a physician liable for the death of a child when a
correct diagnosis and surgery could have saved him.

Exercise 23-D

The factual and legal materials used for this exercise are also used in Exercises 9-D and
14-E. Much of the analysis in this problem is shown in the answers to those exercises in this
manual. The answers here are in outline form.

For the League:

Because the League has the burden of showing that it is entitled to intervene, its
brief must show that it complies with all three elements of the rule. Although the
exercise asks students to draft the argument on only one element of the rule, you can
begin the exercise by reviewing the principles for drafting an argument in Chapter 21

170
(Structure of an Argument). Encourage students to think about the order of arguments
on each sub-issue. Which is the strongest?
We think the possible or actual inadequacy of representation by existing parties
is probably the strongest sub-issue, especially after the city attorney’s recent statement
that the city might be wrong to deny Swift’s proposal. Timeliness is probably next. The
last argument should be based on the possibility that the League will be bound; the
League is vulnerable to an argument that its interest is primarily ideological, but that
concern seems less compelling in light of the first and second arguments. Teachers may
want to discuss other aspects of advocacy here as well.
The principles in Part B (Briefs to a Trial Court) will also influence the drafting
of the argument. The League’s brief needs to show the trial court that the rule and the
case law clearly apply to the facts of this case. Thus, it will base its argument mostly on
the language of the rule and the facts of the Halsey case.
In arguing that the element of the possible or actual inadequacy of representation
by the city, the League’s brief would thus emphasize the fact that the city attorney’s
statement indicates not only that the city is not representing the League’s interest but
also that it is now on the side of rezoning. This goes beyond the minimum requirement,
established in Halsey, that a party must only show that existing representation “may be”
inadequate.
The argument must also show the League in a sympathetic light. Students should
understand that they bring in sympathetic facts while they make legal arguments, not
merely to show the League as sympathetic. Although this can be done most easily in
this case in the context of the sub-issue concerning whether the League may be bound
(by describing the League’s long involvement with this eighteen-square-block area and
the potential domino effect that approval of Swift’s application would have), a
sympathetic view of the League can also be developed in the context of the sub-issue
concerning adequacy of representation. The League engaged in negotiations with city,
state, and federal officials because the mayor said the League’s proposal was “worth
looking into,” and the mayor has repeatedly stated that City Hall “represents the people,
not narrow special interest groups.” These facts can be used to create a sharp contrast
between the League’s reliance on the mayor’s representations and the city’s apparent
sudden change of position. That contrast helps explain why the League can no longer
count on the city to adequately represent its interests.
The argument should also be brief. Students should be able to write the
argument in less than one double-spaced page. The brevity of the argument, of course,
makes it attractive as a teaching tool.
Students will not know who the judge is, and so the fourth principle is irrelevant.
Teachers might invent a judge with a particular disposition and ask students how this
judge’s disposition would influence the drafting of the brief.

For the City:

The element of the possible or actual inadequacy of the city’s representation is


its strongest argument against the League’s request to intervene. The city will argue that
League does not have an interest here that is sufficient to support intervention. This suit
is between a private landowner and the city. The League neither owns nor occupies
property in this area. Thus, it will not be “substantially affected” by a decision in this
case. Even assuming that League members have some interest as citizens, the interests

171
of its members are adequately represented by the city. The city has not indicated that it
will not adequately represent the League’s interest. On the contrary, the city has stated
all along that it will represent all of the people, which includes the interests of the
members of the League.

Exercise 23-E

The factual and legal materials used for this exercise are also used in Exercises 8-C, 14-
C, and 17-B. With regard to the organization of the issues in Greenleaf’s brief: According to
the rule that one places the strongest argument first, Greenleaf would begin with the argument
that Elliot unreasonably interfered with his right to the free flow of light because it is much
stronger than the argument that he has such a right. However, because the right to the free flow
of light is a threshold issue, we decided that logic trumped the relative persuasive value of the
arguments in this case.

1 /2. Each argument would be drafted along the lines suggested in the answers to Exercise 8-
C, as follows:

Outline of argument for Greenleaf:

(1) This state should recognize a legal right to the free flow of light.

a. The rule in Shover is broad enough to include the right to the free flow of
light. There is no meaningful difference between the right to lateral support
and the right to the flow of sunlight.

b. The rule in Cassells that there is no right to the free flow of light is not
binding and should be rejected.

(i) The rule as formulated does not reflect society’s interest in


encouraging the development of alternative energy sources. This
interest greatly outweighs policy reasons underlying the Cassells rule.

(ii) Cassells cites one case, Cohen v. Andrus, that found that a property
owner has a legal right to the free flow of light that can be protected in
a nuisance action. In Cohen the court emphasized the increasing need
for solar energy when it found that shade from defendant’s proposed
construction on adjacent property threatened to interfere with the
plaintiff’s solar collector. Greenleaf faces the same harm that the
plaintiff faced in Cohen. The rule in Cohen is more responsive to
modern concerns and interests than the rule in Cassells.

(iii) Cassells is distinguishable. The damage to Greenleaf is relatively


much greater than the damage suffered in Cassells. There, the hotel
owner anticipated potential economic loss and inconvenience to its
guests. Here, Greenleaf’s business might be destroyed because
potential customers can see that solar energy will not work if
neighbors can block the sunlight. Even if Greenleaf’s business

172
survives, the potential damage is like that in Cohen, where the loss of
sunlight threatened to interfere with plaintiff’s solar collector, and
similar to that in Shover, where the loss of lateral support “greatly
devalued” the property.

(2) Elliot unreasonably interfered with Greenleaf’s right to the free flow of light by
erecting billboards that block the sun from Greenleaf’s solar panels.

a. The interference is unreasonable because it is destroying Greenleaf’s


business. This case is analogous to Blum. In Blum, noise and vibrations
from a landfill owner’s trucks caused the conception rate of the sows on a
neighboring hog farm to decrease from 80% to 30%. The court held that
the landfill owner’s use of his land was unreasonable and a nuisance
because it essentially destroyed the hog farm. Like the trash-hauling trucks
in Blum, Elliot’s billboards are essentially destroying Greenleaf’s property
by making his promotional appeal less attractive and by increasing his cost
of doing business substantially.

b. In balancing the interests of the parties, the trial court could properly
conclude that Greenleaf’s interest in continuing his architectural business
and promoting solar energy outweighs Elliot’s interest in erecting
advertising billboards. First, as the Blum court makes clear, the balance
will always be struck in favor of the one whose business is being
destroyed. Second, Greenleaf’s business has more value to the community
than Elliot’s business. Greenleaf employs twelve people and is also
promoting a product that reduces environmental pollution.

Outline of argument for Elliot:

(1) There is no legal right to the free flow of light.

a. The rule in Cassells is the majority rule. Only one American court has ever
recognized a right to the flow of light. Shover is inapplicable because it
concerned the right to lateral support.

b. The reasons for the Cassells rule are valid. Requiring that every property
owner allow the free flow of sunlight onto neighboring property would
make building construction in high-density areas economically unfeasible.
It would also infringe on individual property rights.

c. The Cassells case is analogous. Both Greenleaf and the hotel owner in
Cassells suffered some economic damage and some inconvenience in
carrying on their respective businesses. As the Cassells court recognized,
such damage does not warrant recognizing a new legal right and
overturning a universally recognized rule.

d. Although the use of solar energy has increased, the fact that Cohen is the
only case that has recognized a legal right to the free flow of light indicates

173
that the potential harm to users of solar energy under the Cassells rule is
greatly outweighed by the economic harm and interference with property
rights that would result if the rule in Cohen were adopted.

(2) The billboards do not constitute a nuisance because they are a mere
inconvenience to Greenleaf, not an unreasonable interference with his
architectural business.

a. The interests of each landowner must be balanced. The consequences to


Elliot if he is required either to remove his billboards or to pay damages to
Greenleaf would be serious. It would amount to a partial destruction of his
business, a result similar to the result the Blum court called unreasonable.
Elliot cannot prevent the damage to Greenleaf without substantial cost.

b. The interference with Greenleaf’s solar panels is not unreasonable. It is not


a virtual destruction of his business, as was the case in Blum. Greenleaf’s
advertising promotion may have suffered and his utility bills may be
higher, but these are reasonable consequences of accommodating the
interests of both parties.

174
24
Preparing and Presenting an Oral Argument

Changes From the Sixth Edition

There are no major changes from the fifth edition.

Teaching Notes

The best way to learn about oral argument is to watch oral arguments by excellent – and
perhaps by not-so-excellent – attorneys and students. Many teachers begin teaching this subject
by providing their students with multiple opportunities to watch oral arguments of all kinds,
and there are countless ways to do that:

 Watch live arguments. Many law schools arrange court visits by their state’s
appellate or supreme courts. If your school does this, require your students to
attend. If your school does not do this, see if such a visit can be arranged, or arrange
for your students to attend arguments at the courthouse if that is feasible. Consider
giving your students a checklist of things to watch for, and/or having them write a
guided reflection after watching an argument. Debrief the experience together in
class.

 Take advantage of a wealth of online resources. Many state courts now stream their
oral arguments live, and/or videotape them and make them available for viewing.
These arguments can be poor, average, or excellent. Teachers can select two or
more arguments and ask students to discuss the strengths and weaknesses of the
advocates. Students can hear recordings of arguments in the United States Supreme
Court by searching the “cases” link on the Oyez Project website (www.oyez.org),
which has both recent arguments and recordings of older arguments in cases such as
Miranda v. Arizona and Brown v. Board of Education.

Arguments in the U.S. Supreme Court are not videotaped, but the internet is a rich
source for videos of oral argument in lower-level federal courts and in state courts.
Arguments in the federal circuit courts may concern high-profile issues that are
familiar to students, and they usually involve accomplished attorneys and active
questioning from the bench. Arguments in the courts of the state where your law
school is located can also be interesting. Students get to see how the process is
handled in the courts where they may one day practice, and may see some of their
future colleagues (some of whom may be law school alumni) at work.

If you find a couple sample arguments that you like, consider using them in
conjunction with this chapter, returning to them multiple times to test each
argument and oralist against the principles set out in the chapter.

175
Suggested Approach to Exercises

The exercises in this chapter are based on two different cases that students will have
encountered earlier in the course: Miller v. Aaron Industries (the Chisolm case) or Hansen v.
Tri-State Fuel Co. We suggest that teachers use all of the exercises that are based on one or
both of these problems to build the foundation for the exercises in this chapter. The exercises
for Chapter 24 are designed as a “capstone” experience: by working through an entire series of
exercises, students will explore a case in depth, from the outline or draft of a brief through the
preparation and delivery of a formal oral argument.
Here is a summary of the skills that students will have studied and practiced in the
context of these two problems:

Miller v. Aaron Industries

Chisolm’s case is developed as an in-text example in Chapter 23 (Write to Your


Audience). In Exercises 23-A and 23-B students are asked to prepare an outline of Aaron’s
argument to a trial court and to an appellate court.

Hansen v. Tri-State Fuel Co.

The factual and legal materials for this problem are presented in Exercises 21-B and 22-
B, and as an additional exercise in this teacher’s manual (Exercise 20-C). In each exercise,
students represent one of the parties, as chosen by the student or the professor. After
completing these exercises students will have:

 Identified their client’s strongest and most significant arguments and their
opponent’s best arguments (Chapter 21, Structure of an Argument);
 Drafted point headings for their argument (Chapter 21, Section D, Point Headings);
 Drafted an argument using persuasive writing techniques to present the law, facts,
and policy considerations in the light most favorable to their client’s position
(Chapter 22, Persuasive Writing Techniques); and perhaps
 Drafted a persuasive Statement of Facts that helps the court to see the case from
their client’s point of view (Chapter 20, Statement of Facts for a Brief).

If your students have worked through either of these problems, they will be well-prepared to
use the exercises in this chapter to prepare, practice, and deliver an oral argument for their
client.

Exercise 24-A

This exercise is included to provide an option for teachers who want to teach preparation
for oral argument by understanding your opponent’s position. Having worked through
Chisolm’s arguments and strategies in the text, this exercise gives students the chance to
explore the strengths and weaknesses of their opponent’s case by going through some of the
same steps. This exercise makes a good in-class exercise, and also lends itself well to small-
group work, as it allows students to debate the themes and arguments that best reflect the
strengths of Aaron Industries’ position.

176
To develop a theme, ask students to use one or more of the techniques described on
pages 429-430: write a 25-word summary of the case, draw a picture or cartoon, or write a
newspaper headline. Decide whether students can select the technique to use or whether you
will assign each student or group to use a particular technique. Consider asking students to use
at least two different techniques, or to develop another technique of their own. If students use a
graphic – a picture or cartoon – ask other students to translate the picture into words to see if
the theme is accurately communicated.

Exercise 24-B

This exercise is based on Hansen v. Tri-State Fuel Co., which is developed in Exercises
21-B and 22-B, as explained above. In preparation for Exercise 24-B, students should review
their answers and notes to the earlier exercises and use that material to draft a theme for their
client’s case and identify their strongest and most essential arguments. See Chapters 21 and 22
in this manual for suggested answers to the questions in those chapters.
As suggested above, ask students to use one or more of the techniques described on
pages 429-430 to develop a theme for their client’s case: write a 25-word summary of the case,
draw a picture or cartoon, or write a newspaper headline. You can decide whether students
should select the technique to use or whether you will assign each student or group to use a
particular technique. Consider asking students to use at least two different techniques, or to
develop another technique of their own. If students use a graphic – a picture or cartoon – ask
other students to translate the picture into words to see if the theme is accurately
communicated.
This exercise can be done as an in-class exercise, or you could assign the exercise to be
done as homework and then presented and discussed in class. As noted in the suggestions for
Exercise 24-A, this exercise works very well when students can collaborate with one another,
either in pairs or in small groups.

Exercise 24-C

This exercise gives students a chance to practice the skills described in several sections
of the text: “Select for presentation your strongest and most essential arguments,” (p. 431),
“Plan and practice your argument,” (p. 433), “Begin with a strong opening,” (p. 436), and
“Close with confidence” (p. 449).

1. The first part of this exercise requires students to draft an outline of their oral
argument. An argument should contain the following components:

Introduction See p. 436


Facts See p. 434
Argument See p. 431
Conclusion See p. 449

Some teachers provide their students with a sample oral argument outline or template;
others encourage students to develop their own outline. If your students have the
opportunity to watch some real oral arguments, either live or online, consider asking
them to outline the arguments of the oralists they observe and then evaluate the
effectiveness of each.

177
The challenge for students in this exercise is to select the issues that they will address in
oral argument, which requires them to prioritize their arguments and consider which
ones must be made in person to the court. Working off the outlines or drafts that they
have already prepared in earlier exercises, students should test each argument against
the considerations described on page 431 in the text to determine which arguments
should be made at oral argument and which can be left to the brief. (See the suggested
approaches to Exercises 23-A and 23-B in this manual for detailed descriptions of an
outline of the issues in Miller v. Aaron Industries (the Chisolm case). See the suggested
approaches to Exercises 21-B and 22-B for detailed descriptions of an outline and draft
of the issues in Hansen v. Tri-State Fuel Co.)

When students have selected their strongest and most essential arguments, have them
exchange their list of arguments with a student or group representing the opposing
party. Does the opponent’s list require them to reconsider their choices? If so, why?
Another approach is to ask students representing each party to present their list of
essential arguments. Compare the lists to see where they are the same and where they
are different. Ask students to explain and defend their choices, and encourage
discussion.

2. Students often have trouble imagining the questions that judges might ask them; when
asked to draft some sample question, they may draft easy questions with clear answers
that allow their client to win. To help students prepare for the questions that judges are
actually likely to ask, consider two sources.

First, review the arguments that you have not selected for oral argument. Students
might have made very good decisions about what issues to address at oral argument, but
a judge may still have questions about other issues raised in the briefs. Make a list of
those questions and be prepared to answer them if they come up.

Second, review the counter-arguments to the arguments that you have selected to
present at oral argument, and make sure that you have strong, compelling responses for
each counter-argument. Many students give too little attention to counter-arguments
when writing a brief and are ill-prepared to defend against them.

A Note About Exercises 24-D and 24-E

These questions are self-explanatory; they do not really have any suggested answers.
Instead, these two exercises are designed not just to help students practice and perfect their
substantive arguments, but to help them overcome their anxiety. Most adults are afraid of
public speaking, and law students are no different. Your students may be anywhere on the scale
between somewhat anxious and terrified at the thought of delivering an oral argument. If your
law school can tap into the resources of the university’s speech or theater department, or if you
have access to an executive coach, communications professionals can offer invaluable
feedback on a student’s oral communication skills. If you have such resources available to you,
consider requiring students to meet with someone who can assess their basic speaking skills
and provide suggestions for improvement. If not, the final two exercises will give your students
a chance to become more at ease while deepening their knowledge of their case. Some ideas for
teaching these exercises follow.

178
Exercise 24-D

Give students time to draft an opening and closing for their argument and to practice it.
This can be done outside of class or at the beginning of class. Then give students an
opportunity to practice in a safe environment. Some of us like to take our students to the
courtroom, where each is given a chance to become familiar with the room and practice
delivering his or her opening and closing at the podium. This can be done in pairs, in small
groups, or with the whole class. Other options are to have students videotape their opening and
closing and then review it with them in pairs or small groups, or to arrange for student practice
arguments with upper-level students who participate in your law school’s Moot Court program.

Exercise 24-E

The same techniques described above can be used for this exercise: students can work
in pairs, in small groups, in class or outside of class, live or on videotape. One of us uses an in-
class variation on this exercise that often turns out to be one of the most successful classes of
the year. In this variation, students send their questions to the professor two or three days
before class. The professor collects the questions and organizes them by party and legal issue –
editing and combining duplicate questions as necessary – and prepares a hand-out that includes
all of the questions submitted. Make an extra copy of the questions and cut them up, putting the
questions for each party in a bag.
On class day, divide the room in half and seat the parties on opposite sides of the room.
Students should sit with a partner. Let each team pick a question and give the class
approximately five minutes to plan an answer to the question they drew. When the time is up,
call on each pair to read their question, explain how they would prepare to answer this question
while preparing for oral argument (e.g. make sure you know the record in the case, review the
key facts of the major cases, etc.) and deliver their answer. Consider allowing a team from the
other side to do an impromptu response and allow the original team a 30-second rebuttal. Then
chose a team from the other side and repeat the process until every team has had a chance to
answer its question. At the end of class each student can take home a list of questions that he or
she can use for further practice.

179
SUPPLEMENTAL
MATERIAL

180
Course Planning
From its inception, A Practical Guide to Legal Writing & Legal Method was designed
to provide students with a comprehensive, yet straightforward, introduction to the unique
characteristics of legal analysis and writing. Although the book has expanded and grown to
reflect the changing needs of legal educators, students, and practitioners, the central purpose of
the book remains unchanged. The text has always taught students a step-by-step approach to
thinking and writing, presenting material in a carefully-sequenced series of lessons from the
introductory material in Part A through the complexities of appellate practice in Part E.
This approach allows teachers to use the text in virtually any course that teaches legal
thinking, reasoning, and writing. Whether you are a new or an experienced teacher, the
material in this section of the Teacher’s Manual will help you design a course in three steps:
(1) setting course goals and student learning objectives, (2) designing a syllabus, and (3)
developing tools for formative and summative assessment of student learning. Tools for
working through each step are provided below. We hope you find these examples useful as
you design your own unique course.

Course Goals and Learning Objectives

The accrediting body for American law schools, recognized by the U.S. Department of
Education, is the Council of the American Bar Association’s Section of Legal Education and
Admissions to the Bar. Beginning in the 2016/2017 academic year, the Council directed law
schools to implement the following new accreditation standard:

Standard 302. A law school shall establish learning outcomes that shall, at a
minimum, include competency in the following: (a) Knowledge and
understanding of substantive and procedural law; (b) Legal analysis and
reasoning, legal research, problem-solving, and written and oral communication
in the legal context; (c) Exercise of proper professional and ethical
responsibilities to clients and the legal system; and (d) Other professional skills
needed for competent and ethical participation as a member of the legal
profession. 10

Interpretation 302-1 of the standard explains that “other professional skills . . . may include
skills such as, interviewing, counseling, negotiation, fact development and analysis, trial
practice, document drafting, conflict resolution, organization and management of legal work,
collaboration, cultural competency, and self-evaluation.”
The skills highlighted in Standard 302 and its interpretation are familiar to every legal
writing teacher: they are the subject matter of our courses. And many teachers learned to be
explicit and transparent with their students – to tell them why we do what we do and what our
expectations are – long before the ABA required law schools to establish learning outcomes.
Now, however, law schools are required to publish their learning outcomes, and many require
faculty members to include learning outcomes on their syllabi.

10

https://www.americanbar.org/content/dam/aba/administrative/legal_education_and_admissions_to_the_bar/standa
rds/2020-2021/2020-21-aba-standards-and-rules-chapter3.pdf (accessed May 24, 2021).
181
Defining course goals and learning outcomes is a crucial first step in course design.
You must decide what you want to accomplish before you can design a course to accomplish it.
To begin, consider what you have to work with – e.g., are you teaching a 2-credit course or a 3-
credit course? How many students will you have? How many times per week will your class
meet? What does the rest of your – and your students’ – work load look like? Based on the
answers to those questions, what can you reasonably expect to accomplish during the course?
In light of your answers to these questions, draft your outcomes and objectives. You may find
some helpful ideas in the examples on pages 183 through 186 below.

182
Curricular Objectives for LRW I
(These detailed objectives were designed for a traditional 1L fall semester course.)

1. Students will understand the structure of both state and federal court systems and how
they function in the common law system of the United States.
2. Students will understand the doctrines of precedent and stare decisis and how they
function in the common law system of the United States.
3. Students will be able to read a case and prepare a case brief.
4. Students will be able to write a case summary, both generally and focused on a
particular point of law addressed in the case.
5. Students will be able to synthesize a group of cases as to how they address a single
point of law.
6. Students will be able to read and understand a statute.
7. Students will be able to synthesize a statute and a case or group of cases addressing a
single point of law.
8. Students will be able to write a synthesis of a statute and a case or group of cases as
they address a specific point of law.
9. Students will be able to write a summary of the law on a particular point.
10. Students will understand the principles of and be able to use analogical reasoning.
11. Students will be able objectively evaluate a set of facts.
12. Students will understand the overall organization of legal analysis.
13. Students will be able to produce a written legal analysis, applying the law to a set of
facts using the principles of analogical reasoning.
14. Students will be able to produce a specific legal document following a defined format.
14. Students will be able to write a statement of a formal legal issue (question presented).
15. Students will be able to write a clear, concise summary of a set of facts for the purpose
of defining the parameters of a written legal analysis (memo statement of facts).
16. Students will be able to write a clear a clear, concise summary of a legal analysis
(memo conclusion section).
17. Students will be able to write clearly, concisely, and professionally, using appropriate
grammar, punctuation, and formal usage conventions, observing the specific
conventions of legal writing, and adopting a tone appropriate for the intended audience.
___________________________________________________________________________________

183
Legal Research & Writing Course Objectives
(These objectives were designed for a two-semester first-year writing course.)

A. Analyze the legal problem(s) presented:


1. identify the significant issues
2. identify sub-issues
3. evaluate the relative significance of the various issues and sub-issues

B. Research the law:


1. understand what research sources are available and how to use them effectively
2. locate and analyze relevant statutes
3. locate and analyze relevant cases
4. locate and analyze relevant secondary authorities
5. use electronic resources effectively

C. Write an office memorandum, both summary-of-the-law and analytical:


1. locate and synthesize the various authorities
2. apply the law to the relevant facts
3. write clear, concise, objective statements of facts
4. write clear, concise, objective statements of the legal issues involved
5. organize the issues effectively
6. develop the discussion of each issue logically
7. evaluate the strengths and weaknesses of your client’s position
8. objectively answer the question(s) posed
9. follow appropriate format

D. Write an appellate brief:


1. locate and synthesize the various authorities
2. apply the law to the relevant facts
3. effectively deal with persuasive authority
4. write clear, concise, persuasive statements of facts
5. write clear, concise, persuasive statements of the legal issues involved
6. effectively organize persuasive arguments
7. present arguments persuasively
8. follow appropriate format

E. Communicate effectively in writing:


1. outline
2. revise
3. clarify ambiguities
4. eliminate unnecessary wordiness
5. write precisely
6. use legal terms correctly
7. use proper punctuation and grammar
8. use proper spelling
9. adopt the appropriate tone and point of view

F. Know and use the proper citation form

184
Learning Goals and Writing Outcomes for a Three-Term Course
(These objectives and final writing outcomes were designed for a three-trimester, 32-week
course. They could easily be revised for a traditional two-semester 1L course of 26-30 weeks.)

Course Objectives for Fall: By the end of the fall trimester students will:

1. Identify the types of legal authority, and describe the hierarchy of legal authorities.
2. Read and understand legal authorities.
3. Use proper legal citation form to cite primary and secondary authorities from a variety of
sources.
4. Synthesize cases and other legal authorities into a comprehensive description of the rules
and principles that govern a particular legal issue.
5. Define standard legal terms and use them properly.
6. Identify and understand the legally significant facts in a case.
7. Use legal authorities to reason by example and by analogy and draw a predictive
conclusion about the likely resolution of a legal question.
8. Use the IRAC/CREAC paradigm to organize a written discussion of a legal issue or issues.
9. Collaborate with peers to solve problems and provide effective feedback.
10. Assess, revise, and edit one’s own legal writing.
11. Practice good time and project management.
12. Access resources for self-help related to study skills, reading, and writing.
13. Use proper grammar, punctuation, and syntax in their written work.

Writing Outcome for Fall

At the end of the course, students will research and write an objective office memorandum to a
supervising attorney. The memorandum will provide the attorney with thorough and accurate
information that will enable him or her to provide competent advice and counsel to a client
regarding a client’s legal concerns.
________________________________________________________________

Course Objectives for Winter: By the end of the winter trimester students will:

1. Practice and improve skills introduced in the fall term.


2. Brief a statute, identifying the applicability/scope, general rule, exceptions, consequences,
enforcement mechanism(s), and purpose/policy (if stated).
3. Synthesize cases and other legal authorities into a comprehensive description of the rules
and principles that govern a particular legal issue.
4. Use proper legal citation form to cite a variety of primary and secondary authorities.
5. Use select rhetorical techniques to effectively frame legal issues and present a persuasive
argument on behalf of a client.
6. Use appropriate rhetorical techniques to de-emphasize the weakness in a client’s case and
to weaken an opponent’s position
7. Understand and make effective use of the differences between written and oral
communication in various legal contexts
8. Plan and present a formal oral argument on a motion to a trial court.
9. Explain the ethical rules that are particularly applicable to their work as legal researchers
and writers.

185
Writing Outcome for Winter

At the end of the course, students will write a brief on behalf of a client to a trial court judge.
The brief will seek to persuade the judge that the student’s interpretation and application of the
law to the facts of the client’s case is superior to the opposing side’s, therefore enabling the
judge to resolve the case in favor of the student’s client.
________________________________________________________________

Course Objectives for Spring: By the end of the spring trimester students will:

1. Practice and improve skills introduced in fall and winter.


2. Read examples of, and practice, advanced brief-writing and argument techniques.
3. Read examples and explain the real-world consequences of unprofessional legal research,
writing, and advocacy.
4. Prepare and deliver an oral presentation as part of a small group.
5. Plan and present a formal oral argument on appeal.
6. Work with a small group to complete a workplace simulation, prepare the requested work
product(s), and report on the completed project.

Writing Outcome for Spring

Students will write a brief to an appellate court on behalf of a client. The brief will comply
with the relevant appellate rules, and seek to persuade the appellate court that the student’s
interpretation and application of the law to the facts of the client’s case is superior to the
opposing side’s, therefore enabling the court to resolve the case in favor of the student’s client.
________________________________________________________________

186
Syllabus Design

A course syllabus can be simple or complex, general or specific, detailed and defined or
open-ended and flexible. In a legal writing course, however, every course syllabus must lead
students through a carefully-sequenced series of lessons that will enable students to complete
one or more written projects of the kind that they will produce in practice. This is true whether
you are teaching a one-credit introductory course for undergraduates or beginning law students
in their pre-1L summer, or a comprehensive three-term course. In order to provide
opportunities for writing, feedback, and rewriting, every legal writing syllabus must be
carefully constructed to keep students moving forward toward the ultimate writing goals of the
course.
Most law school writing courses also incorporate lessons in legal research, citation, and
writing techniques. Typical syllabi therefore often reflect a coordinated series of intertwined
subjects, all of which are essential to a successful professional career. The syllabi provided
here are edited to focus primarily on the use of the textbook; many additional readings,
activities, and projects have been omitted. Ultimately, your syllabus will incorporate additional
material that reflects your own course goals and teaching style.
Sample syllabi for a variety of courses follow on pages 188 through 193.

187
Intro to Legal Analysis & Writing
(A one-credit course taught in six 2.5-hour classes over three weeks,
suitable for a summer pre-law session or college short course.
Final assignment is the analysis section of an objective memorandum.)

Introduction to Law; Sources of Law; Reading and Understanding


Cases
Class 1
Dernbach Chapters 1-3
Read for Today
Case 1
Precedent & Stare Decisis; Reading and Understanding Statutes
Dernbach Chapters 4 & 5
Class 2
Read for Today Case 2
Statute
Understanding Legal Rules; Case Synthesis; Identifying Issues for
Analysis;
Class 3
Read for Today Dernbach Chapters 6 & 7
Read Case 3
Common Law Analysis; Statutory Analysis
Class 4
Read for Today Dernbach Chapters 8 & 9
Organizing a Legal Discussion; Describing and Applying the Law
Class 5
Read for Today Dernbach Chapters 11-13
Drafting the Analysis; Revising and Editing
Class 6 Read for Today Dernbach Chapters 14 & 15
Due [date & time] Analysis assignment

188
Legal Research & Writing
(A five-credit two-semester first-year course.)

Legal Research & Writing 1 – Fall Semester

Text: John C. Dernbach et al., A Practical Guide to Legal Writing & Legal Method (7th Ed.).

Week of Topic Reading Assignment

American Legal System; Stare decisis; Dernbach, Chap.1-5


Case briefing; Reading statutes

Case Synthesis; Analogization & legal reasoning; Dernbach, Chap. 6-8


Intro to organization ,

Statutory Analysis; More complex legal analysis Dernbach, Chap. 9-10

Organization of and drafting a written legal Dernbach, Chap. 11-14


analysis

Purpose & format of a Summary of the Dernbach, Chap. 16-17


Law Memo

Purpose & format of an Analytical memo Dernbach, Chap. 18


Statement of Facts & Questions Presented

Revision & editing Dernbach, Chap. 15

Opinion Letters Dernbach, Chap. 19

More practice on legal analysis, organization


& writing style

Course Requirements: Ungraded assignments must be turned in by the due date to receive
credit unless other arrangements are made. Your final grade will be based on satisfactory
completion of all requirements and the following graded assignments.

Assignment Due Dates


1. Analysis exercise
2. Summary of the Law Memo
3. Closed Memorandum
4. Opinion Letter
5. Open Memorandum

The Honor Code applies to all assignments. On some assignments you will be permitted to
work in groups, however the Open Memo must be completed without assistance. If you have
any questions about a particular assignment, please ask.

189
Legal Research and Writing 2 – Spring Semester

Due Dates for Major Assignments

1. You must successfully complete all training for using electronic resources by ________.

2. Persuasive Writing Assignment – [month/year] specific date TBA

3. Draft of argument -- Due in class [date].

4. Appellate Brief -- Due by 9 a.m. on [date].

5. Oral Argument -- two mandatory rounds of oral argument will be scheduled during early
month/year]; dates and times TBA. Competitive rounds will follow.

Reading Assignments

Week of: Topic Assignment

Intro to persuasive writing; the Dernbach, Ch. 23


appellate process; review of analysis of
legal issues & research methodology; use
of mandatory authority

Use of persuasive authority; structure Dernbach, Ch. 21


and organization of legal argument

Drafting the argument; revising & Dernbach, Ch. 22,


editing Review Ch. 15

Brief format; Questions presented; Dernbach, Ch. 21


statement of facts; point headings

More on persuasive organization and style

Specific stylistic tools for persuasive Bluebook


writing; editing; citation review Draft of argument due

Oral Argument Dernbach Ch. 24


____________________________________________________________________________

190
Legal Method & Communication
(A six-credit, three-term course to be offered in three trimesters or semesters.)

FALL TRIMESTER
Week Topics Assignments Writing Projects
Course Intro; Intro to Dernbach Intro. & Chs. 1-3
O Law; Briefing Cases; Dernbach Ex. 3-A
Read & brief Case 1
Precedent & Stare Dernbach Chs. 4 & 6, pp. 73-77 Common Law
1 Decisis; Legal Rules Dernbach Exs. 4-A, 6-A & B Analysis assigned
Read & brief Case 2
Rule Synthesis Dernbach rest of Ch. 6 & Ch. 7
2 Dernbach Exs. 6-C & D
Read & brief Case 3.
Common Law Analysis; Dernbach Chs. 8, 10, & 11 Common Law
3 Basic Organization Dernbach Ex. 8-A Analysis due betw.
Weeks 3 & 4
Statutory Analysis Dernbach Chs. 5 & 9, review Ch. 7 Statutory Analysis
4
Dernbach Exs. 9-A & B assigned
Describing & Applying Dernbach Chs. 12 & 13 Statutory Analysis
the Law rule, elements
5 chart, & case
briefing chart due
today
Drafting & Revising Dernbach Chs. 14 & 15
6
Dernbach Exs. 15-A & D
Conferences on Draft Draft due 48 hours before Statutory Analysis
7 conference. due betw. Weeks 7
& 8.
The Office Memo Dernbach Chs. 17 & 18, Apps. B & Summary of the
C Law & Analytical
8
Memo & interim
projects assigned
Project Management; Review Dernbach Ch. 18, parts B Research 1 due at
9 Questions Presented & &C beginning of class
Brief Answers
Statement of Facts Review Dernbach Ch. 18, part D Summary of Law
10 due at beginning of
class
Memo Q & A; Report-to- Facts due at
11
Partner conferences beginning of class
Substantive Revision Review Dernbach Chs. 12, 13, &
12
16
Editing & Proofreading Review Dernbach Ch. 15 Analytical Memo
13
due by [date]

191
WINTER TRIMESTER
Week Topics Assignments Writing Projects
Intro to Advocacy & Review Ch. 18 and Apps. B & C Trial Brief &
Persuasion; Dernbach Ch. 20 & Apps. E & F interim projects
1
Research Strategy Review Review Ch. 2 assigned
Read trial brief materials
The Trial Brief; Review Apps. E & F
2 Project Management; Dernbach Ch. 23, Part A
Skills Review Other readings as assigned
Stating Rules & Review Chs. 6 & 7 Research Log due
3 Identifying Issues; Dernbach Ch. 21 in class today
Structure of an Argument Dernbach Ex. 21-A (for discussion)
Using Point Headings; Review Ch. 21, Part D Annotated Outline
4
Trial Brief Q & A Prepare Dernbach Ex. 21-C due end of week
Conferences on Annotated
5 Outline & progress on
Argument Draft
Other Parts of the Trial Review Ch. 20, Parts J & N and Ch. Argument Draft
6 Brief; 23, Part A due end of week
Technical Requirements
Persuasive Writing; Dernbach Ch. 22
7
Substantive Revision Review Chs. 11-13
Conferences on Trial Review Ch. 15 Trial Brief Draft
Brief Draft; due at beginning of
8
Editing & Proofreading week; Trial Brief
due [date]
Arguing a Motion in the Dernbach Ch. 24
9
Trial Court
Oral Arguments Review Ch. 24 Oral Argument
10 delivered this
week.

192
SPRING TRIMESTER
Week Topics Assignments Writing Projects
The Appellate Process; Dernbach Ch. 19 and Appendix D Opinion Letter
1 Standard of Review; Read trial court order in our case
Opinion Letters
Opinion Letters, cont’d Opinion Letter due
2
Plain English betw. Weeks 2 & 3
Advocacy on Appeal; Dernbach Chs. 20 & 23, Part B, §§ Brief Template,
Formatting the Brief; 1 & 2, and Appendices G, H, & I New Research and
3
Legislative History Partial Draft
assigned
Administrative Research; Dernbach Ch. 24, Part B, § 1 Brief Template due
Thinking About Theme;
4
Hierarchy of Argument &
Common Fallacies
Making Effective Dernbach Ch. 23, Part B, §§ 3 & 4 New Research due
5 Arguments; Review Ch. 22, Part B, § 2
Policy Arguments
Effective Brief Writing: Partial Draft due
6 A Judge’s Perspective mid-week
Conferences
Other Requirements of the Review Dernbach Ch. 20
7
Brief
Writing & Editing Review Dernbach Chs. 15 & 22 Appellate Brief due
Review; on Saturday [date]
8
Brief Q & A;
Mini conferences
Oral Argument on Review Dernbach Ch. 24
Appeal: A Judge’s
9
Perspective; Practice
Arguments
Oral Arguments in lieu of
10
class

193
Skills Development and Assessment of Student Learning
The ABA’s Accreditation Standard 314 requires law schools to “utilize both formative
and summative assessment methods in its curriculum to measure and improve student learning
and provide meaningful feedback to students.”11 Interpretation 314-1 defines both terms. In
the context of a single course, formative assessments are “measurements at different points
during a particular course . . . that provide meaningful feedback to improve student learning;”
while summative assessments are “measurements at the culmination of a particular course . . .
that measure the degree of student learning.”12
This standard, has led most law schools to no longer base course grades on a single
cumulative final exam. Although the final exam remains, it may or may not be cumulative,
because law schools have increased their use of interim assessments. A mix of mid-term
exams, periodic quizzes, and practice essays – graded and ungraded – are now common at
many schools. -term exams and quizzes are frequent formative assessments in law school, and
final exams are the most common summative assessment. However, just as an athlete or
musician must practice and receive hands-on coaching to win at Wimbledon or play in the New
York Philharmonic, law students must practice and receive individualized coaching to develop
the skills they will need as lawyers. The assessment tools used in other classes have limited
value in a legal writing course because they cannot provide the kind of feedback that students
need in order to develop the skills we teach.
This teacher’s manual takes a broad view of student assessment. Like most legal writing
teachers, we focus is on proving meaningful feedback that will allow a student to improve –
both during the course, and after it ends. Our focus is therefore less on “formative” versus
“summative” assessment than on whether students can tell how they’re doing, both objectively
and subjectively – that is, both in relation to the expectations of their profession and in relation
to their peers. Our goal is to provide you, the teacher, with a variety of assessment tools that
you can use and adapt to meet the needs of your students. These tools are discussed in two
parts: “Skills Development” and “Rubrics and Checklists.” Although these sections align
fairly well with the ABA’s distinction between formative and summative assessment, many
professors use grading rubrics for both formative and summative assessments.

Skills Development

The chapter exercises help students practice the skills taught. The Chapter Notes in this
teacher’s manual provide teaching suggestions and model answers for each exercise. However,
student engagement increases when you use different approaches in class, and your own
satisfaction as a teacher will likely increase when you develop new ways to help your students
learn. Consider the following approaches as you develop your class plans.

Classroom discussion vs. hand-in assignment

Students can gain valuable insight into their own learning – and you can assess students’

11

https://www.americanbar.org/content/dam/aba/administrative/legal_education_and_admissions_to_the_bar/standa
rds/2020-2021/2020-21-aba-standards-and-rules-chapter3.pdf (accessed May 24, 2021).
12
Id.
194
developing mastery of concepts and skills – via in-class discussion of the exercises or review of
written assignments. The approach you use will likely change from class to class depending on
the complexity of the lesson, the size of your class and the time you have in the classroom, and
your priorities as a teacher.
Simple classroom discussion works well for many of the exercises; it is usually a fairly
good way to convey the basic points made in the chapter, and the dialogue (Socratic and
otherwise) that you have with a few students will be helpful to many of the others in the class.
The disadvantage is the one that occurs in any class: some students prepare and some do not.
One way to solve this problem is to require students to type their answers to the
exercises and turn them in after class. The paper will not be graded; it will merely be reviewed
to see whether the work was done. You (or your research assistant) can review the answers,
determine whether they are adequate, and place a mark in the grading book for those who did
not turn in the assignment or who turned in exceptionally poor work. If you return these papers
marked with a plus or a minus students will receive some very basic – but very important –
formative feedback: they are on the right track, or they are not.
The exercises can also be assigned as homework and handed in. Whether they are graded
or marked on a pass/fail basis, this is obviously a good way to ensure that students actually do
the exercises. Reviewing written assignments also allows you to benchmark common
problems, provide some individualized feedback, and identify students who may be struggling.
Requiring written answers is almost never an adequate substitute for classroom discussion of
the exercise, however. Students can learn a great deal from listening to the discussion of other
student’s approaches and your response to them. Reading and marking the papers can also
overwhelm you, particularly if you have large classes or give many assignments. Too many
written assignments can also overwhelm students.
To get the most out of the exercises, many teachers use a combination of class
discussion and written exercises, and manage the assignments in different ways. If an exercise
is straightforward and does not require a student to process multiple cases and factual scenarios,
in-class discussion can be very effective. If an exercise is more complex, however – e.g., if it
require students to create a chart, analyze cases and statutes, or draft a discussion or letter – a
combination of in-class discussion and individual written assignments is more likely to help
each student to develop those skills. The three options described below allow students not only
to receive feedback from you, but to also compare their understanding and preparation to that of
their peers.
The first option requires students to prepare a written answer to an exercise for
discussion in class. You can ask for volunteers or “cold call” students to present their work.
Allow students to incorporate new insights and ideas in handwritten notes on their own
assignments. Reviewing the assignments will enable you to assess each student’s individual
work and their understanding of the class discussion. Students will have the benefit of your
feedback and their own class notes when you return the papers. Another option, particularly for
lessons that you believe deserve additional emphasis, is to ask students to review an exercise for
in-class discussion when you assign a particular chapter. Have a classroom discussion of the
exercise, then require students to turn in written answers to the exercise sometime after class or
for the next class.
The third option is to assign one or more exercises as small group projects, encouraging
students to work collaboratively. This approach allows you to use the exercises to reinforce
students’ writing and analytical skills without overloading them, and it allows you to provide
meaningful feedback if you have a large number of students. Another collaborative approach is

195
to use a variation of the “think – pair – share” approach: (1) Think – ask all students to outline
their answer to an exercise before class. (2) Pair – when class begins, put students in pairs or
trios to compare answers with one another, then engage the whole class in a discussion of the
exercise. (3) Share – return students to their pairs or trios and ask each group to complete a
final answer to the exercise that they will turn in – i.e., “share” – with you for review.

Role-playing

Role-playing works extremely well for the exercises in Part E (Briefs and Oral
Argument). But it can also work well for other exercises, even exercises involving objectivity
and memo writing. (For example, the Chapter Notes above describe a role-playing approach to
Exercise 4-D, on precedent and stare decisis.)
The easiest way to take a role-playing approach to an exercise is to tell half the students
in the class to prepare as if they represented Party A in the exercise, and the other half to prepare
as if they represented Party B. When they get to class, we have them answer questions from
their party’s point of view. If a student representing Party A makes a certain point, we might ask
a student representing Party B to answer it. Then we might ask the Party A student to respond.
Role-playing can also include assigning the students to approach an exercise as if they are
preparing a memo for a judge on the case. This can encourage objectivity as the students are not
given the task of advocating for one outcome over the other.
Role-playing is a particularly effective way of making certain points that are difficult to
make in other ways. You might use it, for example, to emphasize objectivity in memo writing
by having students representing opposite sides assess the strength of their client’s position. If
you have used the judicial opinion approach, have the students who reached one conclusion
evaluate the analysis of the students who reach the opposite outcome. A similarity in views
about a particular party’s position would suggest objectivity. A substantial difference in views
would suggest lack of objectivity, or perhaps ambiguity in the law.
When students to take on different roles, critique the positions of others, and receive
critiques in return, they gradually become more self-aware. Their ability to see their own ideas
more objectively and to self-correct improves. Role-playing allows the teacher to set the stage
for student-generated learning. In our experience, students feel more engaged when they are
pretending to be someone’s attorney and recommending or advocating something – after all,
this is why they came to law school. Role-playing shows students how their class work is
preparing them for their careers, boosting motivation and making the classroom experience
more meaningful and enjoyable for both students and teacher.

Peer review

Peer review can be another effective means of formative assessment – for the student
reviewer as well as for the student whose work is being reviewed. Unless you are working with
an advanced class and/or with a self-selected group, however, keep the identity of the writers
confidential. Students are sensitive to criticism of their work. Some professors adopt a code
system for peer review projects, or collect the papers in advance and remove students’ names. If
you teach more than one section, consider using the other section’s writing for in-class
critiques, removing the names first.
Require students to give their exercise or draft paper to one or more other students at the
beginning of class. They can critique the other student papers in small groups, or you can have
students talk about other student papers in a discussion setting. Peer pressure is often (although
196
not always) a means of improving performance. We have found that students are very
interested in the work done by their peers. To keep the discussion focused, you should identify
the criteria you expect students to use (e.g., the organizational principles in Chapter 11, or the
“Revising Your Draft” steps in Chapter 15), or give them a checklist of tasks to accomplish
during the review process. To insure that students get meaningful feedback, ask students to
identify one specific thing that the writer did well and then explain why that task or step was
well-done. Then ask students to identify one area where the writer needs to improve and
explain how the writer could do so.
Another effective way to use peer review is to remove the name from three to six papers
(more or less) and hand them out to the entire class. These papers are labeled by number or
letter. The teacher then asks specific questions about Paper A or Paper B. Students get to see
good and bad examples of the particular points being taught. These papers can be pulled from a
pile of hand-in assignments, or certain students can be required to turn in the assignment in
advance. If the second route is chosen, all students should be required to turn in an assignment
early at some point during the semester or year. A similar approach is to select examples of
student writing as you review or grade preliminary or partial drafts of an assignment. Focus on
one lesson at a time – e.g., drafting accurate case descriptions or Questions Presented – and
present each example on a slide for class discussion. Again, have a specific list of the criteria
that students will use to evaluate each example. If there is sufficient time, ask students to edit
the examples to make them stronger.
The criteria that you provide teaches students what is important. Applying those criteria
to someone else’s work helps students learn what to look for in a piece of writing. Ultimately,
the peer review process allows students to begin to apply those criteria to their own work –
initially, as they become more comfortable receiving feedback and using it to revise their work;
later, as they begin to apply the criteria they have learned earlier in the writing process.

Assessment of Student Learning

From its first edition, A Practical Guide to Legal Writing & Legal Method has
recognized the importance of writing as a key to deep learning. Experienced legal writers often
have a well-developed process that they use – almost unconsciously – as they become
increasingly skilled. Although checklists, rubrics, and grading sheets can seem formulaic, they
can be a very useful tool for teaching, learning, and assessment.
For you as a teacher, they require you plan your course thoughtfully. When you design
a checklist, rubric, or grading sheet for a particular project, you must think hard about what the
project requires and what skills you want to prioritize. You can then reverse engineer your
syllabus to ensure that you teach the skills you are going to assess. When you use a tool that
you have designed to assess student work, you can trust that each student is being evaluated on
the basis of the same criteria. If points are allocated to the various components of the project,
assessment tools minimize the risk that certain kinds of errors will have an outsized impact on a
student’s final grade.
Project checklists, rubrics, and grading sheets also provide benefits for students. First,
they teach students the components of a project – e.g., that an office memo must have
Questions Presented and a Statement of Facts. Second, they teach students what those
components should and should not do – e.g., that Questions Presented should combine law and
fact, but should not draw legal conclusions. Third, they help students understand and begin to
internalize the criteria that good lawyers use when they write. Finally, they provide

197
transparency; students know what is expected of them and know how you will assess their
work.
Two caveats about using checklists, rubrics, and grading sheets: (1) They can help you
assess student work efficiently, but they cannot replace individualized feedback. Consider using
these tools in combination with at least some explanatory notes that point out where and how
students met or failed to meet some of the key criteria. (2) Remind students that checking off an
item on a checklist does not necessarily mean that the task was done well. Consider using
detailed rubrics and grading sheets, rather than checklists, for later drafts and final projects. For
each item, include options that describe the quality of the student’s work.
Several examples of checklists, rubrics, and grading sheets can be found in Appendix II
of this teacher’s manual.

198
APPENDICES

199
ABOUT THE APPENDICES

The following three appendices include three kinds of materials: Appendix I contains
sample discussions, annotated to illustrate the principles taught in various chapters of the text,
Appendix II contains examples of assessment tools that can be used to evaluate student work,
and Appendix III contains a variety of other teaching aids.

Appendix I: Annotated Discussions


Students often have trouble applying the lessons learned in class to their own writing.
Understanding a thing and successfully doing the thing can be separated by hours of hard work,
and this can be very frustrating for the new legal writer. One technique that many teachers use
to help students internalize a lesson is the in-class “deconstruction” of a sample. We have
included seven such samples in Appendix I.
Appendix I.A has three sample discussions based on the contract problem that begins in
Exercise 7-A of the text and continues in Exercises 12-A 13-A, and 16-A. Appendix I.B
contains one discussion based on the nuisance problem that begins in Exercise 8-C of the text
and continues in Exercises 14-C, 17-B, and 23-E. Appendix I.C reproduces the summaries of
the law from the two summary-of-the-law memos, which appear as Appendix B in the text.
Appendix I.D reproduces the discussion section of the office memo in Tyler v. Eastern Pacific
University that appears as Appendix C in the text.
All seven of these samples are annotated to show students where the author sets out the
governing legal rule, describes the precedent cases, analyzes the issue, responds to opposing
arguments, or employs organizational principles. You can review these annotated samples in
class, but a better approach is to give your students a “clean,” unannotated, copy of the sample
–and ask them to “deconstruct” it in class, focusing on the particular lesson you are working on.
For example, you might first ask students to find and label the CiRAC components of the
discussion in conjunction with Chapter 11 (Organization). Later you might ask them to identify
thesis statements and topic sentences in conjunction with Chapter 14 (Drafting the Analysis).
As you work through the samples, students will begin to recognize how the various lessons that
you teach operate when put to use in a piece of writing.

Appendix II: Assessment Tools


Appendix II focuses on student assessment. Appendix A contains three examples of the
kind of checklist that can be provided to students so they understand your expectations for a
particular assignment while they are still working on it. Appendix II.B contains two examples
of a grading/criteria sheet. The first example, designed to be used with Exercise 14-A, is
detailed and specific; the second example takes a broader “holistic” approach. Appendix II.C
contains two examples of a different kind of assessment tool, the grading grid or rubric. The
first example in Appendix II.C is designed for an ungraded assignment; the second is designed
for a graded assignment. Many teachers use a variety of feedback mechanisms during the
course of the year, depending on the time of year, the nature of the assignment, the time
allocated to the assignment, and the pedagogical goals of the assessment. Similar grading sheets
and grids can be designed for many of the exercises in the text, or for other writing projects that
you assign.

200
Appendix III: Teaching Aids

Appendix III is designed to give you some ideas for teaching specific lessons or skills.
Appendix III.A has three examples of a self-evaluation exercise. Appendix III.B focuses on
Questions Presented and describes a three-step method for teaching your students how to
construct good questions. Appendix III.C is a short handout that provides some tips for writing
a comprehensive conclusion to an objective analytical memorandum. Finally, Appendix III.D
describes a pre-writing technique that helps students learn how to read, process, and organize
the law and facts relevant to any new problem they are asked to solve. All of these teaching
aids give students tools that will help them become more independent legal thinkers and
writers.

201
Appendix I
Annotated Discussions

202
Appendix I.A
SAMPLE ANNOTATED DISCUSSIONS
Exercises 7-A, 10-A, 12-A, 13-A, and 16-A

The examples below are based on the facts and law used in Exercises 7-A, 10-A, 12-A,
13-A, and 16-A. Examples 1 and 2 illustrate the analysis of the offer issue; Example 3
illustrates the analysis of the acceptance issue.

Example 1: Effective Offer

Assume that an introductory or thesis paragraph precedes this discussion and analyses
of the acceptance and consideration elements follow it.

Thesis McKay made an effective offer to sell his yacht to Green because
Statement/
Conclusion (Ci) Green reasonably believed McKay intended to make a binding offer, even

though McKay did not actually have that intent. An offer is effective in this
Rule (R)
state if the offeror intended to make a binding offer, but intent is defined not

by what the offeror actually intended but by an objective standard:

“[W]hether a reasonable person in the offeree’s shoes would believe that the

offeror intended to make a binding contract.” Derek v. Beir (1985).

Facts, holding, In Derek, the offeror told the offeree, his neighbor, that for five
& reasoning
of Derek dollars he would build a wall between their lots so that he would never have

to look at the offeree’s “ridiculous” face again. The offeree agreed and paid

him five dollars, but the offeror never built the wall. The offeree in that case

knew about the offeror’s reputation for playing practical jokes on his

neighbors. Under those circumstances, the court held that there was no

binding offer because the offeree either knew or should have known that the

offeror was not serious. Id.

203
Application of Unlike the offeree in Derek, who knew the offeror’s reputation for
law (A)
playing practical jokes, Green did not know of McKay’s reputation as a

Facts, inferences, practical joker and so had no reason to doubt that the offer was genuine. His
& reasoning
distinguishing response at the end of the conversation that he needed time to think about
Derek &
supporting the offer and raise the money shows that he took the offer seriously. This
conclusion
attitude was reasonable because McKay gave him no reason to doubt the

sincerity of the offer.

In both Derek and this case, the offeror named an absurd price. In
Facts, inferences,
& reasoning
comparing Derek, the offeror promised to build a ten-foot-high wall between his lot and
Derek &
explaining the offeree’s lot for five dollars. McKay offered to sell his boat for one-tenth
counter position
of its value. A reasonable person might have suspected that McKay’s offer

Reason for was not serious. Green, who knew the value of the boat, did express
rejecting
counter incredulity in response to the offer when he said, “You can’t be serious.”
position
Yet McKay continued the conversation after that and never intimated that

the offer was a joke. Green’s belief that McKay intended to make a binding
Conclusion (C)
offer was therefore reasonable under the circumstances.

204
Example 2: Ineffective Offer

Assume that an introductory or thesis paragraph precedes this discussion and analyses of
the acceptance and consideration elements follow it.

Thesis McKay’s offer to sell his yacht to Green was not effective because
Statement
Conclusion (Ci) a reasonable person would not have believed he intended to make a binding

contract. An offer is effective in this state if the offeror intended to make a


Rule (R)
binding contract, but intent is defined not by what the offeror actually

intended but by an objective standard: “[W]hether a reasonable person in

the offeree’s shoes would believe that the offeror intended to make a

binding contract.” Derek v. Beir (1985).

In Derek, the offeror told the offeree, his neighbor, that for five
Facts, holding,
& reasoning dollars he would build a wall between their lots so that he would never
of Derek
have to look at the offeree’s “ridiculous” face again. The offeree agreed

and paid him five dollars but the offeror never built the wall. The offeree in

that case knew about the offeror’s reputation for playing practical jokes on

his neighbors. Under those circumstances, the court held that there was no

binding offer because the offeree either knew or should have known that

the offeror was not serious. Id.

In Derek, the offeree attempted to enforce a contract based on a


Application of the
law (A) practical joke. The court rejected that attempt, holding that the offeree
Facts, inferences, &
reasoning knew or should have known that the offeror’s promise to build a ten-foot-
comparing
Derek & supporting high wall between their adjoining lots for five dollars was made in jest. Id.
conclusion
This case is similar. As a joke, McKay offered to sell his $100,000 yacht to

205
Green for $10,000. In both cases, the value of the subject of the offer was

greatly disproportionate to the price. Furthermore, Green knew the actual

value of the boat. In fact, he responded to the offer by exclaiming, “You

can’t be serious.” This response probably indicates Green’s belief that

McKay did not intend to make a binding contract. Any reasonable person

in Green’s shoes should have known the offer was not serious.

Facts, inferences, Nevertheless, Green may have believed that McKay intended to
& reasoning
distinguishing make a binding offer. Unlike the offeree in Derek, he did not know of the
Derek &
explaining offeror’s reputation as a practical joker. Also, during the conversation he
the counter
position
requested time to consider the offer and raise the money. He probably

would not have made this request unless he thought the offer was serious.

Reason for rejecting If he had that belief, however, it was not reasonable because of the great
counter position &
conclusion (C) disparity between the boat’s value and the offered price.

206
Example 3: Valid Acceptance

Assume that analysis of the offer element precedes this discussion and analysis of the
consideration element follows it.

Transition & The second requirement of an enforceable contract was met


thesis statement/
Conclusion (Ci) because Green accepted McKay’s offer. A valid acceptance must be
Rule (R) sufficiently communicated, and it must mirror the offer in every respect.

Anselm v. Kinnet Textiles, Inc. (1986).

Thesis Green’s acceptance over the telephone of McKay’s offer was


sentence/conclusion
regarding the first sufficient. In Anselm, the offeree communicated his acceptance of
requirement (Ci)
Anselm’s offer to sell cotton by giving a note to Anselm’s secretary, who
Rule: Facts,
holding, placed it on Anselm’s cluttered desk. Anselm did not see the note for
& reasoning of
Anselm relevant
several weeks and committed the cotton to another buyer. The court
to sufficient
communication (R)
rejected Anselm’s argument that there was no acceptance because he did

not read the note. It found that placement of the note on his desk was

sufficient communication because “it would be unreasonable to require the

offeree to ensure that the acceptance is read.” Id.

Similarly, it would be unreasonable in this case to require the


Application of the
law (A)
offeree to ensure that his acceptance was heard. Green delivered his
Facts, inferences,
& reasoning
supporting acceptance in the same way the offer was made – by telephone. He called
conclusion
McKay’s office and left a message for McKay that he had called. McKay

returned Green’s call from the clubhouse at a golf course. He had been

drinking and passed out before he heard Green’s acceptance. Because

McKay initiated the call, Green could not have known that McKay lost

207
Analogy to consciousness soon after that. He delivered the message, just as the offeree
Anselm
in Anselm did, and he had no duty to make sure McKay received it.

Distinction of This case differs from Anselm, however, because the acceptance in
Anselm –
facts, inferences, Anselm was written rather than oral. Although it is unreasonable to expect
& reasoning
supporting counter one who sends a written message to ensure that the recipient reads it, it is
position
not as unreasonable to expect one talking on the telephone to make sure

that the other party hears the message. When McKay did not reply, Green

could have suspected that something was wrong, but unless McKay’s

manner of speech indicated he was drunk, Green should not be expected to

know that he had passed out and was thus incapable of hearing the

Reason for message. Because the Anselm court focused on whether the offeree
rejecting
counter position
sufficiently communicated his acceptance, not on whether the offeror

Conclusion on first
received it, Green’s oral acceptance to McKay is likely to be considered
requirement (C)
sufficient communication.

Thesis statement/ Green’s acceptance also met the requirement in this state that “the
conclusion (Ci)
on second acceptance must mirror the offer in every respect.” Id. If the acceptance
requirement of
acceptance. varies from the offer, it is a counteroffer and not an acceptance. Id. In
Rule on second
requirement (R) Anselm, the offeror, Anselm, offered to sell 200 bales of cotton to Kinnet
Facts, holding,
& reasoning of
Textiles, Inc., with one hundred bales to be delivered on March 1 and
Anselm relevant
to mirror another hundred on April 1. Kinnet sent a messenger to Anselm’s office
image rule
with a written note that stated: “We agree to your offer in all respects if we

can instead take delivery on March 5 and April 5.” The court held that the

208
attempted acceptance was invalid because the acceptance differed from the

offer. Id.

Application of law Green’s acceptance did not vary from the original offer. He called
(A)
McKay on the telephone and said, “I accept the offer.” The offeree in
Facts, inferences,
& reasoning Anselm, on the other hand, said that he agreed to the offer if the dates were
supporting
conclusion &
changed. This condition turned the attempted acceptance into a
distinguishing
Anselm
counteroffer. After Green accepted the offer, he said, “But I’d like to see

your golf clubs thrown into the deal, too.” Because Green used the verb

“like,” this statement was a suggestion or request rather that a condition or

demand for additional goods.

Facts, inferences, Nevertheless, Green did use the conjunction “but,” which might be
& reasoning
supporting considered a conditional term. If so the statement would be a demand for
counter position
additional goods. “Acceptance conditioned on the seller’s delivering

additional goods not specified in his offer is a counteroffer and a rejection.”

Reasons for Edna C. Simpson, Contracts 30-31 (3d ed. 2006). It is not likely, though,
rejecting counter
position that Green would put a condition on acceptance of an offer that was quite

advantageous to him. It is more likely that he was either joking or trying to

see how far McKay’s generosity would extend. Thus, his acceptance
Conclusion as to
second requirement
(C) mirrored the offer because it did not demand inclusion of additional goods.

Green’s acceptance of McKay’s offer to sell the yacht was valid.


Final conclusion as
to Acceptance (C) Green sufficiently communicated an unconditional acceptance when he

said, “I accept the offer.”

209
Appendix I.B
SAMPLE ANNOTATED DISCUSSION
Exercises 8-C, 14-C, 17-B, and 23-E

The example below is based on the facts and law used in Exercises 8-C, 14-C, 17-B, and 23-E.

Thesis statement/ Bradley Greenleaf has no cause of action in nuisance against Peter
Conclusion (Ci)
Elliot, a neighboring property owner who erected two billboards that block

the flow of sunlight to Greenleaf’s solar panels. The rule in this state is that
Rule regarding
legal right (R) landowners “must so enjoy their property as not to injure . . . a legal right in

the property of another.” Shover v. Scott (1889). An action for nuisance


Synthesized rule
defining action for
will lie if a property owner’s use of the land causes injury to the legal rights
nuisance
of another property owner, Shover, and the effect of the use is

unreasonable, Blum v. Disposal Systems, Inc. (1997). Even though the

billboards are an unreasonable interference with Greenleaf’s use and

enjoyment of his property and have damaged his business financially, Elliot

did not cause injury to a legal right.

Thesis statement/ Greenleaf has no legal right to the free flow of light. In Shover, the
Conclusion on 1st
(threshold) issue court recognized a legal right to lateral support. Thus, the plaintiff had a
(Ci)
cause of action for damages he sustained when an owner of adjoining
Synthesis of law on
1st sub-issue (R) property excavated his land and caused the plaintiff’s land to cave in. Id.
Shover case
description Only one American court, however, has recognized a legal right to the free

flow of light. Cohen v. Andrus, cited in Cassells v. Avery (1959). The court
Cassells case
in Cassells found that Cohen was inapplicable because it concerned the
description
proposed blockage of sunlight from a solar collector, whereas the case

210
before it concerned a hotel owner’s proposed ten-story addition that would

cast a shadow on the adjoining hotel owner’s sunbathing areas. The court

therefore refused to recognize a legal right to the free flow of light. Id.

Greenleaf will suffer economic hardship, as did the hotel owner in


Application of law to
1st sub-issue (A) Cassells, unless the court recognizes a right to the free flow of light. The

court is unlikely to do so due to the lack of precedent. The economic


Analogy to
Cassells & facts, damage in the two cases is similar. Greenleaf paid $300,000 to renovate the
inferences, &
reasoning building housing his architectural firm and to fit it with solar panels. His
supporting
writer’s
conclusion
specialty is solar design for residences. Not only will Greenleaf’s costs of

doing business increase, but the appeal of his solar energy promotion will

also be less attractive because potential customers will realize that solar

panels can be blocked from the sunlight by neighbors who erect billboards

or buildings or who plant tall trees. The hotel owner in Cassells would

suffer economic damage because the neighboring hotel’s addition would

shadow its beaches and sunbathing areas, making them less attractive to

hotel guests. The Cassells court pointed out that the doctrine of nuisance

requires that a landowner injure the legal rights of another. Id. Because the

court did not recognize a legal right to the flow of light, it refused to enjoin

the neighbor’s addition.

To prevail, Greenleaf must convince the court to recognize a new


Facts, inferences,
& reasoning legal right. The court in Shover recognized the right to lateral support. The
supporting
opposite distinction between the right to lateral support and the right to the flow of
conclusion
light is arguably artificial. In each case, a practice that infringes on the right

211
could cause severe economic harm. Furthermore, it is in the interest of

today’s society to encourage the development of solar energy, an interest

that was acknowledged by the court in Cassells. Nevertheless, the reason


Reasons for
rejecting for the rule in Cassells is just as valid today as it was in 1959. Requiring
counterargument
that every landowner allow the free flow of sunlight onto neighboring

property would make building construction, especially in high density

areas, economically unfeasible. It might even prohibit the planting of

certain trees. Greenleaf has no strong argument for a change in the law, and
Conclusion &
reasons for
in the absence of a strong or compelling argument, the court is unlikely to
Conclusion (C)
expand the rule in Shover to include a legal right to the free flow of

sunlight.

Transition to If the court recognizes a legal right to the free flow of sunlight, it
second issue &
thesis statement/ will consider the billboards that block the light from Greenleaf’s solar
Conclusion (Ci)
panels a nuisance. “Whether one’s use of property is reasonable is
Rule (R)
determined by the effect such use has on the neighboring property.” Blum.

Explanation of In Blum, the court focused on the balance between the respective
Blum court’s
interpretation & interests of adjacent landowners. The court found that a landfill operator’s
application of
unreasonable use use of his land unreasonably interfered with his neighbor’s hog farm
element
operation when noise and vibrations generated by trash hauling trucks

caused the conception rate of the neighbor’s sows to decrease from eighty

to thirty percent. The court held that the landfill operation was not “a mere

annoyance or inconvenience” but a nuisance. Id. “[A] use of property that

essentially confiscates or destroys the neighboring property is unreasonable

212
and constitutes a nuisance.” Id.

Balancing the interests of the respective owners in this case presents


Application of the
law (A) Blum a close question. The billboard owner has an interest in using his property
balancing
test to its best advantage. Requiring him to remove the billboards might cause

him to lose substantial business. Although the billboards reduce the

effectiveness of Greenleaf’s solar energy promotional appeal and increase

his costs substantially, they do not amount to as drastic a destruction of his

business interests as was the case in Blum, where the farmer’s sows

suffered a decrease in conception rate of more than fifty percent. The

competing economic interests of the parties in this case are more evenly

balanced than in Blum, but they weigh more heavily in Greenleaf’s favor.

Conclusion & His economic losses, over the years, are likely to be more than the billboard
reasons (C)
owner’s losses.

Greenleaf will probably not succeed in an action against the


Conclusion as
to both issues & billboard owner because the court is unlikely to recognize a legal right to
summary of
reasons (C)
the free flow of sunlight. Therefore, he has suffered no injury recognized at

law. If the court does recognize such a legal right, Greenleaf is likely to

win in a nuisance suit against his neighbor. The erection of the billboards is

an unreasonable interference with Greenleaf’s right to light. In balancing

the interests of the landowners, the court is likely to find that the billboard

owner’s economic loss if he is forced to remove the billboards is

outweighed by the harm he has inflicted on Greenleaf.

213
Appendix I.C
ANNOTATED SUMMARY-OF-THE-LAW
MEMORANDA
See Text Appendix B

Example 1: FROM: Loren Taylor


SENT: September 4, 2019
TO: Maria Hernandez
RE: Tolling of the statute of limitations in fraud cases.

Question Presented: You asked me to research whether an action for negligent


Statement of the
question asked misrepresentation is barred by the statute of limitations when our client did not
discover the misrepresentation until four years after it had occurred. The results of
my research are summarized below.

Brief Answer: The statute of limitations for a negligent misrepresentation claim


Answer to question
regarding when the is three years. The statute may be tolled until the plaintiff discovers the fraud if
statute of limitations may the plaintiff was unaware of the fraud and had no reason or duty to investigate the
be tolled
situation prior to that point. So whether our client’s action is barred depends on
the circumstances and information available to the client prior to the fraud’s
discovery.

Summary of the Law: Negligent misrepresentation is a species of fraud. It is thus


subject to the statute of limitations for fraud, which requires an action be brought
Legal rules regarding the “[w]ithin three years after all of the elements of fraud have been met.” Cal. Civ.
statute of limitations and
when it may be tolled Proc. Code § 338(d) (West 2006). The statute also contains an exception: “The
cause of action in that case is not to be deemed to have accrued until the
discovery, by the aggrieved party, of the facts constituting the fraud.” Id. The term
“discovery” is the point at which the plaintiff has sufficient information to put a
reasonable person on notice that the injury was caused by wrongdoing. Kline v.
Turner, 105 Cal. Rptr. 2d 699, 702 (Ct. App. 2001). Thus, the statute may be
tolled until the plaintiff discovered the fraud.

214
The courts have developed an exception to this rule. The statute is tolled when
Synthesis of cases
illustrating how courts a plaintiff was deceived as to the truth, but not when a plaintiff had sufficient
have applied the law
information to put a reasonable person on notice to inquire further. For example,
relevant to the tolling of
the statute of limitations in Watts v. Crocker-Citizens National Bank, 183 Cal. Rptr. 304, 307 (Ct. App.
1982), the claim for misrepresentation concerning the installation of a water line
was not barred by the statute of limitations. The court reasoned that the defendant
had failed to disclose to the plaintiff that no water was in the pipeline or that the
defendant no longer operated the water system, and the plaintiff was unaware of
facts to the contrary. However, in Bedolla v. Logan & Frazer, 125 Cal. Rptr. 59
(Ct. App. 1975), a fraud claim brought by the limited partners of a business
against the general partners was barred because the plaintiffs’ knowledge of other
financial is management during the three years following the fraud should have
alerted them to suspect other wrongdoing and,
thus, to discover the fraud. Id. at 69–70.
Thus, since the three-year statute of limitations for fraud may be tolled until
Summary of the law
regarding when the the fraud is discovered unless the facts and circumstances surrounding the fraud
statute may be tolled and would have put a reasonable person on notice to inquire further, we need to
recommendation as to
client investigate the facts of our client’s situation before we can determine whether the
statute of limitations may bar an action.

215
Example 2: TO: Maria Hernandez
FROM: Loren Taylor
DATE: September 8, 2019
RE: Law on negligent misrepresentation

Question Presented: You have asked me to summarize the law regarding the elements
Statement of the
question asked necessary to succeed in an action for negligent misrepresentation.

Brief Answer: In order to succeed in an action for negligent misrepresentation,


Answer to question
regarding negligent the plaintiff must prove that the defendant misrepresented a material fact to the plaintiff,
misrepresentation.
without any reasonable basis for believing it to be true, intended that the plaintiff
rely on that fact, and that the plaintiff reasonably relied on that fact to his detriment.

Summary of the Law on Negligent Misrepresentation:


Cal. Civ. Code § 1709 (West 2009) provides: “One who willfully deceives another
Legal rules for
negligent with intent to induce him to alter his position to his injury or risk, is liable for any
misrepresentation damage which he thereby suffers.” Deceit is defined to include: “The assertion, as a fact,
of that which is not true, by one who has no reasonable ground for believing it to be
true.” Cal. Civ. Code § 1710(2) (West 2009). One type of deceit or fraud is negligent
misrepresentation. B.L.M. v. Sabo & Deitsch, 64 Cal. Rptr. 2d 335, 342 (Ct. App. 1997).
A plaintiff making a negligent misrepresentation claim must prove each of the following
Elements of negligent elements: “(1) A misrepresentation of a past or existing material fact, (2) without
misrepresentation
identified/roadmap reasonable grounds for believing it to be true, (3) with intent to induce another’s reliance
on the fact misrepresented, (4) ignorance of the truth and justifiable reliance thereon by
the party to whom the misrepresentation was directed, and (5) damages.” Id. (quoting
Fox v. Pollack, 226 Cal. Rptr. 532, 537 (Ct. App. 1986)).

1. The misrepresentation must be of a past or existing material fact. A statement as to


Statement of first
element value or other casual expression of belief is not actionable. See Gentry v. eBay, Inc., 121
Cal. Rptr. 2d 703, 718–19 (Ct. App. 2002). However, a statement made by one who
Legal rules for first possesses or holds himself out as possessing superior knowledge or information on the
element
subject may be treated as one of fact where the statement, although in the form of an
opinion, is “a deliberate affirmation of the matters stated.” Bily v. Arthur Young & Co.,
834 P.2d 745, 768 (Cal. 1992) (quoting Gagne v. Bertran, 275 P.2d 15, 21 (Cal. 1954)).

216
Thus, an opinion by a trained professional may be a statement of fact, while a casual
opinion offered by one speaking outside of his expertise is not. For example, a statement
Synthesis of cases
illustrating how courts by legal counsel for an insurance company regarding exclusions in a policy was material
have applied the law
relevant to first element fact. Shafer v. Berger, Kahn, Shafton, Moss, Figler, Simon & Gladstone, 131 Cal. Rptr.
2d 777, 793 (Ct. App. 2003). Similarly, the court has held that inaccurate financial
projections in an accountant’s report were statements of fact. Anderson v. Deloitte &
Touche, 66 Cal. Rptr. 2d 512, 516-17 (Ct. App. 1997). In both cases, the court held that
statements by experts, even if presented as opinions, may be relied upon for the facts
asserted. However, an accountant’s opinion as to the appraisal value of real estate was
only his opinion, not a statement of material fact, because he was not an appraiser and
had never represented himself as such. Neu-Visions Sports, Inc. v.
Soren/McAdam/Bartells, 103 Cal. Rptr. 2d 159, 163–64 (Ct. App. 2000). Thus, the key
inquiry is whether the defendant was a trained professional or held himself out as having
superior knowledge or information about the statement or opinion asserted.

Recent cases concerning the distinction between fact and opinion with regard to
professionals have concerned statements by defendants trained in recognized professions
like law and accounting. However, the law on this point is not limited to trained
professionals. In Gagne, an earlier case, the court concluded that a statement regarding
soil composition was one of fact since, even though the defendant was not an engineer or
geologist, he had stated that he was in the business of soil testing and had asserted his
findings as fact rather than as an opinion. Gagne, 275 P.2d at 21. Thus, there is precedent
Summary of the law on
the first element for treating a statement as one of fact rather than opinion where the defendant held
himself out as an expert in that area and represented his statement as one of fact.
Statement of the second 2. The defendant must have had no reasonable basis to believe the statement was true.
element
An incorrect statement made in good faith that was grounded in personal inspection or
Legal rules for second investigation, or in standard industry practice, is not actionable. In Wilbur v. Wilson, 3
element
Cal. Rptr. 770, 773 (Ct. App. 1960), for example, the court held that the seller of a 79-
Synthesis of cases acre farm had no reasonable basis for asserting that his farm had 94 acres, when his
illustrating how courts belief was solely based on statements from the previous owner. Similarly, even though
have applied the law
relevant to second neither the seller nor the previous owner had put fill dirt on a lot, a seller had no factual
element basis on which to assert to the purchaser that the lot was solid and contained no fill.
Gagne, 275 P.2d at 15. However, in Diediker v. Peellee Financial Corp., 70 Cal. Rptr.

217
2d 442 (Ct. App. 1997), a trustee reasonably believed property he sold was
unencumbered because he had received a report from a title insurance company that did
not show an IRS lien on the property. The court noted it was standard industry practice to
Summary of law on rely on such reports to disclose any liens. Id. at 448. Therefore, a statement made in good
second element faith that is based on personal investigation, industry standard, or another reliable source
will not support a claim for negligent misrepresentation, even if it is false.
Statement of the third 3. The statement must have been made with the intent to induce the plaintiff’s
element
reliance. The court has found such intent where the facts show that the defendant made
Legal rules for third the statement to induce specific action by the plaintiff. For example, in Gagne, the court
element
found intent where the defendant told the plaintiffs the lot contained no fill, knowing that
they would rely on his statement and proceed with the purchase of the property. 275 P.2d
Synthesis of cases at 20. The court also found intent in Anderson, where an accountant provided inaccurate
illustrating how courts
have applied the law financial information about several businesses, knowing that potential investors would
relevant to third rely upon the information. 66 Cal. Rptr. 2d at 517. The court concluded that the intent to
element
induce reliance may be established when a professional provides information knowing
that a specific class or group of people will rely upon it, even though the
misrepresentation was not made directly to the plaintiff. Id. The key to this element is
Summary of the law on showing that the defendant knew that the plaintiff or class of persons of whom the
third element
plaintiff was a member would rely upon the information.
Statement of the fourth 4. Plaintiff’s reliance on the statement must have been reasonable. The courts
element
evaluate the reasonableness of a plaintiff’s reliance “in light of his own intelligence and
Legal rules for fourth information.” Seeger v. Odell, 115 P.2d 977, 980–81 (Cal. 1941). The plaintiff may rely
element
on a reasonable- sounding representation, even if he did not conduct his own
investigation, see Ashburn v. Miller, 326 P.2d 229, 234–35 (Cal. Ct. App. 1958).

Roadmap of two sub- Reliance is reasonable when either 1) the position or experience of the person making the
issues of fourth element representation makes it credible, or 2) there were no other facts to raise a question about
the truth of the statement.
Rule for first sub-issue When the person making the representation holds himself out as an expert on the
of fourth element subject of the representation, the plaintiff may reasonably rely on the representation. In
Synthesis of cases Shafer, for instance, the plaintiffs’ reliance was reasonable when the attorney
illustrating how courts
have applied the law representing the insurance company misrepresented exclusions in an insurance policy.
relevant to first sub- 131 Cal. Rptr. 2d at 793. Similarly, in Carroll, the buyers of a mobile home park
issue of the fourth
element reasonably relied on the seller’s statement concerning zoning because the seller had

218
experience with real estate generally and mobile home parks specifically. 159 Cal. Rptr.
at 780–781. Finally, in Gagne, the plaintiffs justifiably relied on misrepresentations about
fill on property they purchased because the defendant held himself out as an expert in soil
testing, 275 P.2d at 15. In all three cases the plaintiff was entitled to rely on the statement
Summary of the law on
first sub-issue of fourth since the defendant represented himself as one who was in a position to know the truth of
element the statement.

Rule for second sub- Second, where the plaintiff has no information that contradicts the misrepresentation,
issue of fourth element
his reliance is reasonable. In Harper v. Silver, 19 Cal. Rptr. 78 (Ct. App. 1962), buyer’s
Synthesis of cases reliance on a seller’s misrepresentation that a boat had matching 275 horsepower engines
illustrating how courts
was reasonable. Id. at 81. Even though the buyer talked to the mechanic who had
have applied the law
relevant to second sub- replaced one of the original engines prior to the sale, the buyer did not ask about the
issue of the fourth
element horsepower of the engines and did not discover anything contradicting the original
misrepresentation. Likewise, in Clement v. Smith, 19 Cal. Rptr. 2d 676, 680 (Ct. App.
1993), the court found that, absent some notice or warning, an insured could reasonably
rely on his agent’s broad assertions regarding the coverage of his policy without
verifying the accuracy of the statement. Thus, the reasonableness of the plaintiff’s
Summary of the law on reliance is based on a two-part inquiry that considers both the position of the person
the fourth element
making the representation and whether the plaintiff was aware of any information
inconsistent with the misrepresentation.
Statement of the fifth 5. Damages. The plaintiff’s reliance on the negligent misrepresentation must have
element caused legally compensable harm. Generally, this requires proof of financial loss
Legal rule for fifth incurred as a result of the plaintiff’s reliance. See, e.g., Bily v. Arthur Young & Co., 834
element
P.2d 745, 768 (Cal. 1992) (investors who relied on accountant’s inaccurate statements in
Synthesis of cases corporate audits suffered financial harm when company went bankrupt); Anderson v.
illustrating how courts
have applied the law Deloitte & Touche, 66 Cal. Rptr. 2d 512, 516–17 (Ct. App. 1997) (investors who relied
relevant to
the fifth element (since on inaccurate financial forecasts prepared for potential investors suffered financial harm
these cases are only when partnership went bankrupt); Shafer v. Berger, Kahn, Shafton, Moss, Figler, Simon
representative of the
type of harm required, & Gladstone, 131 Cal. Rptr. 2d 777, 793 (Ct. App. 2003) (plaintiff suffered financial
full case-summaries are
harm after being induced to settle for less than the judgment amount by attorney’s
not provided)
misrepresentation about defendant’s insurance coverage); Gagne v. Bertran, 275 P.2d
15, 21 (Cal. 1954) (plaintiffs suffered financial loss from higher-than-anticipated
construction costs where original estimate was based on defendant’s misrepresentation).
219
Appendix I.D
ANNOTATED DISCUSSION SECTION – ANALYTICAL MEMORANDUM
See Text Appendix C

DISCUSSION

I. Negligent Misrepresentation

Timothy Tyler can probably recover damages for harm caused by Richard
Conclusion
on negligent Cramer’s negligent misrepresentation that Eastern Pacific’s GPS program had
misrepresentation
issue (Ci)
a preferential admissions policy for Eastern Pacific University graduates.

Legal rules for Damages for misrepresentation are recoverable under Cal. Civ. Code § 1709
negligent
misrepresentation (R) (West 2009): “One who willfully deceives another with intent to induce him to

alter his position to his injury or risk, is liable for any damage which he

thereby suffers.” Deceit is defined to include: “The assertion, as a fact, of that

which is not true, by one who has no reasonable ground for believing it to be

true.” Cal. Civ. Code § 1710(2) (West 2009). One type of deceit or fraud is

negligent misrepresentation. B.L.M. v. Sabo & Deitsch, 64 Cal. Rptr. 2d 335,

342 (Ct. App. 1997). A plaintiff making a negligent misrepresentation claim

must prove each of the following elements: “(1) A misrepresentation of a past

or existing material fact, (2) without reasonable grounds for believing it to be

true, (3) with intent to induce another’s reliance on the fact misrepresented,

(4) ignorance of the truth and justifiable reliance thereon by the party to whom

the misrepresentation was directed, and (5) damages.” Id. (quoting Fox v.

Pollack, 226 Cal. Rptr. 532, 537 (Ct. App. 1986)).

220
Analysis of givens In this case, two elements are easily established. First, Cramer freely
for negligent
misrepresentation admitted that he made the statement to induce Tyler to enroll at Eastern
issue
Pacific, thereby establishing intent to induce reliance. Second, Tyler suffered

damages in giving up his full scholarship at Crater Lake College to attend

Eastern Pacific. He received only a nominal scholarship amount and had to pay

most of Eastern Pacific’s tuition himself, incurring substantial debt. He also

suffered damages because he was not given priority in his application to the

Roadmap showing law GPS program. Therefore, three questions remain: A) whether Cramer’s
for negligent
misrepresentation will statement was a misrepresentation of an existing fact or only his opinion, B)
be applied using three
sub-issues
whether Cramer lacked reasonable grounds for believing his statement to be

true, and C) whether Tyler was justified in believing the statement to be true

and relying on it.

Application of the law A. Cramer’s statement will probably be considered a statement of fact
(A) - Conclusion on
first sub-issue (Ci) rather than an opinion because he held himself out as a trained professional on

the subject of Eastern Pacific’s admissions policies. A misrepresentation must


Legal rules relevant
to first sub-
issue (R) be one of existing fact; a statement as to value or other casual expression of

belief is not actionable. See Gentry v. eBay, Inc., 121 Cal. Rptr. 2d 703, 718–

19 (Ct. App. 2002). However, a statement made by one who possesses or holds

himself out as possessing superior knowledge or information regarding the

subject of the statement may be treated as one of fact where the statement,

although in the form of an opinion, is “a deliberate affirmation of the matters

stated.” Bily v. Arthur Young & Co., 834 P.2d 745, 768 (Cal. 1992) (quoting

Gagne v. Bertran, 275 P.2d 15, 21 (Cal. 1954)).

221
Synthesis of cases The courts have applied this rule to conclude that an opinion by a trained
illustrating how courts
have applied the law professional may be a statement of fact in an action for negligent
relevant to first sub-
issue misrepresentation, while a casual opinion offered by one speaking outside of

his expertise is not. For example, in Shafer v. Berger, Kahn, Shafton, Moss,

Figler, Simon & Gladstone, 131 Cal. Rptr. 2d 777, 793 (Ct. App. 2003), the

court held that a statement by legal counsel for an insurance company

regarding exclusions in a policy was a statement of fact. Similarly, in Anderson

v. Deloitte & Touche, 66 Cal. Rptr. 2d 512, 516–17 (Ct. App. 1997), the court

concluded that inaccurate financial projections in an accountant’s report were

positive assertions of fact sufficient to create a triable issue of negligent

misrepresentation. However, where a principal in an accounting firm stated his

opinion as to the appraisal value of real estate, the court held that his statement

constituted only his opinion because he was not an appraiser and had never

represented himself as such. Neu-Visions Sports, Inc. v.

Soren/McAdam/Bartells, 103 Cal. Rptr. 2d 159, 163–64 (Ct. App. 2000).

Application of the law The defendants in Shafer and Anderson were professionals providing
to show how it
supports the information regarding a matter about which they were known to have specific
conclusion on first
sub-issue (A) information. Similarly, Cramer was a professional who asserted the existence

of a preferential admissions policy, a matter within his professional expertise

as a university recruiter. His situation is markedly different from the

accountant’s in Neu-Visions Sports, who asserted his opinion regarding

appraisal value. As a recruiter, Cramer is presumed to know the admissions

policies of the university that employed him. Even if his assertion that Eastern

222
Pacific’s GPS program gave priority to its own graduates was an opinion, it

was a statement related specifically to his profession. Therefore, like the

statements in Shafer and Anderson, Cramer’s statement should be regarded as

a statement of fact rather than a statement of opinion.

It is worth noting that recent cases concerning the distinction between fact

and opinion in this context have concerned statements by defendants trained in

recognized professions like law and accounting, not university recruiters.

However, in Gagne, an earlier case, the court concluded that a statement

regarding soil composition on a lot being tested for fill was one of fact even

though the defendant had not held himself out as an engineer or geologist. The

court so held because the defendant represented that he was in the business of

testing soil for fill and had asserted his findings as fact rather than as a

statement of his opinion. Gagne, 275 P.2d at 21. That Cramer was a recruiter

rather than an attorney or accountant thus is not likely to make a difference in

the court’s determination of this question. A court is likely to conclude that

Cramer’s statement was a positive assertion of fact, rather than an opinion.

It is possible, but not likely, that the court will regard Cramer’s statement
Statement of
counterargument for as an opinion rather than a statement of fact. His statement that Tyler would
first sub-issue
& explanation how have “a better chance of getting into the GPS program as an Eastern Pacific
law supports
Counterargument
graduate” is not as clearly factual as the attorney’s statements regarding

exclusions in the insurance policy in Shafer or the accountant’s financial

projections in Anderson. This may lead a court to view his statement less as a

professional opinion constituting a statement of fact and more as an expression

223
Why counter- of his personal view of the situation. However, by telling Tyler that he would
argument doesn't
change conclusion have a better chance of being admitted as an Eastern Pacific graduate, Cramer

asserted the existence of a preferential admissions policy as a fact, and did so

Final conclusion on in his capacity as a recruiter for Eastern Pacific. Therefore, his assertion is
first sub-issue (C)
likely to be regarded as a statement of fact, rather than his personal opinion.

Conclusion for second B. A court would likely conclude that Cramer had no reasonable grounds
sub-issue (Ci)
for making the statement because he had no specific information about the

GPS admissions policy and based his statement only on the existence of such a

policy in five other graduate programs. The courts have required persons
Legal rules relevant to
second sub-issue (R) making such representations to have a reasonable factual basis for them,

generally grounded in personal inspection or investigation or in standard

industry practice. In Wilbur v. Wilson, 3 Cal. Rptr. 770, 773 (Ct. App. 1960),
Synthesis of cases
illustrating how courts
for example, the court held that the seller of a 79-acre farm had no reasonable
have applied the law
relevant to second
sub-issue basis for his belief that his farm had 94 acres, when he based his representation

solely on statements from the previous owner. Similarly, it was unreasonable

for the seller of a lot containing substantial fill to tell the purchaser that the lot

was solid when he had no factual basis for the statement, even though neither

the seller nor the previous owner had put fill on the lot. Gagne, 275 P.2d at 15.

However, in Diediker v. Peellee Financial Corp., 70 Cal. Rptr. 2d 442 (Ct.

App. 1997), a trustee who sold property subject to foreclosure reasonably

believed the property was unencumbered because he had received a report

from a title insurance company that did not show an IRS lien. The court noted

that it was standard industry practice to rely on such reports to disclose the

224
existence of any liens. Id. at 448.

In light of these cases, Cramer’s representation is most likely unreasonable


Application of the law
to show how it
supports the because he did not personally investigate the factual basis for his statement
conclusion on the
second sub-issue (A) regarding the GPS admissions policy. Like the defendant in Gagne, who

wrongly assumed that the lot had not been filled, Cramer wrongly assumed

that the GPS program had a preferential admissions policy. Cramer’s statement

may be more egregious than the statement in Wilbur, because the defendant in

that case based his representation about the size of the property on a statement

by the previous owner. In Tyler’s case, no one told Cramer that GPS had a

preferential admissions policy. And, unlike the trustee in Diediker, who relied

upon the title report, Cramer had nothing beyond his personal assumption

regarding the existence of the admissions policy. That Cramer knew five other

graduate programs had preferential admissions policies does not make his

belief reasonable in light of his lack of information specifically regarding the

GPS program.

Statement of To be sure, Cramer based his assertion on accurate information regarding


counterargument for
second sub-issue and the preferential admissions policies of five other graduate programs at Eastern
how law supports
counter Pacific for which he had previously recruited students. He is therefore unlike
argument
the defendants in Wilbur and Gagne, who based their misrepresentations on

inaccurate information. Nevertheless, because Cramer was in a position that


Why counter-
argument does not
change conclusion would have easily allowed him to obtain information regarding admissions

225
Final conclusion on policies at Eastern Pacific, his awareness that he had no specific information
second sub-issue (C)
regarding the GPS program probably makes his belief unreasonable.

C. Finally, Tyler was probably justified in believing Cramer’s statement


Conclusion for third
sub-issue (Ci)
about the preferential admissions policy and relying on it. The courts evaluate

Legal rules relevant the reasonableness of a plaintiff’s reliance “in light of his own intelligence and
to third sub-issue (R)
information.” Seeger v. Odell, 115 P.2d 977, 980–81 (Cal. 1941). The plaintiff

may recover for damages based on a reasonable-sounding representation even if

he did not conduct his own investigation. Ashburn v. Miller, 326 P.2d 229, 234–

35 (Cal. Ct. App. 1958). When the plaintiff has not conducted his own
Establishing two
investigation, the courts consider two factors in determining the reasonableness
factors for third
sub-issue under the
rule as applied by the of the plaintiff’s reliance: the position or experience of the person making the
courts
representation and the existence of facts that should make the plaintiff suspect

the truth of the representation. See Carroll v. Gava, 159 Cal. Rptr. 778, 780–81

(Ct. App. 1979). Cramer’s position as a recruiter for Eastern Pacific University
Transitional
conclusion &
roadmap for two and the absence of any facts contradicting his representation likely justify
factors of
third sub-issue Tyler’s reliance.

Conclusion under first The court will likely conclude that Tyler’s reliance on Cramer’s
factor of third sub-
issue (Ci) misrepresentation was reasonable since, as a recruiter, Cramer represented

himself to be an expert on Eastern Pacific’s admission policies. When the


Law Legal rules
relevant to first factor person making the representation holds himself out as an expert on the subject
of third sub-issue (R))
of the representation, the courts have held that the plaintiff has acted

Synthesis of cases reasonably in relying on the representation. In Shafer, for instance, the court
illustrating how courts
have applied the law held that the plaintiffs’ reliance was reasonable when the misrepresentation
relevant to first factor
of third sub-issue regarding exclusions to the insurance policy was made by the attorney

226
representing the insurance company. 131 Cal. Rptr. 2d at 793. Similarly, in

Carroll, the court held that it was reasonable for the buyers of a mobile home

park to rely on the seller’s statement concerning zoning because the seller had

experience with real estate generally and mobile home parks specifically. 159

Cal. Rptr. at 780–781. Finally, in Gagne, the court held that the plaintiffs were

justified in relying on misrepresentations about fill on the property they

purchased because the defendant held himself out as an expert in soil testing,

Application of the law 275 P.2d at 15. Like the defendants in these three cases, Cramer also held
to show how it supports
the conclusion on the himself out as an expert in his field and made a statement about the existence
first factor of third
sub- issue (A) of a preferential admissions policy for the GPS program. As a recruiter, it was

his job to represent the university and provide information to potential

students. Since providing such information was within the scope of Cramer’s

Conclusion on the first expertise as a recruiter for Eastern Pacific, Tyler’s reliance was probably
factor of third sub-
issue (C) reasonable.

Moreover, Tyler’s reliance was probably reasonable because he had no


Conclusion on second
factor of information that called into question the truth of Cramer’s representation, even
third sub-issue(Ci)
though he was employed in the GPS office as a student. The reasonableness of
Legal rules relevant to
second factor of third
the plaintiff’s reliance is questionable only when the plaintiff is given
sub-issue (R)
information that contradicts the representation. In Harper v. Silver, 19 Cal.

Synthesis of cases Rptr. 78 (Ct. App. 1962), the seller of a boat told the buyer that the boat had
illustrating how courts
have applied the law matching 275 horsepower engines when in fact it had one 250 and one 275
relevant to second
factor of third sub- horsepower engine, causing the boat to run erratically. Even though the buyer
issue
talked to the mechanic who had replaced one of the original engines prior to the sale,

he never asked about the horsepower of the new engine and did not discover any

227
information that contradicted the original misrepresentation. As a result, the

court held that the buyer’s reliance on the original misrepresentation was

reasonable. Id. at 81. Likewise, in Clement v. Smith, 19 Cal. Rptr. 2d 676, 680

(Ct. App. 1993), the court found that, absent some notice or warning, an

insured could reasonably rely on his agent’s broad assertions regarding the

Application of the law coverage of his policy without verifying the accuracy of the statement. Like
to show how it supports
the conclusion on the the plaintiffs in these cases, who had no reason to doubt the truth of the matter
second factor of third
sub-issue (A) represented, Tyler had no reason to doubt Cramer’s assertion since nothing he
Conclusion on second learned from Cramer or through his employment with the GPS office called the
factor of third sub-
issue (C)
matter into question.

The court could conclude that Tyler’s reliance was not reasonable since his
Statement of counter-
argument for second
factor & explanation employment with the staff responsible for admissions gave him easy access to
how law supports
counterargument information regarding admissions policies. But, like the plaintiff in Harper,

Explanation why Tyler received no information that contradicted Cramer’s misrepresentation


counterargument does
not change conclusion regarding the admission policy. He had no reason to make further inquiries until

he applied for and was denied admission to the GPS program. Thus, Tyler can
Final conclusion on
issue of negligent likely establish all the elements to bring an action based on Cramer’s negligent
misrepresentation (C)
misrepresentation regarding the GPS admissions policy.

II. Statute of Limitations

Conclusion on the Tyler’s claim probably is not barred by the statute of limitations even
statute of limitations
issue (Ci) though his suit would be brought more than three years after the

Legal rules for the misrepresentation. Negligent misrepresentation is a species of fraud and is
statute of limitations
issue (R) subject to the statute of limitations for fraud. An action is to be brought

228
“[w]ithin three years after all of the elements of fraud have been met.” Cal.

Civ. Proc. Code § 338(d) (West 2006). The statute also contains an exception:

“The cause of action in that case is not to be deemed to have accrued until the

discovery, by the aggrieved party, of the facts constituting the fraud.” Id. The

term “discovery” has been defined as the point at which the plaintiff has

sufficient information to put a reasonable person on notice that the injury was

caused by wrongdoing. Kline v. Turner, 105 Cal. Rptr. 2d 699, 702 (Ct. App.

2001).

Application of the law Tyler probably did not have information sufficient to make him suspect the
under two discovery
elements (A) truth regarding the admissions policy until six months ago. Courts have tolled
/Conclusion on first
element (Ci) the statute when a plaintiff was deceived as to the truth, but not when a
Legal rules relevant plaintiff had sufficient information to put a reasonable person on notice to
to first element (R)
inquire further. For example, in Watts v. Crocker-Citizens National Bank, 183

Cal. Rptr. 304, 307 (Ct. App. 1982), a property owner’s action for
Synthesis of cases
showing how courts misrepresentation concerning the installation of a water line was not barred by
have applied law
relevant to first the statute of limitations. The court held that the defendant had failed to
element
disclose during ongoing communications with the plaintiff that there was no

water flowing through the pipeline or that the defendant was no longer

operating the water system. However, in Bedolla v. Logan & Frazer, 125 Cal.

Rptr. 59 (Ct. App. 1975), the court held that a claim brought by the limited

partners of a business against the general partners was barred where the

plaintiffs knew of some financial mismanagement by the defendants during

the three years following the fraud. The court stated that this financial

229
mismanagement should have alerted them, as reasonably prudent men, to

suspect the other wrongdoing and therefore to discover the fraud earlier than

they did. Id. at 69–70.

Tyler’s case is more similar to Watts than to Bedolla. Like the plaintiff in
Application of the
law to show how it Watts, who had no notice of any problems with the water line, Tyler had no
supports the
conclusion on first notice that the information regarding the admissions policy was false. Unlike
element (A)
the plaintiffs in Bedolla, who knew of wrongdoing by the defendants that

should have prompted inquiry, Tyler had no reason to investigate the


Final conclusion on
first element (C) admissions policy until his application was denied six months ago.

Nor is it likely that Tyler’s part-time job in the GPS program put him in a
Conclusion on second
discovery element (Ci) position where he should have discovered the misrepresentation. Having

access to information regarding a misrepresentation is not sufficient to trigger


Legal rules relevant to
second element (R)
the statute unless a duty to inquire is imposed by law or the circumstances are

such as to put a reasonably prudent person on notice to inquire further. Vega v.

Jones, Day, Reaves & Pogue, 17 Cal. Rptr. 3d 26, 38 (Ct. App. 2004). In Vega,

the court held that the statute did not bar an action based on fraud concerning

Synthesis of cases on the terms upon which a merger and stock issuance were based, even though the
illustrating how courts
have applied the law
terms of the transaction were a matter of public record in documents filed in
relevant to second
element
Delaware. Although the means of learning about the fraud were available to

the plaintiff earlier, the court concluded that the statute did not begin to run

unless and until the terms of the transaction or other circumstances would have

made a reasonable person suspicious enough to inquire further. Id. By contrast, in

National Automobile & Casualty Insurance Co. v. Payne, 67 Cal. Rptr. 784, 789

(Ct. App. 1968), the court held that an action alleging the fraudulent sale of

230
stock options was barred when the plaintiff corporation had two

representatives on the defendant corporation’s board of directors for eight

years before filing the action. The court found that their duties on the board of

directors and their access to records were sufficient to show that the plaintiff

should have discovered the fraud earlier. Id.

Because Tyler had the means to obtain the relevant information, but he had
Explanation of
how law supports
conclusion on no reason to do so, his situation is more like that in Vega, where the plaintiffs
second element
(A) could have located the information but were under no duty to do so. His job

responsibilities in the GPS program did not involve admissions policy or

admissions decisions, and the preferential admissions policy was never

mentioned during his employment. Unlike the corporate officers in Payne, who

had a duty to discover the corporate fraud, Tyler had no duty to investigate the

admissions policies of the GPS program.

Statement of Although less likely, a court could conclude that Tyler should have
counterargumen
t for second discovered the truth sooner. Unlike the information in Vega, which was filed in
element
another state, Tyler could have easily verified the existence of such a policy

given his employment in the GPS office. Tyler’s employment was similar to the

directors’ position in Payne, whose access to records put them in a position to

Why discover the fraud. But while the directors in Payne had a legal duty to inquire
counterargument
does not change further, Tyler had no such duty, and no factual circumstances put him
conclusion for
second element on inquiry notice. The policy was never discussed during his employment, and

his duties did not involve any matters related to the admissions process. Thus,
Final conclusion
on statute of
limitations issue because he never had a reason to inquire about the truthfulness of Cramer’s
(C)
representation, his claim is probably not barred under the statute of limitations.

231
Appendix II
Assessment Tools

232
Appendix II.A
SAMPLE CHECKLISTS AND REVISION GUIDELINES

Example 1:

Revision Checklist for Analytical Memorandum


(Page numbers track the 7th edition)

Format Requirements
1. Is the font type and size correct and have you placed your margins correctly?
2. Is your identifying information correct (To: From: Date: RE:)?
3. Have you insured you meet the word count/page limit requirements?
Question Presented [Review Dernbach, pp. 300-305]
1. Have you included the core question, the legal principle(s) and a summary of the
relevant facts?
2. Have you stated each legal issue separately?
3. Does your question allow a reader unfamiliar with your case to understand the precise
legal issue presented?
4. Is the tone of your question objective rather than persuasive?
5. Have you omitted proper names and citation?
Statement of Facts [Review Dernbach, pp. 306-313]
1. Do you have a context statement at the beginning of section to identify the parties and
cause of action?
2. Have you included all background facts necessary for the reader's understanding or
narrative flow?
3. Have you eliminated all irrelevant facts?
4. Have you reviewed your discussion to make sure that all facts included in the
discussion are also included in your fact statement?
5. Are your facts presented clearly and logically?
6. Is the tone of your Statement of Facts objective?
7. Do you have the procedural history summarized at the end of your statement?

233
Discussion
1. Does your Discussion begin with your overall conclusion, followed by a summary of the
relevant legal principle(s)? Have you addressed “givens” at the outset of your
discussion? [Dernbach pp. 159-163]
2. Have you used "roadmap" sentences/transitions to explain how your sub-issues relate
to one another and to the overall issue? [Dernbach pp. 252-255]
3. Does your organization follow a logical outline, separately discussing all issues and sub-
issues? [Dernbach, pp. 163-169, 174-176]
4. For each issue, sub-issue or factor, have you started with your conclusion (Ci or thesis
sentence), followed by your explanation of the relevant legal principle? Do you have a
roadmap sentence outlining the court’s methodology, if necessary? [Dernbach, pp.
169-171, 220-222, 252-255]
5. Have you provided summaries of the relevant cases, including the relevant facts,
holding, and rationale? Have you summarized the cases before you compare the facts
of the cases to your own? After comparing the case facts with your own, have you
explained why these comparisons are significant under the court’s reasoning and
drawn your conclusion? [Dernbach, pp. 169-171, 222-226]
6. Have you included any necessary counter-analysis, addressing both factual or
analytical points that may lead the court to the opposite conclusion? [Dernbach, pp.
226-231]
7. Have you answered the question asked?
8. Does your final paragraph include your overall conclusion?
9. Is the tone of your Discussion objective?
Conclusion Section
1. Have you stated your conclusion(s) clearly?
2. Have you summarized the applicable legal principle(s)?
3. Have you explained how the law applies to your facts?
4. Have you avoided the use of citation in your Conclusion?
5. Does the structure of your Conclusion mirror the structure of your Discussion?
Grammar and Style
1. Have you confined each paragraph to one idea and developed it carefully? [Dernbach
pp. 247-251]
2. Have you used topic sentences and transitions? [Dernbach, pp. 251-255]
3. Have you used all legal terms correctly and eliminated any unnecessary legalese?
[Dernbach, pp. 265-266]
234
4. Have you used the appropriate verb tense (generally past tense for your facts and case
discussion, present tense for legal rules)?
5. Have you kept your language simple whenever possible and eliminated unnecessary
nominalizations? [Dernbach, pp. 263-264]
6. Have you avoided using the passive voice unless you have a specific reason for doing
so? [Dernbach, pp. 259-260]
7. Have you eliminated unnecessary modifiers and confirmed that the ones you have
used are correctly positioned? [Dernbach, pp. 264-265]
8. Have you checked for basic grammatical errors -- subject/verb disagreement, sentence
fragments, run-on sentences?
9. Do all pronouns agree with their antecedents?
10. Have you checked your punctuation carefully, particularly the use of commas,
apostrophes, semi-colons, and colons?
11. Have you carefully proofread your document for typographical errors?

235
Example 2:

Common Mistakes to Avoid


1. Formatting errors, including using the wrong font, font size, or not attaching the certification
page to the back of your document.

2. Using proper names or citation in the Question Presented.

3. Citation errors, in both full and short form citations (subsequent history, appropriate case name
abbreviations depending on location, appropriate court abbreviations). “Jones at 224.” is not a
correct short form citation.

4. Mentioning a case for the first time and not providing the full citation until the end of the
sentence or placing citations at the very beginning of sentences.

Correct: “In Yellow v. Orange, 221 S.E.2d 202 (Ga. Ct. App. 1967), the court
concluded. . . . .”

Not: “In Yellow v. Orange, the court concluded . . . . “ 221 S.E.2d 202 (Ga. Ct. App.
1967).

Not: “Yellow v. Orange, 221 S.E.2d 202 (Ga. Ct. App. 1967) held that . . ..”

5. String citing cases without a parenthetical explanation.

6. Not using the proper designation of parties:

– Don’t use proper names from decided cases. Not: “In Jones, Mrs. Jones stated . . . .”

– Do refer to your parties by their proper names or use “Plaintiff” or “Defendant” (with no
article).

– Don’t capitalize “court” unless:


 referring to the United States Supreme Court;
 it is part of the proper name of the court: “the Georgia Court of Appeals”;
 you are directly addressing the court: “this Court has held . . . .”

7. Not correcting apostrophe, comma, or other serious grammatical errors:

– Know the difference between “its” and “it’s.” In formal writing you may use the former,
but not the latter.

– Don’t use contractions (it’s, can’t, wasn’t, didn’t, isn’t).

– A court is an “it” not a “they.”

 It may: hold, conclude, reason, rule, state, note, discuss, etc.;


 it does not: argue, feel, or believe.

236
– A corporation or other institution is an “it” not a “they.”

Correct: “The corporation is responsible for the actions of its employees.”

Not: “The corporation is responsible for the actions of their employees.”

– Use correctly formed possessives to save space and be more direct:

Correct: “The defendant’s actions were reasonable.”

Not: “The actions of the defendant were reasonable. “

– Look for and correct comma splices, run-on sentences, sentence fragments.

8. Over-using quotations, particularly quotations from persuasive cases.

9. Paragraphing errors (including not using any):

– No topic sentence.

– Paragraphs are too long, - addressing too many points together.

– Paragraphs too short - not developed in sufficient detail.

10. Not using the appropriate verb tense. Use past tense for decided cases or past actions, but
present tense for legal rules or policy.

Correct: “In Yellow, the passenger had hailed a cab when the defendant pointed
what appeared to be a gun . . . .”

Correct: “The court held the evidence was sufficient to support a finding of self-
defense.”

Correct: “Under the doctrine of self-defense, a person may use deadly force if
he reasonably believes his life is in danger.”

11. Not editing out common wordiness problems. Refer to Chapter 16 in text.

12. Over-using or mis-using literary devices, e.g., rhetorical questions, alliteration, mixed
metaphors. These techniques can be effective when used appropriately and sparingly.

13. Using sarcasm or slang. Maintain a professional tone. Attack the approach or use of policy in a
case, not the case itself or the court that wrote the opinion.

14. Not running a spell checker and particularly checking commonly misspelled words that a spell
checker won’t catch like “statue” instead of “statute” or “trail” instead of “trial.”

237
Example 3:

TRIAL BRIEF GRADING CRITERIA


Course Name ~ Term ~ Professor [ ]

Your trial brief will be evaluated on multiple criteria, many of which are familiar to you from
assessments of work done earlier this term. Those criteria include, but are not limited to:

I. Formatting (Penalty up to 5% of total points available)

Is the document neatly typed in the proper format (font, margins, line spacing, page limit,
page numbers, indentations, etc.)?

Does the document include the required sections, in the proper order?

Does the writer follow specific instructions regarding file name, use of a pseudonym, outline
conventions, typestyles, etc.?

Does the document look professional and readable?

II. Formal Requirements of the Trial Brief (Approx. 20-25 %)

Court Caption

Does the document include the correct caption, including document name and court file
number?

Introduction

Does the Introduction identity the writer & document?

Does the Introduction explain briefly why the writer’s client should prevail on each issue?

Does the Introduction avoid citations to authority, discussions of the law, and argument?

Questions / Issue(s) Presented

Does the writer identify the issues before the court, incorporating both law and fact in each
question?

Does the writer refrain from drawing conclusions in the Questions?

Statements of Facts

Does the Statement of Facts (1) provide context, (2) include the relevant procedural and
factual history of the case, (3) include necessary cites to the record, (4) use persuasive
techniques effectively, and (5) include only honest and accurate information?

Are the parties introduced and referred to with consistent names or terms?

238
Conclusion/Signature Block

Does the writer explicitly request that the court grant the appropriate relief? Is the signature
block included and formatted as shown on the template provided?

III. Argument (Approx. 50-55 %)

Does the writer set out the governing law for each section or sub-section of the Argument?

Does the writer describe the law with effective and accurate case examples so the reader can
understand how the law operates?

When describing precedent, does the writer provide enough information about each
significant authority so the reader is able to understand its relevance to the writer's analysis?

Does the writer explain how the law applies to the facts of our case by making explicit
analogies and distinctions between the precedent cases and our case?

Does the writer explain each step of their analysis, and avoid conclusory arguments?

Does the writer's argument make clear why the court should accept the writer's arguments
on each issue as opposed to the other side's arguments?

Does the writer reach a clear conclusion on each issue and sub-issue discussed?

Does the writer avoid stretching the truth or other inaccuracies when describing the law and
the facts?

Does the writer confront adverse authority anticipate reasonable counter-arguments,


avoiding responses that are conclusory or that simply restate the writer’s original arguments?

IV. Organization (Approx. 7- 10 %)

Does each Question / Issue Presented correspond to a major point heading, and are they in
the same order?

Does the writer use an “umbrella” to provide context for the Argument?

Does the writer use “roadmaps” to guide the reader through the issues and sub-issues to be
discussed, and explain how any “givens” are met and why they will not be discussed?

If the writer uses sub-headings, do they accurately reflect the structure of the analysis of the
issue being discussed?

Does the writer follow CREAC (or an appropriately abbreviated version of CREAC) in the
discussion of each issue and sub-issue ?

Does the writer follow the parts of CREAC in the right order, and keep the parts separate?

Are the issues and sub-issues analyzed separately, not intermingled?

239
Does the writer use topic sentences and transitions to signal organization to the reader and
explain how ideas connect to one another?

Does each paragraph focus on only one main idea?

V. Writing & Editing (Approx.. 7-10 %)

Is the brief concise, direct, and easy to read?

Is legal vocabulary used correctly?

Do all subjects agree with their verbs?

Did the writer use pronouns correctly and avoid vague pronoun references?

Did the writer use proper sentence structure, avoiding sentence fragments & run-on
sentences?

Did the writer use proper punctuation?

Did the writer avoid spelling errors?

Did the writer avoid contractions and slang?

Is there evidence that the document was proofread carefully and completely?

VI. Use of Authority & Citation (Approx.. 7-10 %)

Does the brief demonstrate that the writer performed sufficient research to describe the law
accurately and thoroughly? Will the reader have confidence that the writer knows the law?

Did the writer use authorities effectively to explain how the law operates or is applied in
different factual circumstances?

Did the writer avoid over-relying on too-few cases?

Did the writer use correct citation form throughout, and place citations where needed?

Did the writer use signals and parentheticals properly and effectively?

VII. Professionalism (Penalty up to 5% of total points available)

Did the writer avoid plagiarism and inaccurate descriptions of the law?

240
Appendix II.B
SAMPLE GRADING/CRITERIA SHEETS

Example 1: Exercise 14-A

Name:

/5 Statement of Thesis: Ralph Watson is/is not liable for an assault upon a customer in
Watson’s grocery store committed by Claude Deemer, one of Watson’s employees.

/10 Rule: An employer is liable for assault, an intentional tort, committed by an employee if:
/3 employee has propensity for violence
/3 employer knew/ should have known of propensity
/3 employer had employee work in public place
/1 cite to Tyus or Hersh

/70 Analysis

/23 Propensity for Violence


/2 Thesis: Deemer did/did not have a propensity for violence
/2 Rule: past behavior is evidence of violence. Tyus & Hersh
/4 Case descriptions: In both Tyus & Hersh, customer and salesman, respectively,
assaulted by employees who had been convicted of violent crimes
/6 Application: propensity
/2 Violent hockey play shows propensity
/2 Suspended from play 21 days for unsportsmanlike conduct and fighting
/1 Deemer’s hockey league plagued with violence
/1 Deemer’s playing violent even by league standards

/5 Application: no propensity
/3 Unlike employees in Tyus & Hersh, had not been convicted of violent
crime
/2 Violent hockey play not necessarily indicative of violence off the rink

/3 Reasoning (logic, coherence, clarity)

/1 Conclusion

/26 Knowledge of propensity


/2 Thesis: Watson did/did not know or should/should not have known of Deemer’s
propensity for violence
/1 Rule: [reference to requirement given above]
/4 Case descriptions: In Tyus & Hersh employer did not know and had no reason to
know of prior conviction; had not seen employee behave violently
/9 Application: knew or should have known
/1 Watson was a hockey fan and season ticket holder
/3 Watson had seen Deemer play in game in which he was suspended
/2 Familiar with Deemer, team, and league; league plagued with violence
/3 Distinguish Tyus & Hersh where employers did not know and had no
reason to know of past violent behavior
241
/6 Application: did not know or have reason to know
/3 Unlike employees in Tyus & Hersh, Deemer has no criminal record
/3 Unfair to require employer to equate violent hockey play with propensity
toward violence off the rink

/3 Reasoning (logic, coherence, clarity)

/1 Conclusion

/21 Public place

/2 Thesis: Watson did/did not hire Deemer to work in a public place


/1 Rule: [reference to requirement given above]
/4 Case descriptions: In Tyus, employee hired to work in service station with
regular customer contact. In Hersh, employee worked in place not usually open
to public.
/5 Application: hired to work in public place
/2 Deemer was maintenance worker in grocery store; contact with public
inevitable
/3 Deemer was cleaning trays in the produce department where customers
were usually present and where assault occurred
/5 Application: not hired to work in public place
/2 Unlike employee in Tyus, Deemer not hired for position which required
contact with public
/3 Like employee in Hersh, contact with public only incidental to primary
duties

/3 (reasoning (logic, coherence, clarity)

/1 Conclusion

/4 Conclusion as to broad issue and reasons

/11 Organization and signposts

/100 TOTAL: Statement of Thesis (5 possible) _____

Rule (10 possible _____

Analysis (70 possible) _____


(Propensity _____)
(Knowledge _____)
(Public place _____)

Conclusion (4 possible)

Organization/signposts (11 possible) ______

Suggested letter grade:

242
Example 2: Memo Discussion

This sheet is designed to give students feedback on a draft. The point allocations are
therefore focused on the large concepts addressed in Chapters 11 – 13 of the text: Organization
(Chapter 11), Describing the Law (Chapter 12), and Applying the Law (Chapter 13).

Draft of Two-Issue Discussion Name:

Rules/Descriptions of the Law – 10 points

Description of the law begins with an accurate statement of the governing rule.
Writer describes both the rule and relevant sub-rules, as appropriate.
Writer includes legally significant language from the sources of law.
Rule section(s) do not include improper references to the facts of our problem.
Writer describes precedent cases, when appropriate.
Case examples include relevant facts, court’s reasoning, & holding.
Only the necessary law is described, and no more.

Analysis/Application of Law to Facts – 16 points

Statutory Law
Writer interprets statutes according to their plain meaning.
Writer uses canons and/or case law if necessary to interpret ambiguous terms.
Writer explains relationship between statutory requirements & facts of our problem.
Writer analogizes and distinguishes relevant cases, as appropriate.

Common Law
Writer identifies legal & factual similarities & differences between precedent cases & our
problem.
Analogies & distinctions are explicit – point(s) of comparison not vague or unclear.

Organization & Writing – 8 points

Discussion follows basic CiRAC structure.


Each issue and sub-issue is discussed separately.
Units of discussion begin & end with clear conclusions.
Writer avoids overlong paragraphs & run-on sentences.
Writer uses proper grammar, punctuation, & spelling.
Writer uses proper syntax& vocabulary.
Writer avoids wordiness & repetition.

Citation – 6 points

Writer uses proper citation form.

POINTS EARNED OUT OF 40 POSSIBLE

243
Appendix II.C
SAMPLE GRADING GRIDS/RUBRICS

Example 1: Ungraded Assignment

Technical No Developing Adequate Proficient Strong


Does the paper meet the course
requirements – e.g., page limits, margins,
font size?
Did the writer include the proper caption?
Is paper free of misspellings and typos?
Writing No Developing Adequate Proficient Strong
Did the writer use proper grammar and
punctuation?
Did the writer avoid run-on sentences &
over-long paragraphs?
Did the writer use legal vocabulary
properly?
Did the writer avoid wordiness; are
sentences clear and direct?
Analysis/CiRAC No Developing Adequate Proficient Strong
Did the writer begin with a clear
conclusion that answers the questions
asked: Does Tami have a claim against Ms.
Griffith under the doctrine of attractive
nuisance?
Did the writer end with a clear conclusion
that restates the writer’s conclusion on the
question asked?
Did the writer state the governing rule
clearly, completely, and accurately?
 Quoted accurately?
 Paraphrased accurately?
 Is the structure & operation of the
rule correct?
Did the writer state the synthesized rule on
the “youth” element clearly, completely,
and accurately?
 Are all relevant factors included?
 Are irrelevant facts omitted?
 Are all cases accounted for?
Do case examples clearly explain how the
synthesized rule operates and has been
applied in the past?
Does the writer apply the rule to the facts
as given, without making unjustified
assumptions or drawing unjustified
inferences about the facts?
244
Example 2: Graded Assignment

Section of Memo &/or Skills Considered Needs Low High Strong


Work Average Average
Caption – 1 pt. 0 .5 1
1. Follows instructions/example(s) provided
Introduction – 3 pts. 1 2 3
1. Provides general overview of case
2. Indicates writer’s theme/theory
3. Identifies issues
4. Gives court reason to rule in your favor
Issues/Questions Presented – 3 pts. 1 1.5 2.5 3
1. Properly identify issues for the court
2. Appropriately blend law and fact
3. If subdivided, accurately reflect relation between
questions.
Statement of Facts – 5 pts. 1 2.5 3.5 5
1. Introduces parties, refers to them consistently
2. Includes legally relevant facts
3. Includes key background facts
4. Logically organized
5. Omits irrelevant facts
6. Uses persuasive techniques
Argument Point Headings – 3 pts. 1 1.5 2.5 3
1. Clearly state precise legal assertions
2. If subdivided, accurately identify sub-issues
3. Persuasively blend law/fact/procedural standard
Argument – 23 pts. 12 16.5 19.5 23
1. Sets out summary judgment rule
2. Explains scope of the case, if necessary
3. Sets forth necessary policy considerations, if any
4. States rules accurately and persuasively
(including rules on independent contractors and
integrated enterprises)
5. Describes relevant facts in cases and problem
6. Makes persuasive arguments in favor of client’s
position
7. Explains reasoning clearly & completely
8. Anticipates & responds effectively to reasonable
counterarguments
Organization & Transitions – 4 pts. 1 2 3 4
1. Writer uses appropriate roadmap paragraphs
2. Discussion of each issue & sub-issue follows
CiRAC paradigm
3. Proper topic & thesis sentences
4. Paragraphs do not cover more than one idea
5. Transitions and signposts used to guide reader

245
Writing – 6 pts. 2 3 4.5 6
1. Uses appropriate tone for the document:
respectful & persuasive
2. Concise & direct - no run-on sentences, legalese,
or wordiness
3. Proper grammar, syntax, usage
4. Proper spelling & punctuation
5. Evidence that paper was carefully proofread
6. Effective use of quotes (not over-use)
Research & Use of Authority – 6 pts. 1.5 3 4.5 6
1. Uses adequate authorities for thorough analysis
2. Uses best binding authority, esp. for two main
legal rules/tests
3. Uses authorities effectively for intended purpose
(rule or illustration/example)
4. Does not over-rely on too few authorities
Citation – 5 pts. 1 2.5 3.5 5
1. Proper cites to documents in Statement of Facts
2. Proper long & short forms for cases
3. Proper long & short forms for statutes & rules
4. Proper placement & frequency
5. Uses pinpoint cites when appropriate
6. Uses signals effectively
7. Uses other rules properly – e.g., quotes
Conclusion – 1 pt. 0 .5 1
1. Follows instructions/example(s) provided

TOTAL POINTS POSSIBLE: 60 TOTAL EARNED:

Penalties
1. Format errors - 10 % max. - 6 pts.
2. Professionalism penalties - 10% max. - 6 pts.
(e.g., inaccurate citation, plagiarism)
3. Late penalty - 20% (12 pts.) per day/part of a day TOTAL PENALTIES:
FINAL SCORE: /60

Class median:

246
Appendix III
Other Teaching Aids

247
Appendix III.A
SELF-EVALUATIONS

(Many of these examples can be adapted for


a peer-review exercise)
___________________________________________________________________________________

Example 1: Self-Evaluation of an Early Project or Draft

NAME: ______________________________

ASSIGNMENT: ______________________________

1. What is the best part of your paper? Why?

2. What part of your paper would you have spent more time on if you could? Why?

3. What is the most important thing you will do to improve your next piece of writing?

4. Other comments/questions.

248
Example 2: Self-Evaluation of a Revision or Final Project

NAME: ______________________________

ASSIGNMENT: Revision/Rewrite Self-Evaluation

1. What is the best part of your paper? Why?

2. What were two of your major goals during the revision of this paper? Why were those tasks
important? Do you feel that you succeeded?

3. What editing/revising tools or techniques did you use? Were they helpful? Why or why not?

4. What is the biggest mistake that you made while completing this project? Why did that happen?

5. Other than correcting the mistake identified above, what is the most important thing you will do
to improve your next piece of writing?

249
Example 3: Writing Skills Self-Evaluation

[This example was used for a fall term memorandum. The goals were:
(1) to introduce all students to the resources available in the law school’s writing center,
(2) to require students to read and think about feedback received on their writing, and
(3) to help students at all skill levels begin to improve their writing early in the year.]
]

Instructions: The following steps are to be completed before, during, and after your scheduled
appointment with the law school writing specialist. Please record your answers using
the form posted on the class website.

Step One: Answer the following questions before you meet with the writing specialist.

1. Print out and read your memo from beginning to end. What is your honest reaction? Why?

2. Look for, mark, and/or revise the following on your paper (revisions/rewrites can be written
between lines or at the top or bottom of the appropriate page of your memo):

 Look for and underline passive voice verbs. Was your use of the passive voice really
necessary, or could the sentence have been changed to active voice with no loss of
meaning? Rewrite at least two sentences to eliminate passive voice verbs.

 Look for and underline the pronouns, “they” and “their” – These are plural personal
pronouns but writers often use them incorrectly to substitute for a singular noun. Did you
use “they” and “their” correctly? Make any necessary corrections.

 Highlight the longest three sentences in the memo – In the margin next to each of these
sentences, write the number of words that were in each sentence. If any sentence was
longer than 40 words, it probably should have been shorter. Shorten your long sentences by
removing useless words, or break them into two or more sentences.

 Highlight wordy phrases such as “in order to,” “in regards to,” “at the present time,” “on a
regular basis,” “due to the fact that,” etc. Phrases such as these can easily be replaced with
one word – e.g., “regarding,” “now,” “because,” etc. One verb can usually substitute for a
chain of verbs or verbals: “They are going to have a meeting on Monday” = “They will
meet on Monday.” Replace each wordy phrase with a one- or two-word alternative.

 Underline verb phrases using forms of “to be” and ‘ing’ words.

 Highlight sentences beginning with “There are” or “There is.” “There is/are” is an
imprecise way to begin a sentence. Avoid such constructions in your writing.

3. Circle the paragraph or series of sentences that you feel best about. What do you think makes
that section of your memo stand out?

4. Is your memo – or part of your memo – difficult or confusing to read? What writing habits or
errors can you identify that might account for the confusion?

250
5. Is your memo – or part of your memo – easy to read or to understand? What are you doing as a
writer that makes your memo clear and readable?

6. What is the most important single thing that you can do on your next paper that will have the
greatest positive impact on the quality of your writing?

Step Two: Work through the following questions during your meeting with the writing specialist

7. Ask: Did you find my memo easy to read and understand? Why and/or why not?

8. Ask: What do you think I need to do first to strengthen my writing? What’s the most important
thing for me to do?

Step Three: Make an another appointment with the writing specialist. In the spaces below,
write down the date and time of your appointment and describe your plan for the
meeting

Appointment date & time: _____________________________________________________

9. What do you need to do before the appointment?

10. What are you going to do during the appointment?

251
Appendix III.B
WRITING QUESTIONS PRESENTED

Since the questions presented of a memo alerts the reader to the specific issues
addressed, they should be drafted as precisely as possible.

A. Questions Presented for a Summary-of the Law Memorandum

The questions presented in a summary-of-law-memorandum are defined by the specific


question asked and the law applicable to that question. As indicated in Chapter 18, summary-
of- the-law memos are sometimes written in a short time-frame or to answer a specific question
relevant to a case or client. Thus, the question presented reminds the reader of the question
asked and defines the scope of the memo. This insures that the reader and the writer are on the
same page as to the information included in the memo.

For example, consider the memo you might write in response to an email that states:

“I need to know when an employer may be liable for the assault of a customer by an
employee.”

The question presented for the summary-of-the-law memo written to answer this
question would simply state: “When is an employer liable for the intentional tort of an
employee?” This ensures that the reader understands that the memo only addresses the law
governing liability for intentional torts and excludes any consideration of liability for an
employee’s negligent acts. The question could be even more specific: “When is an employer
liable for an assault committed by and employee?” But the distinction between intentional torts
and negligence may be a more important aspect to highlight, rather than focusing narrowly on
the specific type of intentional tort.

B. Questions Presented for an Analytical Memorandum

The questions presented in an analytical memorandum serve a different purpose


than those in a summary-of-the-law memorandum since they define legal issues.
Precision requires three things of this type of question. Like the summary-of-the-law
memo, it must define the question or issue asked. But it must also state the relevant
legal rule and include the facts legally relevant to the application of the rule.

Thus, these types of questions presented flow from the rules. Consider the analysis
presented in Answer C of Exercise 15-A, regarding and employer’s liability for intentional
torts. The rule, broken down into elements, states that an employer is liable for the intentional
torts of an employee committed at the workplace and causing injury to a member of the public
if (1) the employee has a propensity toward violence, (2) the employer knew or should have
known of that propensity, and (3) the employer had the employee work in a place open to the
public. Assume that elements one and two are at issue and that element three is given. Below is
the process you would use to frame the questions presented for the analytical memo.

252
Step 1

Begin your outline of the Questions Presented by listing the main issue and sub-issues.

Is an employer liable for his employee’s assault on a customer at the employer’s grocery
store?

A. Did the employee have a propensity toward violence?


B. Did the employer know, or should he have known, of the propensity?

Step 2

List the significant facts under each issue and sub-issue.

Employee assaulted a customer at the employer’s grocery store.

A. Employee had no criminal record, plays in hockey league known for


violence, suspended for fighting.

B. Employer attended all games, including one in which employee was


suspended.
Step 3

Combine steps 1 and 2.

Is an employer liable for his employee’s assault on a customer at the employer’s grocery
store?

A. Did the employee have a propensity toward violence when he had no record of
criminal violent behavior but played hockey in a league that was known for
violent play and was suspended for fighting during a game?

B. If so, did the employer know or should he have known of the employee’s
propensity toward violence when he attended all games in which the employee
played, including the game in which the employee was suspended?

In the previous example, there was one issue divided into two sub-issues. Here is an
example of Questions Presented when there are three separate issues and no sub-issues. This
example is not in the text and may be used to make the point that Questions Presented must be
able to be understood by a reader unfamiliar with the law or the facts.

I. Is a parent liable for damages to a school building under an East Dakota statute that
imposes liability on a parent for harm intentionally caused by her child when the child
placed a fire bomb in a car, intending to destroy the car, and the bomb’s explosion
destroyed both the car and a nearby school building?

253
II. If so, does the provision which limits the parent’s liability to $7,500 in damages permit
each of two plaintiffs, the owner of the car and the owner of the school building, to
recover the statutory limit when the harm resulted from a single act?

III. Is a parent liable at common law for negligent supervision of a child who destroyed
property with a fire bomb when the parent had knowledge that the child had
occasionally started fires in the past when he was angry, the parent had obtained
counseling for the child, and the parent relied on the advice of a child therapist
regarding the child’s condition?

You would have arrived at this final version by using the process described above:

Step 1: List the issues.

I. Is the parent liable under the statute for damages to the school building?

II. Can both plaintiffs recover the statutory limit?

III. Is the parent liable under common law for negligent supervision?

Step 2: List the significant facts under each issue.

I. The statute imposes liability on a parent for harm intentionally caused by her
child; the child intentionally placed a bomb in a car in order to destroy it; the
bomb also destroyed a school building.

II. The statute limits the parent’s liability to $7,500; two plaintiffs were harmed by
a single act.

III. The parent knew the child had started fires in the past, had obtained
counseling, and had relied on a therapist’s advice.

Step 3: Combine steps 1 and 2.

[Note that in this case, the parent’s liability under the statute for damage to the car
is a “given” because the child intentionally caused the harm. At issue is whether
she is liable for unintended damages to the school building resulting from an
intentional act.]

254
Appendix III.C
OFFICE MEMORANDA - THE CONCLUSION

The Conclusion section of an office memorandum can be thought of as a more detailed


set of Brief Answers, or a more succinct Discussion. As Prof. Dernbach says: “[A] busy
reader will read the Brief Answers, a reader who has a little more time will read the Conclusion
section, and a reader who has enough time will read the entire memo.” Therefore, your
Conclusion should do a little more than your Brief Answers, but much less than your
Discussion.

Here are some tips for drafting a good Conclusion that will generally hold true – but of
course, as you know by now, in legal analysis there are exceptions to every rule!

DO: Provide closure for your memo and your reader.

Come to a conclusion; predict how the issue(s) are likely to be decided.

Summarize the reasoning supporting your ultimate conclusion or prediction. Briefly


describe the relevant law and explain how it applies to the facts of the case; briefly
explain why the strongest opposing arguments are less likely to prevail.

Organize your Conclusion to mirror the organization of your Questions Presented and
Discussion.

Remember that this is the last section of the memo that your reader will see. Make it
clear and succinct.

DO NOT: Raise new arguments.

Beg the question.

Cite authority

Confuse or mix the issues/sub-issues

255
Appendix III.D
CREATING AN ELEMENTS & ANALYSIS CHART
See text Chapters 7 – 9, Statute & Cases in Teacher’s Manual Exercises 6-F & 6-G

The following pages show how to combine elements of the charts in Chapters 7 through 9
of the text (Identifying Issues for Analysis, Common Law Analysis, and Statutory Analysis) into
a single chart. This can be a useful exercise when students are preparing their first objective
memorandum. In our experience, many students benefit from having to create such a chart as a
pre-writing exercise because it requires them to process and organize all of the material they are
working with, both facts and law. To create a good chart, students must pay special attention to
the specific requirements of the elements of a rule and to the kinds of facts that are legally
relevant to each element.
The following example is based on the shopkeeper’s privilege statute and cases in
Exercises 6-F and 6-G of this Teacher’s Manual, and on the following facts:1

Charles Elmore, age 67, was shopping at Acme Supermarket, where he


attracted the attention of a store security guard, 38-year old Alan Burke. Mr. Elmore
has high blood pressure but is otherwise in good health. Burke is a part-time coach
for a local high school fencing team.
Burke stopped Elmore as he was leaving the market and accused him of
taking a bottle of aspirin, which Elmore had in his jacket pocket. Elmore explained
that he bought the aspirin at a nearby drug store before coming to Acme. Burke told
Elmore that he had been under surveillance and was seen taking the bottle of aspirin
and “ditching” the box while shopping at the market. Elmore stated that he had been
comparing prices but wanted to “clear things up.” As he walked back into the market
with Burke, Burke told the store manager, Nancy Walling, that he had detained a
shoplifter and asked her to call the police. Burke led Elmore to the stock room in the
back of the store and demanded that Elmore give him his jacket. When Elmore
protested and attempted to leave the room, Burke grabbed Elmore’s hands and
handcuffed him to a large metal container, telling Elmore that he had to “preserve the
evidence” until the police arrived.
Elmore told Burke that he had already swallowed three aspirin before Burke
stopped him and asked Burke to count the tablets remaining in the bottle. He also
said that Burke could find the aspirin box and his receipt in a trash can outside the
drug store across the street. He repeated this story to Ms. Walling and a police officer
when they arrived approximately 12 minutes later. Neither Burke nor Walling
counted the aspirin or checked the trash can by the drug store. When the police
officer left with Elmore, Walling searched the market for the aspirin box that Burke
claimed Elmore had “ditched,” but she did not find any such box in the market.
As a result of this incident Elmore sued Acme Supermarket for false
imprisonment. Will the store be able to establish the shopkeeper’s privilege in
defense of Elmore’s action?

A sample elements and analysis chart for this case appears on the following pages.

1
The facts are based in part on Colonial Stores, Inc. v. Fishel, 288 S.E.2d 21 (Ga. Ct. App. 1981).

256
Element Definition/Explanation Relevant Facts from Cases Relevant Facts from Arguments Parties Can
(§ 15-60-7) of Element Your Problem Make on Element

A storeowner Includes agents & employees Security guard(s) – Kovarik Burke is store security guard Not in dispute
(§ 15-60-7) & Williams

Detains Actions, words, fear of Physically taken to office, Burke stopped Elmore. Not in dispute
force? – Kovarik & Williams held in room with door Elmore went to stock room
locked – Williams willingly. Burke physically
restrained Elmore when he
wanted to leave.

a person thought to be Not in dispute


shoplifting and

against that person’s will No action for false imp. if Plaintiff signed consent Elmore went willingly when Burke – Elmore agreed to
and person consents to detention form, did not ask to leave, asked, to “clear things up.” come with him, expressed
– Kovarik wanted to “clear it up” – No consent form. Elmore desire to “clear things up.”
Kovarik later sought to leave, was Burke only restrained
Physical force – Williams handcuffed to a metal Elmore when Elmore tried to
Grabbed by the arm, taken to container. leave; restraint only needed
office, door locked -- to preserve evidence until
Williams Elmore is 67 & has high police came.
blood pressure. Burke is 38
and a fencing coach. Elmore – Posed no threat to
stronger, younger Burke, and
did not need to be physically
restrained. Although initial
detention was not against his
will, it later became so when
he attempted to leave.

that thought is reasonable Burke claims to have seen Burke – Had Elmore under
and Elmore take item and discard surveillance; saw him take
package. item and discard package.

Elmore – Burke was


mistaken; no evidence.

257
Element Definition/Explanation Facts Relevant to Element – Facts relevant to Element – Arguments Parties Can
of Element from Cases from Your Problem Make on Element
person detained sues for false Not in dispute
imprisonment and
(1) A reasonable person Reports from co-workers – Burke saw Elmore remove Burke – See above re:
would believe person was Kovarik bottle from package, discard reasonableness.
shoplifting and wrappings, place item in
Saw shopper place item in pocket, fail to pay. Elmore – See above re:
purse, not pay – Williams reasonableness.
(2) The detention was:

(a) reasonable in No “gratuitous or Threats, abusive language, Burke handcuffed Elmore to Burke – Physical restraint
manner and unnecessary indignities” – sexism = not reasonable – a metal container. necessary because Elmore
Williams Williams was attempting to leave and
evidence would be lost.

Elmore – He posed no escape


risk given disparity in age
and physical condition
between Elmore and Burke.
Physical restraint was
excessive and humiliating.

(b) reasonable in No specific time; can be 10 minutes not reasonable Before handcuffed = not Burke – held Elmore for the
duration affected by manner of when manner unreasonable known. Time handcuffed minimum time needed to
detention – Williams (see above) -- Williams until police arrived – 12 preserve evidence until
minutes. police arrived. Need for this
manner & length of detention
created by Elmore himself.

Elmore - No evidence of any


effort to ascertain the truth
(e.g., look for wrapper).
Unreasonable duration given
manner of detention,
Elmore’s age & medical
condition.

258

You might also like